Różnica pomiędzy stronami "Liczby kwadratowe i niekwadratowe modulo. Wybrane zagadnienia" i "Szeregi liczbowe"

Z Henryk Dąbrowski
(Różnica między stronami)
Przejdź do nawigacji Przejdź do wyszukiwania
 
 
Linia 1: Linia 1:
<div style="text-align:right; font-size: 130%; font-style: italic; font-weight: bold;">22.04.2023</div>
+
<div style="text-align:right; font-size: 130%; font-style: italic; font-weight: bold;">07.04.2022</div>
  
 
__FORCETOC__
 
__FORCETOC__
Linia 5: Linia 5:
  
  
== Przykłady sum symboli Legendre'a ==
+
== Szeregi nieskończone ==
  
<span style="font-size: 110%; font-weight: bold;">Twierdzenie K1</span><br/>
+
<span style="font-size: 110%; font-weight: bold;">Definicja D1</span><br/>
Niech <math>p</math> będzie liczbą pierwszą nieparzystą, <math>a, d \in \mathbb{Z}</math> i <math>p \nmid d</math>. Pokazać, że
+
Sumę wszystkich wyrazów ciągu nieskończonego <math>(a_n)</math>
  
::<math>\sum_{k = 1}^{p - 1} \left( {\small\frac{k}{p}} \right)_{\small{\!\! L}} = \sum_{k = 0}^{p - 1} \left( {\small\frac{k}{p}} \right)_{\small{\!\! L}} = 0</math>
+
::<math>a_1 + a_2 + a_3 + \ldots + a_n + \ldots = \sum_{k = 1}^{\infty} a_k</math>
  
::<math>\sum_{k = 1}^{p - 1} \left( {\small\frac{k^2}{p}} \right)_{\small{\!\! L}} = \sum_{k = 0}^{p - 1} \left( {\small\frac{k^2}{p}} \right)_{\small{\!\! L}} = p - 1</math>
+
nazywamy szeregiem nieskończonym o&nbsp;wyrazach <math>a_n</math>.
  
::<math>\sum_{k = 0}^{p - 1} \left( {\small\frac{a + k d}{p}} \right)_{\small{\!\! L}} = 0</math>
 
  
{{Spoiler|Style = font-style: italic; font-weight: bold; color: olive; text-decoration: underline;|Show=Dowód|Hide=Ukryj dowód}}
 
  
'''Punkt 1.'''
+
<span style="font-size: 110%; font-weight: bold;">Definicja D2</span><br/>
 +
Ciąg <math>S_n = \sum_{k = 1}^{n} a_k</math> nazywamy ciągiem sum częściowych szeregu <math>\sum_{k = 1}^{\infty} a_k</math>.
  
Wystarczy zauważyć, że wśród liczb <math>1, 2, \ldots, p - 1</math> jest <math>{\small\frac{p - 1}{2}}</math> liczb kwadratowych modulo <math>p</math> i <math>{\small\frac{p - 1}{2}}</math> liczb niekwadratowych modulo <math>p</math>. Zatem
 
  
::<math>\sum_{k = 1}^{p - 1} \left( {\small\frac{k}{p}} \right)_{\small{\!\! L}} = {\small\frac{p - 1}{2}} \cdot 1 + {\small\frac{p - 1}{2}} \cdot (- 1) = 0</math>
 
  
'''Punkt 2.'''
+
<span style="font-size: 110%; font-weight: bold;">Definicja D3</span><br/>
 +
Szereg <math>\sum_{k = 1}^{\infty} a_k</math> będziemy nazywali zbieżnym, jeżeli ciąg sum częściowych <math>\left ( S_n \right )</math> jest zbieżny.
  
Wystarczy zauważyć, że
 
  
::<math>\left( {\small\frac{k^2}{p}} \right)_{\small{\!\! L}} = \left( {\small\frac{k}{p}} \right)_{\small{\!\! L}}^{\! 2}</math>
 
  
oraz że wśród liczb <math>1, 2, \ldots, p - 1</math> jest <math>{\small\frac{p - 1}{2}}</math> liczb kwadratowych modulo <math>p</math> i <math>{\small\frac{p - 1}{2}}</math> liczb niekwadratowych modulo <math>p</math>. Zatem
+
<span style="font-size: 110%; font-weight: bold;">Twierdzenie D4 (warunek konieczny zbieżności szeregu)</span><br/>
 +
Jeżeli szereg <math>\sum_{k = 1}^{\infty} a_k</math> jest zbieżny, to <math>\lim_{n \to \infty} a_n = 0</math>.
  
::<math>\sum_{k = 1}^{p - 1} \left( {\small\frac{k}{p}} \right)_{\small{\!\! L}} = {\small\frac{p - 1}{2}} \cdot 1^2 + {\small\frac{p - 1}{2}} \cdot (- 1)^2 = p - 1</math>
+
{{Spoiler|Style = font-style: italic; font-weight: bold; color: olive; text-decoration: underline;|Show=Dowód|Hide=Ukryj dowód}}
 
+
Niech <math>S_n = \sum_{k = 1}^{n} a_k</math> będzie ciągiem sum częściowych, wtedy <math>a_{n + 1} = S_{n + 1} - S_n</math>. Z&nbsp;założenia ciąg <math>(S_n)</math> jest zbieżny, zatem
'''Punkt 3.'''
 
 
 
Z założenia liczby <math>p</math> i <math>d</math> są względnie pierwsze. Z&nbsp;twierdzenia C57 wiemy, że reszty <math>r_1, r_2, \ldots, r_p</math> z&nbsp;dzielenia <math>p</math> kolejnych liczb postaci
 
 
 
::<math>x_k = a + k d</math>
 
  
przez liczbę <math>p</math> są wszystkie różne i&nbsp;tworzą zbiór <math>S = \{ 0, 1, \ldots, p - 1 \}</math>. Czyli wśród reszt <math>r_1, r_2, \ldots, r_p</math> jest <math>{\small\frac{p - 1}{2}}</math> liczb kwadratowych modulo <math>p</math>, tyle samo liczb niekwadratowych modulo <math>p</math>, a&nbsp;jedna z&nbsp;tych reszt jest podzielna przez <math>p</math>. Z&nbsp;własności symbolu Legendre'a wiemy, że licznik wpływa na wartość symbolu jedynie modulo mianownik (zobacz J33 p. 2). Zatem możemy napisać
+
::<math>\lim_{n \to \infty} a_{n + 1} = \lim_{n \to \infty} \left ( S_{n+1} - S_{n} \right ) = \lim_{n \to \infty} S_{n + 1} - \lim_{n \to \infty} S_n = 0</math><br/>
 
 
::<math>\sum_{k = 0}^{p - 1} \left( {\small\frac{a + k d}{p}} \right)_{\small{\!\! L}}
 
= \sum_{j = 0}^{p - 1} \left( {\small\frac{r_j}{p}} \right)_{\small{\!\! L}}
 
= {\small\frac{p - 1}{2}} \cdot 1 + {\small\frac{p - 1}{2}} \cdot (- 1) + 0
 
= 0</math>
 
 
 
Co należało pokazać.<br/>
 
 
&#9633;
 
&#9633;
 
{{\Spoiler}}
 
{{\Spoiler}}
Linia 53: Linia 38:
  
  
<span style="font-size: 110%; font-weight: bold;">Twierdzenie K2* (George Pólya, Iwan Winogradow, 1918)</span><br/>
+
Okazuje się, że bardzo łatwo podać przykład szeregów, dla których warunek <math>\lim_{n \to \infty} a_n = 0</math> jest warunkiem wystarczającym. Opisany w&nbsp;poniższym twierdzeniu rodzaj szeregów nazywamy szeregami naprzemiennymi.<br/>
Jeżeli <math>p</math> jest liczbą pierwszą nieparzystą i <math>m, n \in \mathbb{N}_0</math>, to prawdziwe jest oszacowanie
+
<span style="font-size: 110%; font-weight: bold;">Twierdzenie D5 (kryterium Leibniza)</span><br/>
 
+
Niech ciąg <math>(a_n)</math> będzie ciągiem malejącym o&nbsp;wyrazach nieujemnych. Jeżeli
::<math>\left| \sum_{t = m}^{m + n} \left( {\small\frac{t}{p}} \right)_{\small{\!\! L}} \right| < \sqrt{p} \log p</math>
 
 
 
  
 +
::<math>\underset{n \rightarrow \infty}{\lim} a_n = 0</math>
  
<span style="font-size: 110%; font-weight: bold;">Twierdzenie K3</span><br/>
+
to szereg <math>\underset{k = 1}{\overset{\infty}{\sum}} (- 1)^{k + 1} \cdot a_k</math> jest zbieżny.
Jeżeli <math>p</math> jest liczbą pierwszą nieparzystą i <math>a, b \in \mathbb{Z}</math>, to
 
 
 
::<math>\sum_{k = 0}^{p - 1} \left( {\small\frac{k + a}{p}} \right)_{\small{\!\! L}} \left( {\small\frac{k + b}{p}} \right)_{\small{\!\! L}}
 
= \begin{cases}
 
\;\;\:\,      - 1 & \text{gdy } \, p \nmid (a - b) \\
 
    p - 1 & \text{gdy } \, p \mid (a - b) \\
 
\end{cases}</math>
 
  
 
{{Spoiler|Style = font-style: italic; font-weight: bold; color: olive; text-decoration: underline;|Show=Dowód|Hide=Ukryj dowód}}
 
{{Spoiler|Style = font-style: italic; font-weight: bold; color: olive; text-decoration: underline;|Show=Dowód|Hide=Ukryj dowód}}
 +
Grupując wyrazy szeregu po dwa, otrzymujemy sumę częściową postaci
  
'''1. Przypadek, gdy <math>\boldsymbol{p \mid (a - b)}</math>'''
+
::<math>S_{2 m} = (a_1 - a_2) + (a_3 - a_4) + \ldots + (a_{2 m - 1} - a_{2 m})</math>
  
Z założenia <math>b \equiv a \!\! \pmod{p}</math>
+
Ponieważ ciąg <math>(a_n)</math> jest ciągiem malejącym, to każde wyrażenie w&nbsp;nawiasie jest liczbą nieujemną. Z&nbsp;drugiej strony
  
::<math>\sum_{k = 0}^{p - 1} \left( {\small\frac{k + a}{p}} \right)_{\small{\!\! L}} \left( {\small\frac{k + b}{p}} \right)_{\small{\!\! L}}
+
::<math>S_{2 m} = a_1 - (a_2 - a_3) - (a_4 - a_5) - \ldots - (a_{2 m - 2} - a_{2 m - 1}) {- a_{2 m}} < a_1</math>
= \sum_{k = 0}^{p - 1} \left( {\small\frac{k + a}{p}} \right)_{\small{\!\! L}} \left( {\small\frac{k + a}{p}} \right)_{\small{\!\! L}}
 
= \sum_{k = 0}^{p - 1} \left( {\small\frac{k + a}{p}} \right)_{\small{\!\! L}}^{\! 2}</math>
 
  
Z&nbsp;twierdzenia C57 wiemy, że reszty <math>r_1, r_2, \ldots, r_p</math> z&nbsp;dzielenia <math>p</math> kolejnych liczb postaci
+
Zatem dla każdego <math>m</math> ciąg sum częściowych <math>S_{2 m}</math> jest rosnący i&nbsp;ograniczony od góry, skąd na mocy twierdzenia C10 jest zbieżny, czyli
  
::<math>x_k = a + k</math>
+
::<math>\lim_{m \to \infty} S_{2 m} = g</math>
  
przez liczbę <math>p</math> są wszystkie różne i&nbsp;tworzą zbiór <math>S = \{ 0, 1, \ldots, p - 1 \}</math>. Czyli wśród reszt <math>r_1, r_2, \ldots, r_p</math> jest <math>{\small\frac{p - 1}{2}}</math> liczb kwadratowych modulo <math>p</math>, tyle samo liczb niekwadratowych modulo <math>p</math>, a&nbsp;jedna z&nbsp;tych reszt jest podzielna przez <math>p</math>. Z&nbsp;własności symbolu Legendre'a wiemy, że licznik wpływa na wartość symbolu jedynie modulo mianownik (zobacz J33 p. 2). Zatem możemy napisać
+
Pozostaje zbadać sumy częściowe <math>S_{2 m + 1}</math>. Rezultat jest natychmiastowy
  
::<math>\sum_{k = 0}^{p - 1} \left( {\small\frac{k + a}{p}} \right)_{\small{\!\! L}}^{\! 2}
+
::<math>\lim_{m \to \infty} S_{2 m + 1} = \lim_{m \to \infty} (S_{2 m} + a_{2 m + 1}) = \lim_{m \to \infty} S_{2 m} + \lim_{m \to \infty} a_{2 m + 1} = g + 0 = g</math>
= \sum_{k = 0}^{p - 1} \left( {\small\frac{r_k}{p}} \right)_{\small{\!\! L}}^{\! 2}
 
= p - 1</math>
 
  
Co należało pokazać.
+
Co kończy dowód.<br/>
 
 
'''2. Przypadek, gdy <math>\boldsymbol{p \nmid (a - b)}</math>'''
 
 
 
Kładąc <math>j = k + a</math> i&nbsp;sumując od <math>a</math> do <math>p - 1 + a</math>, otrzymujemy
 
 
 
::<math>\sum_{k = 0}^{p - 1} \left( {\small\frac{k + a}{p}} \right)_{\small{\!\! L}} \left( {\small\frac{k + b}{p}} \right)_{\small{\!\! L}}
 
= \sum_{j = a}^{p - 1 + a} \left( {\small\frac{j}{p}} \right)_{\small{\!\! L}} \left( {\small\frac{j + b - a}{p}} \right)_{\small{\!\! L}}</math>
 
 
 
Wśród <math>p</math> kolejnych liczb <math>a, a + 1, \ldots, p - 1 + a</math> istnieje dokładnie jedna liczba podzielna przez <math>p</math>. Możemy ją pominąć, bo nie wnosi ona wkładu do wyliczanej sumy.
 
 
 
::<math>\sum_{k = 0}^{p - 1} \left( {\small\frac{k + a}{p}} \right)_{\small{\!\! L}} \left( {\small\frac{k + b}{p}} \right)_{\small{\!\! L}}
 
= \underset{p \nmid j}{\sum_{j = a}^{p - 1 + a}} \left( {\small\frac{j}{p}} \right)_{\small{\!\! L}} \left( {\small\frac{j + b - a}{p}} \right)_{\small{\!\! L}}</math>
 
 
 
::::::::<math>\;\;\, = \underset{p \nmid j}{\sum_{j = a}^{p - 1 + a}} \left( {\small\frac{j}{p}} \right)_{\small{\!\! L}} \left( {\small\frac{j + (b - a) j j^{- 1}}{p}} \right)_{\small{\!\! L}}</math>
 
 
 
::::::::<math>\;\;\, = \underset{p \nmid j}{\sum_{j = a}^{p - 1 + a}} \left( {\small\frac{j^2}{p}} \right)_{\small{\!\! L}} \left( {\small\frac{1 + (b - a) j^{- 1}}{p}} \right)_{\small{\!\! L}}</math>
 
 
 
::::::::<math>\;\;\, = \underset{p \nmid j}{\sum_{j = a}^{p - 1 + a}} \left( {\small\frac{1 + (b - a) j^{- 1}}{p}} \right)_{\small{\!\! L}}</math>
 
 
 
Z własności symbolu Legendre'a wiemy, że licznik wpływa na wartość symbolu jedynie modulo mianownik. Liczby <math>j = k + a</math>, gdzie <math>k = 0, 1, \ldots, p - 1</math>, są wszystkie różne modulo <math>p</math> (zobacz H20). Niech zbiór <math>S</math> będzie zbiorem wszystkich liczb <math>j = k + a</math>, które nie są podzielne przez <math>p</math>. Na mocy twierdzenia H25 zbiory <math>R = \{ 1, \ldots, p - 1 \}</math>, <math>S</math> oraz <math>T = \{ s^{- 1}_1, \ldots, s^{- 1}_{p - 1} \}</math>, gdzie <math>s_k \in S</math>, są równe modulo <math>p</math>. Zatem od sumowania po <math>j</math> możemy przejść do sumowania po <math>r \in R</math>.
 
 
 
::<math>\sum_{k = 0}^{p - 1} \left( {\small\frac{k + a}{p}} \right)_{\small{\!\! L}} \left( {\small\frac{k + b}{p}} \right)_{\small{\!\! L}}
 
= \sum_{r = 1}^{p - 1} \left( {\small\frac{1 + (b - a) r}{p}} \right)_{\small{\!\! L}}</math>
 
 
 
::::::::<math>\;\;\, = - \left( {\small\frac{1}{p}} \right)_{\small{\!\! L}} + \sum_{r = 0}^{p - 1} \left( {\small\frac{1 + (b - a) r}{p}} \right)_{\small{\!\! L}}</math>
 
 
 
::::::::<math>\;\;\, = - 1</math>
 
 
 
Ostatnia z&nbsp;wypisanych sum jest równa zero, co wynika z&nbsp;trzeciego wzoru twierdzenia K1 i&nbsp;faktu, że <math>p \nmid (b - a)</math>. Co należało pokazać.<br/>
 
 
&#9633;
 
&#9633;
 
{{\Spoiler}}
 
{{\Spoiler}}
Linia 124: Linia 69:
  
  
<span style="font-size: 110%; font-weight: bold;">Twierdzenie K4</span><br/>
+
<span style="font-size: 110%; font-weight: bold;">Twierdzenie D6</span><br/>
Jeżeli <math>p</math> jest liczbą pierwszą nieparzystą i <math>n \in \mathbb{Z}</math>, to
+
Dla <math>s > 1</math> prawdziwy jest następujący związek
  
::<math>\sum_{k = 0}^{p - 1} \left( {\small\frac{k^2 + n}{p}} \right)_{\small{\!\! L}} =
+
::<math>\sum_{k = 1}^{\infty} \frac{(- 1)^{k + 1}}{k^s} = (1 - 2^{1 - s}) \sum_{k = 1}^{\infty} \frac{1}{k^s}</math>
\begin{cases}
 
\;\;\:\,      - 1 & \text{gdy } \, p \nmid n \\
 
    p - 1 & \text{gdy } \, p \mid n \\
 
\end{cases}</math>
 
  
 
{{Spoiler|Style = font-style: italic; font-weight: bold; color: olive; text-decoration: underline;|Show=Dowód|Hide=Ukryj dowód}}
 
{{Spoiler|Style = font-style: italic; font-weight: bold; color: olive; text-decoration: underline;|Show=Dowód|Hide=Ukryj dowód}}
 +
Zauważmy, że założenie <math>s > 1</math> zapewnia zbieżność szeregu po prawej stronie. Zapiszmy szereg <math>\sum_{k = 1}^{\infty} \frac{1}{k^s}</math> w&nbsp;postaci sumy dla <math>k</math> parzystych i&nbsp;nieparzystych
  
'''Przypadek, gdy <math>\boldsymbol{p \mid n}</math>
+
::<math>\sum_{k = 1}^{\infty} \frac{1}{k^s} = 1 + \frac{1}{2^s} + \frac{1}{3^s} + \frac{1}{4^s} + \frac{1}{5^s} + \ldots =</math>
  
Z drugiego wzoru twierdzenia K1 otrzymujemy
+
::::<math>\:\, = \sum_{k = 1}^{\infty} \frac{1}{(2 k - 1)^s} + \sum_{k = 1}^{\infty} \frac{1}{(2 k)^s} =</math>
  
::<math>\sum_{k = 0}^{p - 1} \left( {\small\frac{k^2 + n}{p}} \right)_{\small{\!\! L}} = \sum_{k = 0}^{p - 1} \left( {\small\frac{k^2}{p}} \right)_{\small{\!\! L}} = p - 1</math>
+
::::<math>\:\, = \sum_{k = 1}^{\infty} \frac{1}{(2 k - 1)^s} + \frac{1}{2^s} \sum_{k = 1}^{\infty} \frac{1}{k^s}</math>
  
'''Przypadek, gdy <math>\boldsymbol{p \nmid n}</math>
+
Otrzymujemy wzór
  
Jeżeli liczby <math>a, b</math> są obie liczbami kwadratowymi lub obie liczbami niekwadratowymi modulo <math>p</math>, to istnieje taka liczba <math>r</math>, że
+
::<math>\sum_{k = 1}^{\infty} \frac{1}{(2 k - 1)^s} = (1 - 2^{- s}) \sum_{k = 1}^{\infty} \frac{1}{k^s}</math>
  
::<math>a \equiv b r^2 \!\! \pmod{p}</math>
 
  
(zobacz J34). Zatem
+
Podobnie rozpiszmy szereg naprzemienny
  
::<math>S(a) = \sum_{k = 0}^{p - 1} \left( {\small\frac{k^2 + a}{p}} \right)_{\small{\!\! L}}</math>
+
::<math>\sum_{k = 1}^{\infty} \frac{(- 1)^{k + 1}}{k^s} = 1 - \frac{1}{2^s} + \frac{1}{3^s} - \frac{1}{4^s} + \frac{1}{5^s} - \ldots =</math>
  
:::<math>\;\;\; = \sum^{p - 1}_{k = 0} \left( {\small\frac{k^2 + b r^2}{p}} \right)_{\small{\!\! L}}</math>
+
::::::<math>= \sum_{k = 1}^{\infty} \frac{1}{(2 k - 1)^s} - \sum_{k = 1}^{\infty} \frac{1}{(2 k)^s} =</math>
  
:::<math>\;\;\; = \sum_{k = 0}^{p - 1} \left( {\small\frac{r^2 \left[ (k r^{- 1})^2 + b \right] }{p}} \right)_{\small{\!\! L}}</math>
+
::::::<math>= (1 - 2^{- s}) \sum_{k = 1}^{\infty} \frac{1}{k^s} - \frac{1}{2^s} \sum_{k = 1}^{\infty} \frac{1}{k^s} =</math>
  
:::<math>\;\;\; = \left( {\small\frac{r^2}{p}} \right)_{\small{\!\! L}} \sum_{k = 0}^{p - 1} \left( {\small\frac{(k r^{- 1})^2 + b}{p}} \right)_{\small{\!\! L}}</math>
+
::::::<math>= (1 - 2^{1 - s}) \sum_{k = 1}^{\infty} \frac{1}{k^s}</math>
  
:::<math>\;\;\; = \sum_{k = 0}^{p - 1} \left( {\small\frac{(k r^{- 1})^2 + b}{p}} \right)_{\small{\!\! L}}</math>
+
gdzie skorzystaliśmy ze znalezionego wyżej wzoru dla sumy szeregu <math>\sum_{k = 1}^{\infty} \frac{1}{(2 k - 1)^s}</math><br/>
 +
&#9633;
 +
{{\Spoiler}}
  
Z twierdzenia C57 wiemy, że gdy <math>k</math> przebiega zbiór <math>T = \{ 0, 1, \ldots, p - 1 \}</math>, to <math>k r^{- 1}</math> przebiega zbiór <math>T'</math> identyczny ze zbiorem <math>T</math> modulo <math>p</math>. Zatem
 
  
::<math>S(a) = \sum_{x = 0}^{p - 1} \left( {\small\frac{x^2 + b}{p}} \right)_{\small{\!\! L}} = S (b)</math>
 
  
 +
<span style="font-size: 110%; font-weight: bold;">Przykład D7</span><br/>
 +
Szeregi niekończone często definiują ważne funkcje. Dobrym przykładem może być funkcja eta Dirichleta<ref name="DirichletEta"/>, którą definiuje szereg naprzemienny
  
Wynika stąd, że dla wszystkich liczb kwadratowych (odpowiednio niekwadratowych) modulo <math>p</math> wyrażenie <math>S(n)</math> ma taką samą wartość i&nbsp;jeśli wybierzemy liczby <math>a, b</math> tak, aby jedna była liczbą kwadratową, a&nbsp;druga liczbą niekwadratową modulo <math>p</math>, to możemy napisać
+
::<math>\eta (s) = \sum_{k = 1}^{\infty} \frac{(- 1)^{k + 1}}{k^s}</math>
  
::<math>\sum_{n = 1}^{p - 1} S (n) = {\small\frac{p - 1}{2}} (S (a) + S (b))</math>
+
lub funkcja dzeta Riemanna<ref name="RiemannZeta"/>, którą definiuje inny szereg
  
 +
::<math>\zeta (s) = \sum_{k = 1}^{\infty} \frac{1}{k^s}</math>
  
Z drugiej strony
+
Na podstawie twierdzenia D6 funkcje te są związane wzorem
  
::<math>\sum_{n = 1}^{p - 1} S (n) = \sum_{n = 1}^{p - 1} \sum_{k = 0}^{p - 1} \left( {\small\frac{k^2 + n}{p}} \right)_{\small{\!\! L}}</math>
+
::<math>\eta (s) = (1 - 2^{1 - s}) \zeta (s)</math>
  
::::<math>\;\;\;\: = \sum_{k = 0}^{p - 1} \sum_{n = 1}^{p - 1} \left( {\small\frac{k^2 + n}{p}} \right)_{\small{\!\! L}}</math>
+
Dla <math>s \in \mathbb{R}_+</math> funkcja eta Dirichleta jest zbieżna. Możemy ją wykorzystać do znajdowania sumy szeregu naprzemiennego <math>\sum_{k = 1}^{\infty} \frac{(- 1)^{k + 1}}{k^s}</math>.
  
::::<math>\;\;\;\: = \sum_{k = 0}^{p - 1} \left[ - \left( {\small\frac{k^2}{p}} \right)_{\small{\!\! L}} + \sum_{n = 0}^{p - 1} \left( {\small\frac{k^2 + n}{p}} \right)_{\small{\!\! L}} \right]</math>
+
::{| class="wikitable plainlinks"  style="font-size: 100%; text-align: left; margin-right: auto;"
 +
|-
 +
| <math>s = \frac{1}{2}</math>
 +
| <math>\sum_{k = 1}^{\infty} \frac{(- 1)^{k + 1}}{\sqrt{k}} = 0.604898643421 \ldots</math>
 +
| [https://www.wolframalpha.com/input/?i=DirichletEta%5B1%2F2%5D WolframAlpha]
 +
|-
 +
| <math>s = 1</math>
 +
| <math>\sum_{k = 1}^{\infty} \frac{(- 1)^{k + 1}}{k} = \log 2 = 0.693147180559 \ldots</math>
 +
| [https://www.wolframalpha.com/input/?i=DirichletEta%5B1%5D WolframAlpha]
 +
|-
 +
| <math>s = 2</math>
 +
| <math>\sum_{k = 1}^{\infty} \frac{(- 1)^{k + 1}}{k^2} = \frac{\pi^2}{12} = 0.822467033424 \ldots</math>
 +
| [https://www.wolframalpha.com/input/?i=DirichletEta%5B2%5D WolframAlpha]
 +
|}
  
::::<math>\;\;\;\: = - \sum_{k = 0}^{p - 1} \left( {\small\frac{k}{p}} \right)_{\small{\!\! L}}^{\! 2}</math>
 
  
::::<math>\;\;\;\: = - (p - 1)</math>
 
  
bo z&nbsp;twierdzenia K1 wiemy, że
+
<span style="font-size: 110%; font-weight: bold;">Twierdzenie D8</span><br/>
 +
Niech <math>N \in \mathbb{Z}_+</math>. Szeregi <math>\sum_{k = 1}^{\infty} a_k</math> oraz <math>\sum_{k = N}^{\infty} a_k</math> są jednocześnie zbieżne lub jednocześnie rozbieżne. W&nbsp;przypadku zbieżności zachodzi związek
  
::<math>\sum_{n = 0}^{p - 1} \left( {\small\frac{n + k^2}{p}} \right)_{\small{\!\! L}} = 0</math>
+
::<math>\sum_{k = 1}^{\infty} a_k = \left ( a_1 + a_2 + \ldots + a_{N - 1} \right ) + \sum_{k = N}^{\infty} a_k</math>
  
 +
{{Spoiler|Style = font-style: italic; font-weight: bold; color: olive; text-decoration: underline;|Show=Dowód|Hide=Ukryj dowód}}
 +
Niech <math>S(n) =\sum_{k = 1}^{n} a_k</math> (gdzie <math>n \geqslant 1</math>) oznacza sumę częściową pierwszego szeregu, a <math>T(n) = \sum_{k = N}^{\infty} a_k</math> (gdzie <math>n \geqslant N</math>) oznacza sumę częściową drugiego szeregu. Dla <math>n \geqslant N</math> mamy
  
Łącząc uzyskane rezultaty, dostajemy
+
::<math>S(n) = (a_1 + a_2 + \ldots + a_{N - 1}) + T (n)</math>
  
::<math>- (p - 1) = {\small\frac{p - 1}{2}} (S (a) + S (b))</math>
+
Widzimy, że dla <math>n</math> dążącego do nieskończoności zbieżność (rozbieżność) jednego ciągu implikuje zbieżność (rozbieżność) drugiego.<br/>
 +
&#9633;
 +
{{\Spoiler}}
  
Zatem
 
  
::<math>S(a) + S (b) = - 2</math>
 
  
 +
<span style="font-size: 110%; font-weight: bold;">Twierdzenie D9 (kryterium porównawcze)</span><br/>
 +
Jeżeli istnieje taka liczba całkowita <math>N_0</math>, że dla każdego <math>k > N_0</math> jest spełniony warunek
  
Z twierdzenia K3 mamy
+
::<math>0 \leqslant a_k \leqslant b_k</math>
  
::<math>S(- 1) = \sum_{k = 0}^{p - 1} \left( {\small\frac{k^2 - 1}{p}} \right)_{\small{\!\! L}}
+
to
= \sum^{p - 1}_{k = 0} \left( {\small\frac{k - 1}{p}} \right)_{\small{\!\! L}} \left( {\small\frac{k + 1}{p}} \right)_{\small{\!\! L}}
 
= - 1</math>
 
  
bo <math>p \nmid 2</math>. Dla ustalenia uwagi przyjmijmy, że <math>a</math> jest liczbą kwadratową, a <math>b</math> jest liczbą niekwadratową modulo <math>p</math>. Jeżeli <math>- 1</math> jest liczbą kwadratową modulo <math>p</math>, to <math>S(a) = - 1</math> i&nbsp;natychmiast otrzymujemy, że <math>S(b) = - 1</math>. Jeżeli <math>- 1</math> jest liczbą niekwadratową modulo <math>p</math>, to <math>S(b) = - 1</math> i&nbsp;natychmiast otrzymujemy, że <math>S(a) = - 1</math>. Zatem bez względu na to, czy <math>n</math> jest liczbą kwadratową, czy liczbą niekwadratową modulo <math>p</math>, musi być <math>S(n) = - 1</math>. Co należało pokazać.<br/>
+
#&nbsp;&nbsp;&nbsp;zbieżność szeregu <math>\sum_{k = 1}^{\infty} b_k</math> pociąga za sobą zbieżność szeregu <math>\sum_{k = 1}^{\infty} a_k</math>
&#9633;
+
#&nbsp;&nbsp;&nbsp;rozbieżność szeregu <math>\sum_{k = 1}^{\infty} a_k</math> pociąga za sobą rozbieżność szeregu <math>\sum_{k = 1}^{\infty} b_k</math>
{{\Spoiler}}
 
  
 +
{{Spoiler|Style = font-style: italic; font-weight: bold; color: olive; text-decoration: underline;|Show=Dowód|Hide=Ukryj dowód}}
 +
Dowód przeprowadzimy dla szeregów <math>\sum_{k = N_0}^{\infty} a_k</math> oraz <math>\sum_{k = N_0}^{\infty} b_k</math>, które są (odpowiednio) jednocześnie zbieżne lub jednocześnie rozbieżne z&nbsp;szeregami <math>\sum_{k = 1}^{\infty} a_k</math> oraz <math>\sum_{k = 1}^{\infty} b_k</math>.
  
 +
'''Punkt 1.'''<br/>
 +
Z założenia szereg <math>\sum_{k = N_0}^{\infty} b_k</math> jest zbieżny. Niech <math>\sum_{k = N_0}^{\infty} b_k = b</math>, zatem z&nbsp;założonych w&nbsp;twierdzeniu nierówności dostajemy
  
<span style="font-size: 110%; font-weight: bold;">Zadanie K5</span><br/>
+
::<math>0 \leqslant \sum_{k = N_0}^{n} a_k \leqslant \sum_{k = N_0}^{n} b_k \leqslant b</math>
Pokazać, że jeżeli <math>p</math> jest liczbą pierwszą nieparzystą i <math>r , s \in \mathbb{Z}</math>, to
 
  
::<math>\sum_{k = 0}^{p - 1} \left( {\small\frac{k^2 + r k + s}{p}} \right)_{\small{\!\! L}} =
+
Zauważmy, że ciąg sum częściowych <math>A_n = \sum_{k = N_0}^{n} a_k</math> jest ciągiem rosnącym (bo <math>a_k \geqslant 0</math>) i&nbsp;ograniczonym od góry. Wynika stąd, że ciąg <math>\left ( A_n \right )</math> jest zbieżny, zatem szereg <math>\sum_{k = N_0}^{\infty} a_k</math> jest zbieżny.
\begin{cases}
 
\;\;\:\,      - 1 & \text{gdy } \, p \nmid (r^2 - 4 s) \\
 
    p - 1 & \text{gdy } \, p \mid (r^2 - 4 s) \\
 
\end{cases}</math>
 
  
{{Spoiler|Style = font-style: italic; font-weight: bold; color: olive; text-decoration: underline;|Show=Rozwiązanie|Hide=Ukryj rozwiązanie}}
+
'''Punkt 2.'''<br/>
 +
Z założenia szereg <math>\sum_{k = N_0}^{\infty} a_k</math> jest rozbieżny, a&nbsp;z&nbsp;założonych w&nbsp;twierdzeniu nierówności dostajemy
  
::<math>\sum_{k = 0}^{p - 1} \left( {\small\frac{k^2 + r k + s}{p}} \right)_{\small{\!\! L}} = \sum_{k = 0}^{p - 1} \left( {\small\frac{2^2}{p}} \right)_{\small{\!\! L}} \left( {\small\frac{k^2 + r k + s}{p}} \right)_{\small{\!\! L}}</math>
+
::<math>0 \leqslant \sum_{k = N_0}^{n} a_k \leqslant \sum_{k = N_0}^{n} b_k</math>
  
:::::::<math>\;\;\;\, = \sum_{k = 0}^{p - 1} \left( {\small\frac{4 k^2 + 4 r k + 4 s}{p}} \right)_{\small{\!\! L}}</math>
+
Rosnący ciąg sum częściowych <math>A_n = \sum_{k = N_0}^{n} a_k</math> nie może być ograniczony od góry, bo przeczyłoby to założeniu, że szereg <math>\sum_{k = N_0}^{\infty} a_k</math> jest rozbieżny. Wynika stąd i&nbsp;z&nbsp;wypisanych wyżej nierówności, że również ciąg sum częściowych <math>B_n = \sum_{k = N_0}^{n} b_k</math> nie może być ograniczony od góry, zatem szereg <math>\sum_{k = N_0}^{\infty} b_k</math> jest rozbieżny.<br/>
 
 
:::::::<math>\;\;\;\, = \sum^{p - 1}_{k = 0} \left( {\small\frac{(2 k + r)^2 + 4 s - r^2}{p}} \right)_{\small{\!\! L}}</math>
 
 
 
Z twierdzenia C57 wiemy, że gdy <math>k</math> przebiega zbiór <math>T = \{ 0, 1, \ldots, p - 1 \}</math>, to <math>2 k + r</math> przebiega zbiór <math>T'</math> identyczny ze zbiorem <math>T</math> modulo <math>p</math>. Zatem
 
 
 
::<math>\sum_{k = 0}^{p - 1} \left( {\small\frac{k^2 + r k + s}{p}} \right)_{\small{\!\! L}} = \sum_{x = 0}^{p - 1} \left( {\small\frac{x^2 + 4 s - r^2}{p}} \right)_{\small{\!\! L}}</math>
 
 
 
Z twierdzenia K4 wynika natychmiast teza dowodzonego twierdzenia.<br/>
 
 
&#9633;
 
&#9633;
 
{{\Spoiler}}
 
{{\Spoiler}}
Linia 234: Linia 183:
  
  
<span style="font-size: 110%; font-weight: bold;">Twierdzenie K6</span><br/>
+
<span style="font-size: 110%; font-weight: bold;">Twierdzenie D10</span><br/>
Jeżeli <math>p</math> jest liczbą pierwszą nieparzystą i <math>n \in \mathbb{Z}</math>, to dla sumy
+
Jeżeli szereg <math>\sum_{k = 1}^{\infty} \left | a_k  \right |</math> jest zbieżny, to szereg <math>\sum_{k = 1}^{\infty} a_k</math> jest również zbieżny.
 
 
::<math>S(n) = \sum_{k = 0}^{p - 1} \left( {\small\frac{k (k^2 + n)}{p}} \right)_{\small{\!\! L}}</math>
 
 
 
prawdziwe są następujące wzory
 
 
 
::(a) <math>\;\; S(n) = 0 \qquad \qquad \text{gdy } \; p = 4 k + 3</math>
 
 
 
::(b) <math>\;\; | S (n) | < 2 \sqrt{p} \qquad \text{gdy } \; p = 4 k + 1</math>
 
  
 
{{Spoiler|Style = font-style: italic; font-weight: bold; color: olive; text-decoration: underline;|Show=Dowód|Hide=Ukryj dowód}}
 
{{Spoiler|Style = font-style: italic; font-weight: bold; color: olive; text-decoration: underline;|Show=Dowód|Hide=Ukryj dowód}}
 +
Niech <math>b_k = a_k + | a_k |</math>. Z&nbsp;definicji prawdziwe jest następujące kryterium porównawcze
  
'''Punkt (a)'''
+
::<math>0 \leqslant b_k \leqslant 2 | a_k |</math>
 
 
Zauważmy, że zbiory <math>R = \{ 0, 1, 2, \ldots, p - 1 \}</math> oraz <math>T = \{ - p + 1, - p + 2, \ldots, - p + (p - 1), 0 \}</math> są identyczne modulo <math>p</math>. Z&nbsp;własności symbolu Legendre'a wiemy, że licznik wpływa na wartość symbolu jedynie modulo mianownik (zobacz J33 p.2). Zatem możemy sumowanie po <math>k \in R</math> zastąpić sumowaniem po <math>j \in T .</math> Otrzymujemy
 
 
 
::<math>S(n) = \sum_{j = - p + 1}^{0} \left( {\small\frac{j (j^2 + n)}{p}} \right)_{\small{\!\! L}}</math>
 
 
 
Kładąc <math>j = - r</math> i&nbsp;sumując po <math>r</math> od <math>0</math> do <math>p - 1</math>, dostajemy
 
 
 
::<math>S(n) = \sum_{r = 0}^{p - 1} \left( {\small\frac{- r}{p}} \right)_{\small{\!\! L}} \left( {\small\frac{(- r)^2 + n}{p}} \right)_{\small{\!\! L}}
 
= \sum_{r = 0}^{p - 1} \left( {\small\frac{- 1}{p}} \right)_{\small{\!\! L}} \left( {\small\frac{r}{p}} \right)_{\small{\!\! L}} \left( {\small\frac{r^2 + n}{p}} \right)_{\small{\!\! L}} 
 
= \left( {\small\frac{- 1}{p}} \right)_{\small{\!\! L}} S (n)</math>
 
 
 
Jeżeli <math>p = 4 k + 3</math>, to <math>S (n) = - S (n)</math>, czyli <math>S(n) = 0</math>.
 
 
 
'''Punkt (b)'''
 
 
 
Pomysł dowodu zaczerpnęliśmy z materiałów szkoleniowych Międzynarodowej Olimpiady Matematycznej<ref name="Dukic1"/>.
 
 
 
Jeżeli liczby <math>a, b</math> są obie liczbami kwadratowymi lub obie liczbami niekwadratowymi modulo <math>p</math>, to istnieje taka liczba <math>r</math>, że
 
  
::<math>a \equiv b r^2 \!\! \pmod{p}</math>
+
Zatem z&nbsp;punktu 1. twierdzenia D9 wynika, że szereg <math>\sum_{k = 1}^{\infty} b_k</math> jest zbieżny. Z&nbsp;definicji wyrazów ciągu <math>\left ( b_k \right )</math> mamy <math>a_k = b_k - | a_k |</math> i&nbsp;możemy napisać
  
(zobacz J34). Zatem
+
::<math>\sum_{k = 1}^{\infty} a_k = \sum_{k = 1}^{\infty} b_k - \sum_{k = 1}^{\infty} | a_k |</math>
  
::<math>S(a) = S (b r^2) = \sum_{k = 0}^{p - 1} \left( {\small\frac{k (k^2 + b r^2)}{p}} \right)_{\small{\!\! L}}</math>
+
Ponieważ szeregi po prawej stronie są zbieżne, to zbieżny jest też szereg <math>\sum_{k = 1}^{\infty} a_k</math>. Zauważmy, że jedynie w&nbsp;przypadku, gdyby obydwa szeregi po prawej stronie były rozbieżne, nie moglibyśmy wnioskować o&nbsp;zbieżności / rozbieżności szeregu <math>\sum_{k = 1}^{\infty} a_k</math>, bo suma szeregów rozbieżnych może być zbieżna.<br/>
 
 
::::::<math>\;\:\, = \sum_{k = 0}^{p - 1} \left( {\small\frac{r^3 (k r^{- 1}) \left[ (k r^{- 1})^2 + b \right] }{p}} \right)_{\small{\!\! L}}</math>
 
 
 
::::::<math>\;\:\, = \left( {\small\frac{r^3}{p}} \right)_{\small{\!\! L}} \sum_{k = 0}^{p - 1} \left( {\small\frac{(k r^{- 1}) \left[ (k r^{- 1})^2 + b \right] }{p}} \right)_{\small{\!\! L}}</math>
 
 
 
::::::<math>\;\:\, = \left( {\small\frac{r}{p}} \right)_{\small{\!\! L}} \sum_{k = 0}^{p - 1} \left( {\small\frac{(k r^{- 1}) \left[ (k r^{- 1})^2 + b \right] }{p}} \right)_{\small{\!\! L}}</math>
 
 
 
Z twierdzenia C57 wiemy, że gdy <math>k</math> przebiega zbiór <math>T = \{ 0, 1, \ldots, p - 1 \}</math>, to <math>k r^{- 1}</math> przebiega zbiór <math>T'</math> identyczny ze zbiorem <math>T</math> modulo <math>p</math>. Zatem
 
 
 
::<math>S(a) = \left( {\small\frac{r}{p}} \right)_{\small{\!\! L}} \sum_{x = 0}^{p - 1} \left( {\small\frac{x (x^2 + b)}{p}} \right)_{\small{\!\! L}} = \left( {\small\frac{r}{p}} \right)_{\small{\!\! L}} S (b)</math>
 
 
 
Czyli <math>S (a)^2 = S (b)^2</math>. Wynika stąd, że dla wszystkich liczb kwadratowych (odpowiednio niekwadratowych) modulo <math>p</math> wyrażenie <math>S (n)^2</math> ma taką samą wartość i&nbsp;jeśli wybierzemy liczby <math>a, b</math> tak, aby jedna była liczbą kwadratową, a&nbsp;druga liczbą niekwadratową modulo <math>p</math>, to prawdziwa jest równość
 
 
 
::<math>\sum_{n = 1}^{p - 1} S (n)^2 = {\small\frac{p - 1}{2}} (S (a)^2 + S (b)^2)</math>
 
 
 
Jak łatwo zauważyć <math>S(0) = 0</math>, zatem możemy napisać
 
 
 
::<math>\sum_{n = 0}^{p - 1} S (n)^2 = {\small\frac{p - 1}{2}} (S (a)^2 + S (b)^2)</math>
 
 
 
Z drugiej strony
 
 
 
::<math>S (n)^2 = \sum_{k = 1}^{p - 1} \left( {\small\frac{k (k^2 + n)}{p}} \right)_{\small{\!\! L}} \sum^{p - 1}_{j = 1} \left( {\small\frac{j (j^2 + n)}{p}} \right)_{\small{\!\! L}}</math>
 
 
 
:::<math>\quad \,\, = \sum_{k = 1}^{p - 1} \sum_{j = 1}^{p - 1} \left( {\small\frac{k (k^2 + n)}{p}} \right)_{\small{\!\! L}} \left( {\small\frac{j (j^2 + n)}{p}} \right)_{\small{\!\! L}}</math>
 
 
 
:::<math>\quad \,\, = \sum_{k = 1}^{p - 1} \sum_{j = 1}^{p - 1} \left( {\small\frac{k j (k^2 + n) (j^2 + n)}{p}} \right)_{\small{\!\! L}}</math>
 
 
 
Zatem
 
 
 
::<math>\sum_{n = 0}^{p - 1} S (n)^2 = \sum_{n = 0}^{p - 1} \sum_{k = 1}^{p - 1} \sum_{j = 1}^{p - 1} \left( {\small\frac{k j (k^2 + n) (j^2 + n)}{p}} \right)_{\small{\!\! L}}</math>
 
 
 
:::::<math>\;\! = \sum_{k = 1}^{p - 1} \sum_{j = 1}^{p - 1} \left( {\small\frac{k j}{p}} \right)_{\small{\!\! L}} \sum_{n = 0}^{p - 1} \left( {\small\frac{(n + k^2) (n + j^2)}{p}} \right)_{\small{\!\! L}}</math>
 
 
 
 
 
Z twierdzenia K3 wiemy, że
 
 
 
::<math>\sum_{n = 0}^{p - 1} \left( {\small\frac{(n + k^2) (n + j^2)}{p}} \right)_{\small{\!\! L}} =
 
\begin{cases}
 
\;\;\:\,      - 1 & \text{gdy } \, p \nmid (k^2 - j^2) \\
 
    p - 1 & \text{gdy } \, p \mid (k^2 - j^2) \\
 
\end{cases}</math>
 
 
 
 
 
Zbadajmy, kiedy <math>p \mid (k^2 - j^2)</math>, czyli kiedy <math>p \mid [(k - j) (k + j)]</math>. Mamy
 
 
 
::* <math>\; 0 \leqslant | k - j | \leqslant p - 2</math>
 
 
 
::* <math>\; 2 \leqslant k + j \leqslant 2 p - 2</math>
 
 
 
Zatem <math>p \mid [(k - j) (k + j)]</math> gdy
 
 
 
::* <math>\; j = k</math>
 
 
 
::* <math>\; j = p - k</math>
 
 
 
 
 
Pozwala to zapisać rozpatrywaną sumę w&nbsp;postaci
 
 
 
::<math>\sum_{n = 0}^{p - 1} S (n)^2 = \sum_{k = 1}^{p - 1} \sum_{j = 1}^{p - 1} \left( {\small\frac{k j}{p}} \right)_{\small{\!\! L}} \cdot
 
\left\{ \begin{array}{rll}
 
  - 1  & \text{gdy } \; j \neq k \;\;\;\; \text{ i } \;\;\;\; j \neq p - k \\
 
  p - 1 & \text{gdy } \; j = k \;\; \text{ lub } \;\; j = p - k \\
 
\end{array} \right\}</math>
 
 
 
<div style="margin-top: 1em; margin-bottom: 1em;">
 
:::::<math>\:\! = (p - 1) \underset{j = k \; \text{ lub } \; j = p - k}{\sum^{p - 1}_{k = 1} \sum_{j = 1}^{p - 1}} \left( {\small\frac{k j}{p}} \right)_{\small{\!\! L}} - \underset{j \neq k \; \text{ i } \; j \neq p - k}{\sum_{k = 1}^{p - 1} \sum_{j = 1}^{p - 1}} \left( {\small\frac{k j}{p}} \right)_{\small{\!\! L}}</math>
 
</div>
 
 
 
<div style="margin-top: 1em; margin-bottom: 1em;">
 
:::::<math>\:\! = (p - 1) \underset{j = k \; \text{ lub } \; j = p - k}{\sum^{p - 1}_{k = 1} \sum_{j = 1}^{p - 1}} \left( {\small\frac{k j}{p}} \right)_{\small{\!\! L}} - \sum_{k = 1}^{p - 1} \sum_{j = 1}^{p - 1} \left( {\small\frac{k j}{p}} \right)_{\small{\!\! L}} + \underset{j = k \; \text{ lub } \; j = p - k}{\sum_{k = 1}^{p - 1} \sum_{j = 1}^{p - 1}} \left( {\small\frac{k j}{p}} \right)_{\small{\!\! L}}</math>
 
</div>
 
 
 
<div style="margin-top: 1em; margin-bottom: 1em;">
 
:::::<math>\:\! = p \underset{j = k \; \text{ lub } \; j = p - k}{\sum^{p - 1}_{k = 1} \sum_{j = 1}^{p - 1}} \left( {\small\frac{k j}{p}} \right)_{\small{\!\! L}} - \sum_{k = 1}^{p - 1} \sum_{j = 1}^{p - 1} \left( {\small\frac{k}{p}} \right)_{\small{\!\! L}} \left( {\small\frac{j}{p}} \right)_{\small{\!\! L}}</math>
 
</div>
 
 
 
<div style="margin-top: 1em; margin-bottom: 1em;">
 
:::::<math>\:\! = p \left[ \sum_{k = 1}^{p - 1} \left( {\small\frac{k^2}{p}} \right)_{\small{\!\! L}} + \sum_{k = 1}^{p - 1} \left( {\small\frac{k (p - k)}{p}} \right)_{\small{\!\! L}} \right] - \sum_{k = 1}^{p - 1} \left( {\small\frac{k}{p}} \right)_{\small{\!\! L}} \sum^{p - 1}_{j = 1} \left( {\small\frac{j}{p}} \right)_{\small{\!\! L}}</math>
 
</div>
 
 
 
<div style="margin-top: 1em; margin-bottom: 1em;">
 
:::::<math>\:\! = p \left[ (p - 1) + \sum_{k = 1}^{p - 1} \left( {\small\frac{- k^2}{p}} \right)_{\small{\!\! L}} \right]</math>
 
</div>
 
 
 
<div style="margin-top: 1em; margin-bottom: 1em;">
 
:::::<math>\:\! = p \left[ (p - 1) + \left( {\small\frac{-1}{p}} \right)_{\small{\!\! L}} \sum_{k = 1}^{p - 1} \left( {\small\frac{k^2}{p}} \right)_{\small{\!\! L}} \right]</math>
 
</div>
 
 
 
:::::<math>\:\! = 2 p (p - 1)</math>
 
 
 
Zauważmy, że <math>\left( {\small\frac{- 1}{p}} \right)_{\small{\!\! L}} = 1</math>, bo <math>p = 4 k + 1</math>.
 
 
 
 
 
Ponieważ wcześniej pokazaliśmy, że
 
 
 
::<math>\sum_{n = 0}^{p - 1} S (n)^2 = {\small\frac{p - 1}{2}} (S (a)^2 + S (b)^2)</math>
 
 
 
to otrzymujemy
 
 
 
::<math>{\small\frac{p - 1}{2}} (S (a)^2 + S (b)^2) = 2 p (p - 1)</math>
 
 
 
Czyli
 
 
 
::<math>S (a)^2 + S (b)^2 = 4 p</math>
 
 
 
Wynika stąd, że bez względu na to, czy <math>n</math> jest liczbą kwadratową, czy liczbą niekwadratową modulo <math>p</math>, prawdziwe jest oszacowanie
 
 
 
::<math>| S (n) | \leqslant 2 \sqrt{p}</math>
 
 
 
Równość <math>S (n)^2 = 4 p</math> nie jest możliwa, bo dzielnik pierwszy <math>p</math> występuje po prawej stronie w&nbsp;potędze nieparzystej. Zatem mamy nieco silniejsze oszacowanie
 
 
 
::<math>| S (n) | < 2 \sqrt{p}</math>
 
 
 
Co kończy dowód.<br/>
 
 
&#9633;
 
&#9633;
 
{{\Spoiler}}
 
{{\Spoiler}}
Linia 391: Linia 201:
  
  
<span style="font-size: 110%; font-weight: bold;">Twierdzenie K7</span><br/>
+
<span style="font-size: 110%; font-weight: bold;">Twierdzenie D11</span><br/>
Jeżeli <math>p</math> jest liczbą pierwszą nieparzystą i <math>a, b \in \mathbb{Z}</math>, to dla sumy
+
Niech <math>n \in \mathbb{Z}_+</math>. Jeżeli wyrazy ciągu <math>(a_n)</math> można zapisać w&nbsp;jednej z&nbsp;postaci
  
::<math>S(a, b) = \sum_{x = 0}^{p - 1} \left( {\small\frac{x^3 + a x + b}{p}} \right)_{\small{\!\! L}}</math>
+
# <math>\quad a_k = f_k - f_{k + 1}</math>
 +
# <math>\quad a_k = f_{k - 1} - f_k</math>
  
prawdziwe są następujące wzory
+
to odpowiadający temu ciągowi szereg nazywamy szeregiem teleskopowym. Suma częściowa szeregu teleskopowego jest odpowiednio równa
  
:: (a) <math>\;\; S(a, b) = - \left( {\small\frac{6 b}{p}} \right)_{\small{\!\! L}} \qquad \qquad \, \text{gdy } \; p \mid (4 a^3 + 27 b^2)</math>
+
# <math>\quad \sum_{k = m}^{n} a_k = f_m - f_{n + 1}</math>
 
+
# <math>\quad \sum_{k = m}^{n} a_k = f_{m - 1} - f_n</math>
:: (b) <math>\;\; | S (a, b) | < 2 \sqrt{p} \qquad \qquad \;\;\;\; \text{gdy } \; p \nmid (4 a^3 + 27 b^2)</math>
 
  
 
{{Spoiler|Style = font-style: italic; font-weight: bold; color: olive; text-decoration: underline;|Show=Dowód|Hide=Ukryj dowód}}
 
{{Spoiler|Style = font-style: italic; font-weight: bold; color: olive; text-decoration: underline;|Show=Dowód|Hide=Ukryj dowód}}
Niech <math>p \geqslant 5</math>. W&nbsp;ogólnym przypadku interesująca nas suma ma postać
+
::<math>\sum_{k = m}^{n} a_k = \sum_{k = m}^{n} (f_k - f_{k + 1}) =</math>
 
 
::<math>\sum_{t = 0}^{p - 1} \left( {\small\frac{a t^3 + b t^2 + c t + d}{p}} \right)_{\small{\!\! L}}</math>
 
 
 
gdzie <math>p \nmid a</math>. Mnożąc licznik przez <math>a^2</math> nie zmieniamy wartości sumy
 
 
 
::<math>\sum_{t = 0}^{p - 1} \left( {\small\frac{a^3 t^3 + a^2 b t^2 + a^2 c t + a^2 d}{p}} \right)_{\small{\!\! L}}</math>
 
 
 
Podstawiając <math>x \equiv a t + r \!\! \pmod{p}</math>, dostajemy
 
 
 
::<math>\sum_{x = 0}^{p - 1} \left( {\small\frac{x^3 + x^2 (b - 3 r) + x [a c - r (2 b - 3 r)] + [a^2 d - a c r + r^2 (b - r)]}{p}} \right)_{\small{\!\! L}}</math>
 
 
 
bo, gdy <math>t</math> przebiega zbiór <math>\{ 0, 1, \ldots, p - 1 \}</math>, to (modulo <math>p</math>) liczby <math>a t + r</math> przebiegają taki sam zbiór (zobacz C57). Ponieważ <math>p \geqslant 5</math>, to liczbę <math>r</math> możemy wybrać tak, aby było
 
 
 
::<math>3 r \equiv b \!\! \pmod{p}</math>
 
 
 
Ostatecznie otrzymujemy
 
 
 
::<math>\sum_{x = 0}^{p - 1} \left( {\small\frac{x^3 + x (a c - 3 r^2) + (a^2 d - a c r + 2 r^3)}{p}} \right)_{\small{\!\! L}}</math>
 
 
 
 
 
Widzimy, że bez zmniejszania ogólności, możemy ograniczyć się do badania sumy postaci
 
 
 
::<math>S(a, b) = \sum_{x = 0}^{p - 1} \left( {\small\frac{x^3 + a x + b}{p}} \right)_{\small{\!\! L}}</math>
 
 
 
Liczbę <math>- \left( 4 a^3 + 27 b^2 \right)</math> nazywamy wyróżnikiem wielomianu <math>x^3 + a x + b</math>.
 
 
 
Pokażemy, że w&nbsp;przypadku, gdy <math>4 a^3 + 27 b^2 \equiv 0 \!\! \pmod{p}</math> i <math>p \geqslant 3</math> prawdziwy jest wzór
 
 
 
::<math>S(a, b) = \sum_{x = 0}^{p - 1} \left( {\small\frac{x^3 + a x + b}{p}} \right)_{\small{\!\! L}} = - \left( {\small\frac{6 b}{p}} \right)_{\small{\!\! L}}</math>
 
 
 
 
 
W przypadku, gdy <math>p = 3</math> z&nbsp;warunku <math>4 a^3 + 27 b^2 \equiv 0 \pmod{3}</math> wynika, że <math>3 \mid a</math>. Zakładając, że reszta z&nbsp;dzielenia liczby <math>b</math> przez <math>3</math> wynosi <math>r</math>, otrzymujemy
 
 
 
::<math>S(a, b) = \sum_{x = 0}^{2} \left( {\small\frac{x^3 + b}{3}} \right)_{\small{\!\! L}}
 
= \left( {\small\frac{b}{3}} \right)_{\small{\!\! L}} + \left( {\small\frac{1 + b}{3}} \right)_{\small{\!\! L}} + \left( {\small\frac{8 + b}{3}} \right)_{\small{\!\! L}}
 
= \left( {\small\frac{r}{3}} \right)_{\small{\!\! L}} + \left( {\small\frac{r + 1}{3}} \right)_{\small{\!\! L}} + \left( {\small\frac{r + 2}{3}} \right)_{\small{\!\! L}}
 
= \left( {\small\frac{0}{3}} \right)_{\small{\!\! L}} + \left( {\small\frac{1}{3}} \right)_{\small{\!\! L}} + \left( {\small\frac{2}{3}} \right)_{\small{\!\! L}}
 
= 0</math>
 
 
 
 
 
Jeżeli <math>p \geqslant 5</math> i <math>p \mid a</math>, to <math>p \mid b</math> i&nbsp;łatwo znajdujemy, że
 
 
 
::<math>S(a, b) = \sum_{x = 0}^{p - 1} \left( {\small\frac{x^3 + a x + b}{p}} \right)_{\small{\!\! L}}
 
= \sum_{x = 0}^{p - 1} \left( {\small\frac{x^3}{p}} \right)_{\small{\!\! L}}
 
= 0</math>
 
 
 
 
 
Jeżeli <math>p \geqslant 5</math> i <math>p \nmid a</math>, to
 
 
 
::<math>x^3 + a x + b \equiv (x - x_1) (x - x_2)^2 \!\! \pmod{p}</math>
 
 
 
gdzie
 
  
::<math>x_1 \equiv 3 b a^{- 1} \!\! \pmod{p}</math>
+
::::<math>= (f_m - f_{m + 1}) + (f_{m + 1} - f_{m + 2}) + (f_{m + 2} - f_{m + 3}) + \ldots + (f_{n - 1} - f_n) + (f_n - f_{n + 1}) =</math>
  
::<math>x_2 \equiv - 3 b 2^{- 1} a^{- 1} \!\! \pmod{p}</math>
+
::::<math>= f_m - f_{m + 1} + f_{m + 1} - f_{m + 2} + f_{m + 2} - f_{m + 3} + \ldots + f_{n - 1} - f_n + f_n - f_{n + 1} =</math>
  
Co Czytelnik może łatwo sprawdzić, pamiętając o&nbsp;tym, że <math>27 b^2 \cdot 2^{- 2} a^{- 3} \equiv - 1 \!\! \pmod{p}</math>. Mamy
+
::::<math>= f_m + (- f_{m + 1} + f_{m + 1}) + (- f_{m + 2} + f_{m + 2}) + (- f_{m + 3} + \ldots + f_{n - 1}) + (- f_n + f_n) - f_{n + 1} =</math>
  
::<math>S(a, b) = \sum_{x = 0}^{p - 1} \left( {\small\frac{x - x_2}{p}} \right)_{\small{\!\! L}}^{\! 2} \left( {\small\frac{x - x_1}{p}} \right)_{\small{\!\! L}}</math>
+
::::<math>= f_m - f_{n + 1}</math>
  
Niech <math>t = x - x_2</math>. Jeżeli <math>x</math> przebiega zbiór <math>\{ 0, 1, \ldots, p - 1 \}</math>, to (modulo <math>p</math>) <math>t</math> przebiega taki sam zbiór (zobacz C57). Zatem
 
  
::<math>S(a, b) = \sum_{t = 0}^{p - 1} \left( {\small\frac{t}{p}} \right)_{\small{\!\! L}}^{\! 2} \left( {\small\frac{t + x_2 - x_1}{p}} \right)_{\small{\!\! L}}
 
= \sum_{t = 1}^{p - 1} \left( {\small\frac{t + x_2 - x_1}{p}} \right)_{\small{\!\! L}}
 
= - \left( {\small\frac{x_2 - x_1}{p}} \right)_{\small{\!\! L}} + \sum_{t = 0}^{p - 1} \left( {\small\frac{t + x_2 - x_1}{p}} \right)_{\small{\!\! L}}
 
= - \left( {\small\frac{x_2 - x_1}{p}} \right)_{\small{\!\! L}}</math>
 
  
Uwzględniając, że
+
::<math>\sum_{k = m}^{n} a_k = \sum_{k = m}^{n} (f_{k - 1} - f_k) =</math>
  
::<math>x_2 - x_1 \equiv - 3 b 2^{- 1} a^{- 1} - 3 b a^{- 1} \equiv - 3 b 2^{- 1} a^{- 1} - 6 b 2^{- 1} a^{- 1} \equiv - 9 b 2^{- 1} a^{- 1} \!\! \pmod{p}</math>
+
::::<math>= (f_{m - 1} - f_m) + (f_m - f_{m + 1}) + (f_{m + 1} - f_{m + 2}) + \ldots + (f_{n - 2} - f_{n - 1}) + (f_{n - 1} - f_n) =</math>
  
otrzymujemy
+
::::<math>= f_{m - 1} - f_m + f_m - f_{m + 1} + f_{m + 1} - f_{m + 2} + \ldots + f_{n - 2} - f_{n - 1} + f_{n - 1} - f_n =</math>
  
::<math>S(a, b) = - \left( {\small\frac{x_2 - x_1}{p}} \right)_{\small{\!\! L}}
+
::::<math>= f_{m - 1} + (- f_m + f_m) + (- f_{m + 1} + f_{m + 1}) + (- f_{m + 2} + \ldots + f_{n - 2}) + (- f_{n - 1} + f_{n - 1}) - f_n =</math>
= - \left( {\small\frac{- 9 b 2^{- 1} a^{- 1}}{p}} \right)_{\small{\!\! L}}
 
= - \left( {\small\frac{- 2 a b}{p}} \right)_{\small{\!\! L}}
 
= - \left( {\small\frac{- 8 a^3 b}{p}} \right)_{\small{\!\! L}}
 
= - \left( {\small\frac{- 2 b \cdot (- 27 b^2)}{p}} \right)_{\small{\!\! L}}
 
= - \left( {\small\frac{6 b}{p}} \right)_{\small{\!\! L}}</math>
 
  
 
+
::::<math>= f_{m - 1} - f_n</math><br/>
W przypadku, gdy <math>4 a^3 + 27 b^2 \not\equiv 0 \!\! \pmod{p}</math>, pokażemy, że wartość sumy
 
 
 
::<math>S(a, b) = \sum_{x = 0}^{p - 1} \left( {\small\frac{x^3 + a x + b}{p}} \right)_{\small{\!\! L}}</math>
 
 
 
jest ściśle związana z&nbsp;ilością rozwiązań kongruencji
 
 
 
::<math>y^2 \equiv x^3 + a x + b \!\! \pmod{p}</math>
 
 
 
 
 
Niech <math>N_p</math> oznacza ilość rozwiązań powyższej kongruencji i&nbsp;niech <math>N_+, N_0, N_-</math> oznaczają ilości liczb <math>k \in \{ 0, 1, \ldots, p - 1 \}</math>, dla których symbol Legendre'a <math>\left( {\small\frac{x^3 + a x + b}{p}} \right)_{\small{\!\! L}}</math> jest równy odpowiednio <math>+ 1, 0, - 1</math>. Oczywiście
 
 
 
::<math>N_+ + N_0 + N_- = p</math>
 
 
 
::<math>S(a, b) = N_+ - N_-</math>
 
 
 
Zauważmy, że jeżeli dla pewnego <math>x</math> jest <math>p \mid (x^3 + a x + b)</math>, to <math>\left( {\small\frac{x^3 + a x + b}{p}} \right)_{\small{\!\! L}} = 0</math> i&nbsp;mamy dokładnie jedno rozwiązanie rozważanej kongruencji
 
 
 
::<math>0^2 \equiv x^3 + a x + b \!\! \pmod{p}</math>
 
 
 
Jeżeli dla pewnego <math>x</math> jest <math>\left( {\small\frac{x^3 + a x + b}{p}} \right)_{\small{\!\! L}} = + 1</math>, to <math>p \nmid (x^3 + a x + b)</math>, a&nbsp;liczba <math>x^3 + a x + b</math> jest liczbą kwadratową modulo <math>p</math>, czyli istnieje taka liczba <math>y \in \mathbb{Z}</math>, że
 
 
 
::<math>y^2 \equiv x^3 + a x + b \!\! \pmod{p}</math>
 
 
 
i mamy dwa rozwiązania rozpatrywanej kongruencji: jedno stanowi para <math>(x, y)</math>, a&nbsp;drugie para <math>(x, - y)</math>. Zatem
 
 
 
::<math>N_p = 2 N_+ + N_0</math>
 
 
 
Łatwo zauważamy, że
 
 
 
::<math>N_p - p = (2 N_+ + N_0) - (N_+ + N_0 + N_-) = N_+ - N_- = S (a, b)</math>
 
 
 
 
 
W 1936 roku Helmut Hasse<ref name="Hasse1"/><ref name="Hasse2"/> udowodnił, że
 
 
 
::<math>| N_p - p | < 2 \sqrt{p}</math>
 
 
 
Elementarny dowód tego twierdzenia podał Jurij Manin<ref name="Manin1"/>.
 
 
 
 
 
Wynika stąd, że w&nbsp;przypadku, gdy <math>4 a^3 + 27 b^2 \not\equiv 0 \!\! \pmod{p}</math> prawdziwe jest oszacowanie
 
 
 
::<math>| S (a, b) | = \left| \sum_{x = 0}^{p - 1} \left( {\small\frac{x^3 + a x + b}{p}} \right)_{\small{\!\! L}} \right| < 2 \sqrt{p}</math>
 
 
 
Co należało pokazać.<br/>
 
 
&#9633;
 
&#9633;
 
{{\Spoiler}}
 
{{\Spoiler}}
Linia 535: Linia 239:
  
  
<span style="font-size: 110%; font-weight: bold;">Zadanie K8</span><br/>
+
<span style="font-size: 110%; font-weight: bold;">Twierdzenie D12</span><br/>
Pokazać, że jeżeli <math>p \geqslant 7</math> jest liczbą pierwszą, to wśród liczb <math>1, 2, \ldots, p - 1</math> istnieją:
+
Następujące szeregi są zbieżne
 
 
:* dwie kolejne liczby będące liczbami kwadratowymi modulo <math>p</math>
 
:* dwie kolejne liczby będące liczbami niekwadratowymi modulo <math>p</math>
 
 
 
{{Spoiler|Style = font-style: italic; font-weight: bold; color: olive; text-decoration: underline;|Show=Rozwiązanie|Hide=Ukryj rozwiązanie}}
 
Dla <math>p = 7</math> łatwo sprawdzamy, że twierdzenie jest prawdziwe.
 
 
 
'''Punkt 1.'''
 
 
 
Zauważmy, że przynajmniej jedna z&nbsp;liczb <math>2, 5, 10</math> jest liczbą kwadratową. Zakładając, że tak nie jest, otrzymujemy natychmiast sprzeczność
 
 
 
::<math> -1 = \left( {\small\frac{10}{p}} \right)_{\small{\!\! L}} = \left( {\small\frac{2}{p}} \right)_{\small{\!\! L}} \cdot \left( {\small\frac{5}{p}} \right)_{\small{\!\! L}} = (- 1) \cdot (- 1) = 1</math>
 
 
 
W zależności od tego, która z&nbsp;liczb <math>2, 5, 10</math> jest liczbą kwadratową, mamy następujące pary kolejnych liczb kwadratowych
 
 
 
::{| class="wikitable plainlinks"  style="font-size: 90%; text-align: center; margin-right: auto;"
 
|-
 
| <math>2</math> || <math>1, 2 \; \text{ oraz } \; 8, 9</math>
 
|-
 
| <math>5</math> || <math>4, 5</math>
 
|-
 
| <math>10</math> || <math>9, 10</math>
 
|}
 
 
 
'''Punkt 2.'''
 
 
 
Rozważmy wszystkie możliwe wartości <math>\left( {\small\frac{k}{p}} \right)_{\small{\!\! L}}</math> dla <math>k = 1, 2, 3, 4</math> i <math>p \geqslant 11</math>. Zauważmy, że <math>\left( {\small\frac{6}{p}} \right)_{\small{\!\! L}} = \left( {\small\frac{2}{p}} \right)_{\small{\!\! L}} \cdot \left( {\small\frac{3}{p}} \right)_{\small{\!\! L}}</math>.
 
  
::{| class="wikitable plainlinks"  style="font-size: 90%; text-align: center; margin-right: auto;"
+
::{| class="wikitable plainlinks"  style="font-size: 100%; text-align: left; margin-right: auto;"
 
|-
 
|-
! <math>\boldsymbol{k}</math> || <math>\,\, \boldsymbol{1} \,\,</math> || <math>\boldsymbol{2}</math> || <math>\boldsymbol{3}</math> || <math>\,\, \boldsymbol{4} \,\,</math> || <math>\,\, \boldsymbol{5} \,\,</math> || <math>\boldsymbol{6}</math> || <math>\boldsymbol{()}</math> || <math>\boldsymbol{p-1}</math>
+
| 1. <math>\quad \sum^{\infty}_{k = 1} \frac{1}{k (k + 1)} = 1</math>
 +
|  
 
|-
 
|-
! <math>\boldsymbol{A.}</math>
+
| 2. <math>\quad \sum^{\infty}_{k = 2} \frac{1}{k (k - 1)} = 1</math>
| <math>1</math> || <math>1</math> || <math>1</math> || <math>1</math> || <math></math> || <math>1</math> || <math></math> || <math></math>
+
|  
 
|-
 
|-
! <math>\boldsymbol{B.}</math>
+
| 3. <math>\quad \sum^{\infty}_{k = 2} \frac{1}{k^2 - 1} = \frac{3}{4}</math>
| <math>1</math> || <math>1</math> || <math>-1</math> || <math>1</math> || <math></math> || <math>-1</math> || <math></math> || <math></math>
+
|  
 
|-
 
|-
! <math>\boldsymbol{C.}</math>
+
| 4. <math>\quad \sum^{\infty}_{k = 1} \frac{1}{k^2} = \frac{\pi^2}{6} = 1.644934066848 \ldots</math>
| <math>1</math> || <math>-1</math> || <math>1</math> || <math>1</math> || <math></math> || <math>-1</math> || <math></math> || <math></math>
+
| [https://oeis.org/A013661 A013661], [https://www.wolframalpha.com/input/?i=Zeta%282%29 WolframAlpha]
|-
 
! <math>\boldsymbol{D.}</math>  
 
| <math>1</math> || <math>-1</math> || <math>-1</math> || <math>1</math> || <math></math> || <math>1</math> || <math></math> || <math></math>
 
 
|}
 
|}
 
'''A.''' W&nbsp;tym przypadku liczby <math>2, 3</math> są liczbami kwadratowymi modulo <math>p</math>. Gdyby w&nbsp;pozostałych komórkach miało nie być ani jednej pary kolejnych liczb niekwadratowych modulo <math>p</math>, to musielibyśmy <math>{\small\frac{p - 1}{2}}</math> liczb niekwadratowych umieścić wśród pozostałych <math>p - 5</math> komórek tak, aby między nimi zawsze była liczba kwadratowa modulo <math>p</math>. Wartość <math>\left( {\small\frac{6}{p}} \right)_{\small{\!\! L}}</math> wymusza, aby liczby niekwadratowe modulo <math>p</math> umieszczać w&nbsp;komórkach „nieparzystych”. Po wypełnieniu tych komórek pozostaną nam dwie liczby, które będziemy zmuszeni umieścić w&nbsp;komórkach „parzystych”. Co oznacza, że muszą pojawić się dwie pary kolejnych liczb niekwadratowych modulo <math>p .</math>
 
 
'''B. i&nbsp;C.''' W&nbsp;tym przypadku dokładnie jedna z&nbsp;liczb <math>2, 3</math> jest liczbą kwadratową modulo <math>p</math>. Gdyby w&nbsp;pozostałych komórkach miało nie być ani jednej pary kolejnych liczb niekwadratowych modulo <math>p</math>, to musielibyśmy <math>{\small\frac{p - 3}{2}}</math> liczb niekwadratowych umieścić wśród pozostałych <math>p - 5</math> komórek tak, aby między nimi zawsze była liczba kwadratowa modulo <math>p</math>. Wartość <math>\left( {\small\frac{6}{p}} \right)_{\small{\!\! L}}</math> wymusza, aby liczby niekwadratowe modulo <math>p</math> umieszczać w&nbsp;komórkach „parzystych”. Po wypełnieniu tych komórek pozostanie nam jedna liczba, którą będziemy zmuszeni umieścić w&nbsp;komórce „nieparzystej”. Co oznacza, że musi pojawić się jedna para kolejnych liczb niekwadratowych modulo <math>p .</math>
 
 
'''D.''' W&nbsp;tym przypadku nie musimy niczego dowodzić, bo liczby <math>2, 3</math> są kolejnymi liczbami niekwadratowymi modulo <math>p .</math><br/>
 
&#9633;
 
{{\Spoiler}}
 
 
 
 
<span style="font-size: 110%; font-weight: bold;">Uwaga K9</span><br/>
 
Wzmocnimy wynik uzyskany w&nbsp;poprzednim zadaniu. Zauważmy, jak użycie symbolu Legendre'a pozwala sformalizować problem.
 
 
 
 
<span style="font-size: 110%; font-weight: bold;">Twierdzenie K10</span><br/>
 
Jeżeli <math>p</math> jest liczbą pierwszą nieparzystą, to
 
 
:* istnieje <math>\left\lfloor {\small\frac{p - 3}{4}} \right\rfloor</math> różnych par kolejnych liczb kwadratowych modulo <math>p</math>
 
:* istnieje <math>\left\lfloor {\small\frac{p - 1}{4}} \right\rfloor</math> różnych par kolejnych liczb niekwadratowych modulo <math>p</math>
 
  
 
{{Spoiler|Style = font-style: italic; font-weight: bold; color: olive; text-decoration: underline;|Show=Dowód|Hide=Ukryj dowód}}
 
{{Spoiler|Style = font-style: italic; font-weight: bold; color: olive; text-decoration: underline;|Show=Dowód|Hide=Ukryj dowód}}
 +
'''Punkt 1.'''<br/>
 +
Dla dowodu wykorzystamy fakt, że rozpatrywany szereg jest szeregiem teleskopowym
  
'''Punkt 1.'''
+
::<math>\frac{1}{k (k + 1)} = \frac{1}{k} - \frac{1}{k + 1}</math>
  
Chcemy znaleźć ilość takich liczb <math>k \in \{ 1, 2, \ldots, p - 2 \}</math>, dla których
+
Zatem
 
 
::<math>\left( {\small\frac{k}{p}} \right)_{\small{\!\! L}} = \left( {\small\frac{k + 1}{p}} \right)_{\small{\!\! L}} = 1</math>
 
 
 
Ilość liczb <math>k</math> spełniających powyższy warunek łatwo zapisać korzystając z&nbsp;symbolu Legendre'a
 
 
 
::<math>N = {\small\frac{1}{4}} \sum_{k = 1}^{p - 2} \left[ 1 + \left( {\small\frac{k}{p}} \right)_{\small{\!\! L}} \right] \left[ 1 + \left( {\small\frac{k + 1}{p}} \right)_{\small{\!\! L}} \right]</math>
 
 
 
Tylko w&nbsp;przypadku, gdy obie liczby <math>k</math> i <math>k + 1</math> są liczbami kwadratowymi modulo <math>p</math>, iloczyn wyrażeń w&nbsp;nawiasach kwadratowych jest różny od zera i&nbsp;równy <math>4</math> (stąd czynnik <math>{\small\frac{1}{4}}</math> przed sumą).
 
 
 
::<math>4 N = \sum_{k = 1}^{p - 2} \left[ 1 + \left( {\small\frac{k}{p}} \right)_{\small{\!\! L}} + \left( {\small\frac{k + 1}{p}} \right)_{\small{\!\! L}} + \left( {\small\frac{k}{p}} \right)_{\small{\!\! L}} \left( {\small\frac{k + 1}{p}} \right)_{\small{\!\! L}} \right]</math>
 
 
 
<div style="margin-top: 1em; margin-bottom: 1em;">
 
:::<math>\: = p - 2 + \sum_{k = 1}^{p - 2} \left( {\small\frac{k}{p}} \right)_{\small{\!\! L}} + \sum_{k = 1}^{p - 2} \left( {\small\frac{k + 1}{p}} \right)_{\small{\!\! L}} + \sum_{k = 1}^{p - 2} \left( {\small\frac{k}{p}} \right)_{\small{\!\! L}} \left( {\small\frac{k + 1}{p}} \right)_{\small{\!\! L}}</math>
 
</div>
 
 
 
Po kolei wyliczamy sumy po prawej stronie
 
 
 
<div style="margin-top: 0em; margin-bottom: 1em;">
 
::<math>\sum_{k = 1}^{p - 2} \left( {\small\frac{k}{p}} \right)_{\small{\!\! L}}
 
= - \left( {\small\frac{p - 1}{p}} \right)_{\small{\!\! L}} + \sum_{k = 1}^{p - 1} \left( {\small\frac{k}{p}} \right)_{\small{\!\! L}}
 
= - \left( {\small\frac{- 1}{p}} \right)_{\small{\!\! L}}</math>
 
</div>
 
 
 
<div style="margin-top: 1em; margin-bottom: 1em;">
 
::<math>\sum_{k = 1}^{p - 2} \left( {\small\frac{k + 1}{p}} \right)_{\small{\!\! L}}
 
= - \left( {\small\frac{1}{p}} \right)_{\small{\!\! L}} + \sum^{p - 1}_{k = 0} \left( {\small\frac{k + 1}{p}} \right)_{\small{\!\! L}}
 
= - 1</math>
 
</div>
 
 
 
<div style="margin-top: 1em; margin-bottom: 1em;">
 
::<math>\sum_{k = 1}^{p - 2} \left( {\small\frac{k}{p}} \right)_{\small{\!\! L}} \left( {\small\frac{k + 1}{p}} \right)_{\small{\!\! L}}
 
= \sum_{k = 0}^{p - 1} \left( {\small\frac{k}{p}} \right)_{\small{\!\! L}} \left( {\small\frac{k + 1}{p}} \right)_{\small{\!\! L}}
 
= - 1</math>
 
</div>
 
 
 
(zobacz K1 i&nbsp;K3). Zatem
 
 
 
::<math>N = {\small\frac{1}{4}} \left[ p - 4 - \left( {\small\frac{- 1}{p}} \right)_{\small{\!\! L}} \right]</math>
 
 
 
Czyli
 
 
 
::<math>N =
 
\begin{cases}
 
  {\large\frac{p - 5}{4}} & \text{ gdy } \; p = 4 k + 1 \\
 
  {\large\frac{p - 3}{4}} & \text{ gdy } \; p = 4 k + 3 \\
 
\end{cases}</math>
 
 
 
Powyższy wynik można zapisać w&nbsp;postaci
 
 
 
::<math>N = \left\lfloor {\small\frac{p - 3}{4}} \right\rfloor</math>
 
 
 
'''Punkt 2.'''
 
 
 
Chcemy znaleźć ilość takich liczb <math>k \in \{ 1, 2, \ldots, p - 2 \}</math>, dla których
 
 
 
::<math>\left( {\small\frac{k}{p}} \right)_{\small{\!\! L}} = \left( {\small\frac{k + 1}{p}} \right)_{\small{\!\! L}} = - 1</math>
 
 
 
Ilość liczb <math>k</math> spełniających powyższy warunek łatwo zapisać korzystając z&nbsp;symbolu Legendre'a
 
 
 
::<math>N = {\small\frac{1}{4}} \sum_{k = 1}^{p - 2} \left[ - 1 + \left( {\small\frac{k}{p}} \right)_{\small{\!\! L}} \right] \left[ - 1 + \left( {\small\frac{k + 1}{p}} \right)_{\small{\!\! L}} \right]</math>
 
 
 
Tylko w&nbsp;przypadku, gdy obie liczby <math>k</math> i <math>k + 1</math> są liczbami niekwadratowymi modulo <math>p</math>, iloczyn wyrażeń w&nbsp;nawiasach kwadratowych jest różny od zera i&nbsp;równy <math>4</math> (stąd czynnik <math>{\small\frac{1}{4}}</math> przed sumą).
 
  
::<math>4 N = \sum_{k = 1}^{p - 2} \left[ 1 - \left( {\small\frac{k}{p}} \right)_{\small{\!\! L}} - \left( {\small\frac{k + 1}{p}} \right)_{\small{\!\! L}} + \left( {\small\frac{k}{p}} \right)_{\small{\!\! L}} \left( {\small\frac{k + 1}{p}} \right)_{\small{\!\! L}} \right]</math>
+
::<math>\sum^n_{k = 1} \frac{1}{k (k + 1)} = \sum^n_{k = 1} \left( \frac{1}{k} - \frac{1}{k + 1} \right) = 1 - \frac{1}{n + 1}</math>
  
<div style="margin-top: 1em; margin-bottom: 1em;">
+
Przechodząc z <math>n</math> do nieskończoności, dostajemy
:::<math>\: = p - 2 - \sum_{k = 1}^{p - 2} \left( {\small\frac{k}{p}} \right)_{\small{\!\! L}} - \sum_{k = 1}^{p - 2} \left( {\small\frac{k + 1}{p}} \right)_{\small{\!\! L}} + \sum_{k = 1}^{p - 2} \left( {\small\frac{k}{p}} \right)_{\small{\!\! L}} \left( {\small\frac{k + 1}{p}} \right)_{\small{\!\! L}}</math>
 
</div>
 
  
Wartości sum wyliczyliśmy już w&nbsp;punkcie 1. Zatem
+
::<math>\sum^{\infty}_{k = 1} \frac{1}{k (k + 1)} = 1</math>
  
::<math>N = {\small\frac{1}{4}} \left[ p - 2 + \left( {\small\frac{- 1}{p}} \right)_{\small{\!\! L}} \right]</math>
+
'''Punkt 2.'''<br/>
 +
Szereg jest identyczny z&nbsp;szeregiem z&nbsp;punktu 1., co łatwo zauważyć zmieniając zmienną sumowania <math>k = s + 1</math> i&nbsp;odpowiednio granice sumowania.
  
Czyli
+
'''Punkt 3.'''<br/>
 +
Należy skorzystać z&nbsp;tożsamości
  
::<math>N =  
+
::<math>\frac{1}{k^2 - 1} = \frac{1}{2} \left[ \left( \frac{1}{k} - \frac{1}{k + 1} \right) + \left( \frac{1}{k - 1} - \frac{1}{k} \right) \right]</math>
\begin{cases}
 
  {\large\frac{p - 1}{4}} & \text{ gdy } \; p = 4 k + 1 \\
 
  {\large\frac{p - 3}{4}} & \text{ gdy } \; p = 4 k + 3 \\
 
\end{cases}</math>
 
  
Powyższy wynik można zapisać w&nbsp;postaci
+
'''Punkt 4.'''<br/>
 +
Ponieważ dla <math>k \geqslant 2</math> prawdziwa jest nierówność
  
::<math>N = \left\lfloor {\small\frac{p - 1}{4}} \right\rfloor</math>
+
::<math>0 < \frac{1}{k^2} < \frac{1}{k^2 - 1}</math>
  
Co należało pokazać.<br/>
+
to na mocy kryterium porównawczego (twierdzenie D9) ze zbieżności szeregu <math>\sum^{\infty}_{k = 2} \frac{1}{k^2 - 1}</math> wynika zbieżność szeregu <math>\sum_{k = 1}^{\infty} \frac{1}{k^2}</math><br/>
 
&#9633;
 
&#9633;
 
{{\Spoiler}}
 
{{\Spoiler}}
Linia 699: Linia 290:
  
  
<span style="font-size: 110%; font-weight: bold;">Twierdzenie K11</span><br/>
+
<span style="font-size: 110%; font-weight: bold;">Twierdzenie D13</span><br/>
Niech <math>p</math> będzie liczbą pierwszą nieparzystą. Słowo „trójka” oznacza tutaj trzy kolejne liczby kwadratowe (niekwadratowe) modulo <math>p</math>.
+
Następujące szeregi są zbieżne
  
Jeżeli <math>p = 4 k + 3</math>, to liczba różnych trójek liczb kwadratowych (niekwadratowych) jest równa
+
::{| class="wikitable plainlinks"  style="font-size: 100%; text-align: left; margin-right: auto;"
 +
|-
 +
| 1. <math>\quad \sum_{k = 1}^{\infty} \frac{1}{(k + 1) \sqrt{k}} = 1.860025079221 \ldots</math>
 +
|
 +
|-
 +
| 2. <math>\quad \sum^{\infty}_{k = 2} \frac{\log k}{k (k + 1)} = 0.788530565911 \ldots</math>
 +
| [https://oeis.org/A085361 A085361]
 +
|-
 +
| 3. <math>\quad \sum^{\infty}_{k = 2} \frac{\log k}{k (k - 1)} = 1.257746886944 \ldots</math>
 +
| [https://oeis.org/A131688 A131688]
 +
|-
 +
| 4. <math>\quad \sum^{\infty}_{k = 3} \frac{1}{k \cdot \log^2 \! k} = 1.069058310734 \ldots</math>
 +
| [https://oeis.org/A115563 A115563]
 +
|}
  
::<math>N = \left\lfloor {\small\frac{p - 3}{8}} \right\rfloor</math>
+
{{Spoiler|Style = font-style: italic; font-weight: bold; color: olive; text-decoration: underline;|Show=Dowód|Hide=Ukryj dowód}}
 +
'''Punkt 1.'''<br/>
  
Jeżeli <math>p = 4 k + 1</math>, to liczba różnych trójek liczb niekwadratowych jest równa
+
Wystarczy zauważyć, że
  
::<math>N = {\small\frac{p - 3 - S (- 1)}{8}} > {\small\frac{p - 3 - 2 \sqrt{p}}{8}}</math>
+
::<math>\frac{1}{\sqrt{k}} - \frac{1}{\sqrt{k + 1}} = \frac{\sqrt{k + 1} - \sqrt{k}}{\sqrt{k} \cdot \sqrt{k + 1}} =</math>
  
Jeżeli <math>p = 4 k + 1</math>, to liczba różnych trójek liczb kwadratowych jest równa
+
::::::<math>\quad\: = \frac{1}{\sqrt{k} \cdot \sqrt{k + 1} \cdot \left( \sqrt{k + 1} + \sqrt{k} \right)} ></math>
  
::<math>N = {\small\frac{p - 15 + S (- 1)}{8}} > {\small\frac{p - 15 - 2 \sqrt{p}}{8}} \qquad \quad \text{ gdy } \; p = 8 k + 1</math>
+
::::::<math>\quad\: > \frac{1}{\sqrt{k} \cdot \sqrt{k + 1} \cdot 2 \sqrt{k + 1}}</math>
  
::<math>N = {\small\frac{p - 7 + S (- 1)}{8}} > {\small\frac{p - 7 - 2 \sqrt{p}}{8}} \qquad \quad \;\;\; \text{ gdy } \; p = 8 k + 5</math>
+
::::::<math>\quad\: = \frac{1}{2 (k + 1) \sqrt{k}}</math>
  
Gdzie przez <math>S(- 1)</math> oznaczyliśmy sumę
+
Zatem
  
::<math>S(- 1) = \sum_{k = 0}^{p - 1} \left( {\small\frac{k (k^2 - 1)}{p}} \right)_{\small{\!\! L}}</math>
+
::<math>\sum_{k = 1}^n \frac{1}{(k + 1) \sqrt{k}} = 2 \sum_{k = 1}^n \frac{1}{2 (k + 1) \sqrt{k}} <</math>
  
{{Spoiler|Style = font-style: italic; font-weight: bold; color: olive; text-decoration: underline;|Show=Dowód|Hide=Ukryj dowód}}
+
::::::<math>\;\;\;\: < 2 \sum_{k = 1}^n \left( \frac{1}{\sqrt{k}} - \frac{1}{\sqrt{k + 1}} \right) =</math>
  
'''Przypadek pierwszy: trójki liczb kwadratowych modulo''' <math>\boldsymbol{p}</math>
+
::::::<math>\;\;\;\: = 2 \left( 1 - \frac{1}{\sqrt{n + 1}} \right) <</math>
  
Chcemy znaleźć ilość takich liczb <math>k \in \{ 2, 3, \ldots, p - 2 \}</math>, dla których
+
::::::<math>\;\;\;\: < 2</math>
  
::<math>\left( {\small\frac{k - 1}{p}} \right)_{\small{\!\! L}} = \left( {\small\frac{k}{p}} \right)_{\small{\!\! L}} = \left( {\small\frac{k + 1}{p}} \right)_{\small{\!\! L}} = + 1</math>
+
Ponieważ ciąg sum częściowych szeregu jest rosnący i&nbsp;ograniczony, to szereg jest zbieżny.
  
Ilość liczb <math>k</math> spełniających powyższy warunek łatwo zapisać korzystając z&nbsp;symbolu Legendre'a
+
'''Punkt 2.'''<br/>
 +
Korzystając z&nbsp;twierdzenia A37, możemy napisać oszacowanie
  
::<math>N = {\small\frac{1}{8}} \sum_{k = 2}^{p - 2} \left[ 1 + \left( {\small\frac{k - 1}{p}} \right)_{\small{\!\! L}} \right] \left[ 1 + \left( {\small\frac{k}{p}} \right)_{\small{\!\! L}} \right] \left[ 1 + \left( {\small\frac{k + 1}{p}} \right)_{\small{\!\! L}} \right]</math>
+
::<math>0 < \frac{\log k}{k (k + 1)} < \frac{2 \sqrt{k}}{k (k + 1)} < \frac{2}{(k + 1) \sqrt{k}}</math>
  
Tylko w&nbsp;przypadku, gdy wszystkie trzy liczby <math>k - 1, k, k + 1</math> są liczbami kwadratowymi modulo <math>p</math>, iloczyn wyrażeń w&nbsp;nawiasach kwadratowych jest różny od zera i&nbsp;równy <math>8</math> (stąd czynnik <math>{\small\frac{1}{8}}</math> przed sumą).
+
Zatem na mocy kryterium porównawczego ze zbieżności szeregu <math>\sum_{k = 1}^{\infty} \frac{1}{(k + 1) \sqrt{k}}</math> wynika zbieżność szeregu <math>\sum^{\infty}_{k = 2} \frac{\log k}{k (k + 1)}</math>
  
::<math>8 N = \sum_{k = 2}^{p - 2} \left[ 1
+
'''Punkt 3.'''<br/>
+ \left( {\small\frac{k - 1}{p}} \right)_{\small{\!\! L}}
+
Zauważmy, że
+ \left( {\small\frac{k}{p}} \right)_{\small{\!\! L}}
 
+ \left( {\small\frac{k + 1}{p}} \right)_{\small{\!\! L}}
 
+ \left( {\small\frac{k - 1}{p}} \right)_{\small{\!\! L}} \left( {\small\frac{k}{p}} \right)_{\small{\!\! L}}
 
+ \left( {\small\frac{k - 1}{p}} \right)_{\small{\!\! L}} \left( {\small\frac{k + 1}{p}} \right)_{\small{\!\! L}}
 
+ \left( {\small\frac{k}{p}} \right)_{\small{\!\! L}} \left( {\small\frac{k + 1}{p}} \right)_{\small{\!\! L}}
 
+ \left( {\small\frac{k - 1}{p}} \right)_{\small{\!\! L}} \left( {\small\frac{k}{p}} \right)_{\small{\!\! L}} \left( {\small\frac{k + 1}{p}} \right)_{\small{\!\! L}}
 
\right]</math>
 
  
:::<math>\: = p - 3 + \sum_{k = 2}^{p - 2} \left( {\small\frac{k - 1}{p}} \right)_{\small{\!\! L}}  
+
::<math>\frac{\log (k - 1)}{k - 1} - \frac{\log (k)}{k} = \frac{k \log (k - 1) - (k - 1) \log (k)}{k (k - 1)} =</math>
+ \sum_{k = 2}^{p - 2} \left( {\small\frac{k}{p}} \right)_{\small{\!\! L}}  
 
+ \sum_{k = 2}^{p - 2} \left( {\small\frac{k + 1}{p}} \right)_{\small{\!\! L}}
 
+ \sum_{k = 2}^{p - 2} \left( {\small\frac{k - 1}{p}} \right)_{\small{\!\! L}} \left( {\small\frac{k}{p}} \right)_{\small{\!\! L}}
 
+ \sum_{k = 2}^{p - 2} \left( {\small\frac{k - 1}{p}} \right)_{\small{\!\! L}} \left( {\small\frac{k + 1}{p}} \right)_{\small{\!\! L}}
 
+ \sum_{k = 2}^{p - 2} \left( {\small\frac{k}{p}} \right)_{\small{\!\! L}} \left( {\small\frac{k + 1}{p}} \right)_{\small{\!\! L}}  
 
+ \sum_{k = 2}^{p - 2} \left( {\small\frac{k (k^2 - 1)}{p}} \right)_{\small{\!\! L}}</math>
 
  
 +
::::::::<math>\;\;\, = \frac{k \log \left( k \left( 1 - \frac{1}{k} \right) \right) - (k - 1) \log (k)}{k (k - 1)} =</math>
  
Po kolei wyliczamy sumy po prawej stronie
+
::::::::<math>\;\;\, = \frac{k \log (k) + k \log \left( 1 - \frac{1}{k} \right) - k \log (k) + \log (k)}{k (k - 1)} ></math>
  
::<math>\sum_{k = 2}^{p - 2} \left( {\small\frac{k - 1}{p}} \right)_{\small{\!\! L}} = - \left( {\small\frac{- 1}{p}} \right)_{\small{\!\! L}} - \left( {\small\frac{- 2}{p}} \right)_{\small{\!\! L}}</math>
+
::::::::<math>\;\;\, > \frac{\log (k) - k \cdot \frac{1}{k - 1}}{k (k - 1)} =</math>
  
::<math>\sum_{k = 2}^{p - 2} \left( {\small\frac{k}{p}} \right)_{\small{\!\! L}} = - 1 - \left( {\small\frac{- 1}{p}} \right)_{\small{\!\! L}}</math>
+
::::::::<math>\;\;\, = \frac{\log (k)}{k (k - 1)} - \frac{1}{(k - 1)^2}</math>
  
::<math>\sum_{k = 2}^{p - 2} \left( {\small\frac{k + 1}{p}} \right)_{\small{\!\! L}} = - 1 - \left( {\small\frac{2}{p}} \right)_{\small{\!\! L}}</math>
+
Czyli prawdziwe jest oszacowanie
  
 +
::<math>\frac{\log (k)}{k (k - 1)} < \left[ \frac{\log (k - 1)}{k - 1} - \frac{\log (k)}{k} \right] + \frac{1}{(k - 1)^2}</math>
  
::<math>\sum_{k = 2}^{p - 2} \left( {\small\frac{k - 1}{p}} \right)_{\small{\!\! L}} \left( {\small\frac{k}{p}} \right)_{\small{\!\! L}} = - 1 - \left( {\small\frac{2}{p}} \right)_{\small{\!\! L}}</math>
+
Zatem możemy napisać
  
::<math>\sum_{k = 2}^{p - 2} \left( {\small\frac{k - 1}{p}} \right)_{\small{\!\! L}} \left( {\small\frac{k + 1}{p}} \right)_{\small{\!\! L}} = - 1 - \left( {\small\frac{- 1}{p}} \right)_{\small{\!\! L}}</math>
+
::<math>\sum_{k = 2}^{n} \frac{\log (k)}{k (k - 1)} < \sum_{k = 2}^{n} \left[ \frac{\log (k - 1)}{k - 1} - \frac{\log (k)}{k} \right] + \sum_{k = 2}^{n} \frac{1}{(k - 1)^2}</math>
  
::<math>\sum_{k = 2}^{p - 2} \left( {\small\frac{k}{p}} \right)_{\small{\!\! L}} \left( {\small\frac{k + 1}{p}} \right)_{\small{\!\! L}} = - 1 - \left( {\small\frac{2}{p}} \right)_{\small{\!\! L}}</math>
+
::::::<math>\: < - \frac{\log (n)}{n} + \sum_{j = 1}^{n - 1} \frac{1}{j^2}</math>
  
 +
::::::<math>\: < \sum_{j = 1}^{\infty} \frac{1}{j^2} =</math>
  
::<math>\sum_{k = 2}^{p - 2} \left( {\small\frac{k (k^2 - 1)}{p}} \right)_{\small{\!\! L}} = \sum^{p - 1}_{k = 0} \left( {\small\frac{k (k^2 - 1)}{p}} \right)_{\small{\!\! L}} = S (- 1)</math>
+
::::::<math>\: = \frac{\pi^2}{6}</math>
  
 +
Ponieważ ciąg sum częściowych szeregu jest rosnący i&nbsp;ograniczony, to szereg jest zbieżny.
  
(zobacz K1, K3 i K6). Oznaczenie <math>S(- 1)</math> nawiązuje do oznaczenia wprowadzonego w&nbsp;twierdzeniu K6. Wykorzystamy też znalezione w&nbsp;tym twierdzeniu oszacowanie <math>| S (- 1) |</math>.
+
'''Punkt 4.'''<br/>
 +
Zauważmy, że
  
Zatem
+
::<math>\frac{1}{\log (k)} - \frac{1}{\log (k + 1)} = \frac{\log (k + 1) - \log (k)}{\log (k) \log (k + 1)} =</math>
  
::<math>8 N = p - 8 - 3 \left( {\small\frac{- 1}{p}} \right)_{\small{\!\! L}} - 3 \left( {\small\frac{2}{p}} \right)_{\small{\!\! L}} - \left( {\small\frac{- 2}{p}} \right)_{\small{\!\! L}} + S (- 1)</math>
+
::::::::<math>\;\;\, = \frac{\log \left( 1 + \frac{1}{k} \right)}{\log (k) \log (k + 1)} <</math>
  
Jeżeli <math>p = 8 k + 1</math>
+
::::::::<math>\;\;\, < \frac{1}{k \cdot \log (k) \log (k + 1)} <</math>
  
::<math>N = {\small\frac{p - 15 + S (- 1)}{8}} > {\small\frac{p - 15 - 2 \sqrt{p}}{8}}</math>
+
::::::::<math>\;\;\, < \frac{1}{k \cdot \log^2 \! k}</math>
  
Jeżeli <math>p = 8 k + 3</math>
+
Z drugiej strony mamy
  
::<math>N = {\small\frac{p - 3}{8}}</math>
+
::<math>\frac{1}{\log (k - 1)} - \frac{1}{\log (k)} = \frac{\log (k) - \log (k - 1)}{\log (k - 1) \log (k)} =</math>
  
Jeżeli <math>p = 8 k + 5</math>
+
::::::::<math>\;\;\, = \frac{\log \left( 1 + \frac{1}{k - 1} \right)}{\log (k - 1) \log (k)} ></math>
  
::<math>N = {\small\frac{p - 7 + S (- 1)}{8}} > {\small\frac{p - 7 - 2 \sqrt{p}}{8}}</math>
+
::::::::<math>\;\;\, > \frac{1}{k \cdot \log (k - 1) \log (k)} ></math>
  
Jeżeli <math>p = 8 k + 7</math>
+
::::::::<math>\;\;\, > \frac{1}{k \cdot \log^2 \! k}</math>
  
::<math>N = {\small\frac{p - 7}{8}}</math>
+
Wynika stąd następujący ciąg nierówności
  
 +
::<math>\frac{1}{\log (k)} - \frac{1}{\log (k + 1)} < \frac{1}{k \cdot \log^2 \! k} < \frac{1}{\log (k - 1)} - \frac{1}{\log (k)}</math>
  
'''Przypadek drugi: trójki liczb niekwadratowych modulo''' <math>\boldsymbol{p}</math>
 
  
Chcemy znaleźć ilość takich liczb <math>k \in \{ 2, 3, \ldots, p - 2 \}</math>, dla których
+
Rezultat ten wykorzystamy w&nbsp;pełni w&nbsp;przykładzie D14, a&nbsp;do pokazania zbieżności szeregu wystarczy nam prawa nierówność. Mamy
  
::<math>\left( {\small\frac{k - 1}{p}} \right)_{\small{\!\! L}} = \left( {\small\frac{k}{p}} \right)_{\small{\!\! L}} = \left( {\small\frac{k + 1}{p}} \right)_{\small{\!\! L}} = - 1</math>
+
::<math>\sum_{k = 3}^{n} \frac{1}{k \cdot \log^2 \! k} < \sum_{k = 3}^{n} \left[ \frac{1}{\log (k - 1)} - \frac{1}{\log (k)} \right] =</math>
  
Ilość liczb <math>k</math> spełniających powyższy warunek łatwo zapisać korzystając z&nbsp;symbolu Legendre'a
+
::::::<math>\; = \frac{1}{\log 2} - \frac{1}{\log (n)} <</math>
  
::<math>N = - {\small\frac{1}{8}} \sum_{k = 2}^{p - 2} \left[ - 1 + \left( {\small\frac{k - 1}{p}} \right)_{\small{\!\! L}} \right] \left[ - 1 + \left( {\small\frac{k}{p}} \right)_{\small{\!\! L}} \right] \left[ - 1 + \left( {\small\frac{k + 1}{p}} \right)_{\small{\!\! L}} \right]</math>
+
::::::<math>\; < \frac{1}{\log 2}</math>
  
Tylko w&nbsp;przypadku, gdy wszystkie trzy liczby <math>k - 1, k, k + 1</math> są liczbami niekwadratowymi modulo <math>p</math>, iloczyn wyrażeń w&nbsp;nawiasach kwadratowych jest różny od zera i&nbsp;równy <math>- 8</math> (stąd czynnik <math>- {\small\frac{1}{8}}</math> przed sumą).
+
Ponieważ ciąg sum częściowych szeregu jest rosnący i&nbsp;ograniczony, to szereg jest zbieżny.<br/>
 +
&#9633;
 +
{{\Spoiler}}
  
::<math>8 N = \sum_{k = 2}^{p - 2} \left[ 1
 
- \left( {\small\frac{k - 1}{p}} \right)_{\small{\!\! L}}
 
- \left( {\small\frac{k}{p}} \right)_{\small{\!\! L}}
 
- \left( {\small\frac{k + 1}{p}} \right)_{\small{\!\! L}}
 
+ \left( {\small\frac{k - 1}{p}} \right)_{\small{\!\! L}} \left( {\small\frac{k}{p}} \right)_{\small{\!\! L}}
 
+ \left( {\small\frac{k - 1}{p}} \right)_{\small{\!\! L}} \left( {\small\frac{k + 1}{p}} \right)_{\small{\!\! L}}
 
+ \left( {\small\frac{k}{p}} \right)_{\small{\!\! L}} \left( {\small\frac{k + 1}{p}} \right)_{\small{\!\! L}}
 
- \left( {\small\frac{k - 1}{p}} \right)_{\small{\!\! L}} \left( {\small\frac{k}{p}} \right)_{\small{\!\! L}} \left( {\small\frac{k + 1}{p}} \right)_{\small{\!\! L}}
 
\right]</math>
 
  
:::<math>\: = p - 3 - \sum_{k = 2}^{p - 2} \left( {\small\frac{k - 1}{p}} \right)_{\small{\!\! L}}
 
- \sum_{k = 2}^{p - 2} \left( {\small\frac{k}{p}} \right)_{\small{\!\! L}}
 
- \sum_{k = 2}^{p - 2} \left( {\small\frac{k + 1}{p}} \right)_{\small{\!\! L}}
 
+ \sum_{k = 2}^{p - 2} \left( {\small\frac{k - 1}{p}} \right)_{\small{\!\! L}} \left( {\small\frac{k}{p}} \right)_{\small{\!\! L}}
 
+ \sum_{k = 2}^{p - 2} \left( {\small\frac{k - 1}{p}} \right)_{\small{\!\! L}} \left( {\small\frac{k + 1}{p}} \right)_{\small{\!\! L}}
 
+ \sum_{k = 2}^{p - 2} \left( {\small\frac{k}{p}} \right)_{\small{\!\! L}} \left( {\small\frac{k + 1}{p}} \right)_{\small{\!\! L}}
 
- \sum_{k = 2}^{p - 2} \left( {\small\frac{k (k^2 - 1)}{p}} \right)_{\small{\!\! L}}</math>
 
  
 +
<span style="font-size: 110%; font-weight: bold;">Przykład D14</span><br/>
 +
Na przykładzie szeregu <math>\sum_{k = 3}^{\infty} \frac{1}{k \cdot \log^2 k}</math> pokażemy, jak należy obliczać przybliżoną wartość sumy szeregu.
  
Wartości sum już policzyliśmy, rozpatrując przypadek liczb kwadratowych modulo <math>p</math>. Zatem
+
Ponieważ nie jesteśmy w&nbsp;stanie zsumować nieskończenie wielu wyrazów, zatem najlepiej będzie podzielić szereg na dwie części
  
::<math>8 N = p - 4 + \left( {\small\frac{- 1}{p}} \right)_{\small{\!\! L}} - \left( {\small\frac{2}{p}} \right)_{\small{\!\! L}} + \left( {\small\frac{- 2}{p}} \right)_{\small{\!\! L}} - S (- 1)</math>
+
::<math>\sum_{k = 3}^{\infty} \frac{1}{k \cdot \log^2 k} = \sum_{k = 3}^{m} \frac{1}{k \cdot \log^2 k} + \sum_{k = m + 1}^{\infty} \frac{1}{k \cdot \log^2 k}</math>
  
  
Jeżeli <math>p = 8 k + 1</math>
+
Wartość pierwszej części możemy policzyć bezpośrednio, a&nbsp;dla drugiej części powinniśmy znaleźć jak najlepsze oszacowanie.
  
::<math>N = {\small\frac{p - 3 - S (- 1)}{8}} > {\small\frac{p - 3 - 2 \sqrt{p}}{8}}</math>
+
Dowodząc twierdzenie D13, w&nbsp;punkcie 4. pokazaliśmy, że prawdziwy jest ciąg nierówności
  
Jeżeli <math>p = 8 k + 3</math>
+
::<math>\frac{1}{\log (k)} - \frac{1}{\log (k + 1)} < \frac{1}{k \cdot \log^2 k} < \frac{1}{\log (k - 1)} - \frac{1}{\log (k)}</math>
  
::<math>N = {\small\frac{p - 3}{8}}</math>
 
  
Jeżeli <math>p = 8 k + 5</math>
+
Wykorzystamy powyższy wzór do znalezienia potrzebnego nam oszacowania. Sumując strony nierówności, dostajemy
  
::<math>N = {\small\frac{p - 3 - S (- 1)}{8}} > {\small\frac{p - 3 - 2 \sqrt{p}}{8}}</math>
+
::<math>\sum_{k = m + 1}^{n} \left( \frac{1}{\log (k)} - \frac{1}{\log (k + 1)} \right) < \sum_{k = m + 1}^{n} \frac{1}{k \cdot \log^2 k} < \sum_{k = m + 1}^{n} \left( \frac{1}{\log (k - 1)} - \frac{1}{\log (k)} \right)</math>
  
Jeżeli <math>p = 8 k + 7</math>
 
  
::<math>N = {\small\frac{p - 7}{8}}</math>
+
Ponieważ szeregi po lewej i&nbsp;po prawej stronie są szeregami teleskopowymi, to łatwo znajdujemy, że
  
Co kończy dowód.<br/>
+
::<math>\frac{1}{\log (m + 1)} - \frac{1}{\log (n + 1)} < \sum_{k = m + 1}^{n} \frac{1}{k \cdot \log^2 k} < \frac{1}{\log m} - \frac{1}{\log n}</math>
&#9633;
 
{{\Spoiler}}
 
  
  
 +
Przechodząc z <math>n</math> do nieskończoności, otrzymujemy oszacowanie
  
<span style="font-size: 110%; font-weight: bold;">Uwaga K12</span><br/>
+
::<math>\frac{1}{\log (m + 1)} < \sum_{k = m + 1}^{\infty} \frac{1}{k \cdot \log^2 k} < \frac{1}{\log m}</math>
Korzystając z&nbsp;twierdzenia K11, łatwo można pokazać, że każda liczba pierwsza <math>p \geqslant 19</math> ma co najmniej dwie różne trójki kolejnych liczb kwadratowych modulo <math>p</math> i&nbsp;co najmniej dwie różne trójki kolejnych liczb niekwadratowych modulo <math>p</math>.
 
  
  
 +
Teraz pozostaje dodać sumę wyrazów szeregu od <math>k = 3</math> do <math>k = m</math>
  
 +
::<math>\frac{1}{\log (m + 1)} + \sum_{k = 3}^{m} \frac{1}{k \cdot \log^2 k} < \sum_{k = 3}^{\infty} \frac{1}{k \cdot \log^2 k} < \frac{1}{\log m} + \sum_{k = 3}^{m} \frac{1}{k \cdot \log^2 k}</math>
  
  
== Najmniejsze liczby niekwadratowe modulo ==
+
Poniżej przedstawiamy wartości oszacowania sumy szeregu znalezione przy pomocy programu PARI/GP dla kolejnych wartości <math>m</math>. Wystarczy proste polecenie
  
&nbsp;<br/>
+
for(n=1, 8, s = sum( k = 3, 10^n, 1/k/(log(k))^2 ); print("n= ", n, "  a= ", s+1/log(10^n+1), "  b= ", s+1/log(10^n) ))
 
 
{| style="border-spacing: 5px; border: 2px solid black; background: transparent;"
 
| &nbsp;'''A.''' Najmniejsze dodatnie liczby niekwadratowe modulo <math>p</math>&nbsp;
 
|}
 
 
 
<span style="font-size: 110%; font-weight: bold;">Przykład K13</span><br/>
 
W tabeli przedstawiliśmy najmniejsze dodatnie liczby niekwadratowe modulo <math>p</math>
 
  
 
::{| class="wikitable plainlinks"  style="font-size: 100%; text-align: center; margin-right: auto;"
 
::{| class="wikitable plainlinks"  style="font-size: 100%; text-align: center; margin-right: auto;"
! <math>\boldsymbol{m}</math>
 
| <math>3</math> || <math>5</math> || <math>7</math> || <math>9</math> || <math>11</math> || <math>13</math> || <math>15</math> || <math>17</math> || <math>19</math> || <math>21</math> || <math>23</math> || <math>25</math> || <math>27</math> || <math>29</math> || <math>31</math> || <math>33</math> || <math>35</math> || <math>37</math> || <math>39</math> || <math>41</math> || <math>43</math> || <math>45</math> || <math>47</math> || <math>49</math> || <math>51</math>
 
 
|-
 
|-
!  <math>\boldsymbol{\mathbb{n}( p )}</math>
+
| <math>m = 10^1</math> || <math>1.06</math> || <math>1.07</math>
| <math>2</math> || <math>2</math> || <math>3</math> || <math>-</math> || <math>2</math> || <math>2</math> || <math>-</math> || <math>3</math> || <math>2</math> || <math>-</math> || <math>5</math> || <math>-</math> || <math>-</math> || <math>2</math> || <math>3</math> || <math>-</math> || <math>-</math> || <math>2</math> || <math>-</math> || <math>3</math> || <math>2</math> || <math>-</math> || <math>5</math> || <math>-</math> || <math>-</math>
+
|-
 +
| <math>m = 10^2</math> || <math>1.068</math> || <math>1.069</math>
 +
|-
 +
| <math>m = 10^3</math> || <math>1.06904</math> || <math>1.06906</math>
 +
|-
 +
| <math>m = 10^4</math> || <math>1.069057</math> || <math>1.069058</math>
 +
|-
 +
| <math>m = 10^5</math> || <math>1.0690582</math> || <math>1.0690583</math>
 +
|-
 +
| <math>m = 10^6</math> || <math>1.06905830</math> || <math>1.06905831</math>
 +
|-
 +
| <math>m = 10^7</math> || <math>1.0690583105</math> || <math>1.0690583109</math>
 +
|-
 +
| <math>m = 10^8</math> || <math>1.06905831071</math> || <math>1.06905831074</math>
 
|}
 
|}
  
 +
Dysponując oszacowaniem reszty szeregu, znaleźliśmy wartość sumy szeregu z&nbsp;dokładnością 10 miejsc po przecinku.
  
 +
Natomiast samo zsumowanie <math>10^8</math> wyrazów szeregu daje wynik
  
<span style="font-size: 110%; font-weight: bold;">Uwaga K14</span><br/>
+
::<math>\sum_{k = 3}^{10^8} \frac{1}{k \cdot \log^2 k} = 1.014 771 500 510 916 \ldots</math>
Do wyszukiwania liczb <math>\mathbb{n} = \mathbb{n} (p)</math> Czytelnik może wykorzystać prostą funkcję napisaną w&nbsp;PARI/GP
 
 
 
<span style="font-size: 90%; color:black;">A(p) =
 
{
 
'''if'''( p == 2, '''return'''(0) );
 
'''if'''( !'''isprime'''(p), '''return'''(0) );
 
'''forprime'''(q = 2, p, '''if'''( jacobi(q, p) == -1, '''return'''(q) ));
 
}</span>
 
 
 
Zauważmy, że choć wyliczamy symbol Jacobiego, to jest to w&nbsp;rzeczywistości symbol Legendre'a, '''bo wiemy''', że liczba <math>p</math> jest liczbą pierwszą (w przypadku, gdy <math>p</math> jest liczbą złożoną, funkcja zwraca zero).
 
 
 
 
 
 
 
<span style="font-size: 110%; font-weight: bold;">Twierdzenie K15</span><br/>
 
Niech <math>\mathbb{n} \in \mathbb{Z}_+</math> i&nbsp;niech <math>p</math> będzie liczbą pierwszą nieparzystą. Jeżeli <math>\mathbb{n}</math> jest najmniejszą liczbą niekwadratową modulo <math>p</math>, to jest liczbą pierwszą.
 
 
 
{{Spoiler|Style = font-style: italic; font-weight: bold; color: olive; text-decoration: underline;|Show=Dowód|Hide=Ukryj dowód}}
 
Przypuśćmy, że <math>\mathbb{n} = a b</math> jest liczbą złożoną, gdzie <math>1 < a, b < \mathbb{n}</math>. Z&nbsp;założenia <math>\mathbb{n}</math> jest najmniejszą liczbą niekwadratową modulo <math>p</math>, zatem liczby <math>a, b</math> są liczbami kwadratowymi modulo <math>p</math>. Z&nbsp;definicji liczb kwadratowych muszą istnieć takie liczby <math>r, s</math>, że
 
 
 
::<math>r^2 \equiv a \pmod{p}</math>
 
 
 
::<math>s^2 \equiv b \pmod{p}</math>
 
 
 
Skąd wynika, że
 
 
 
::<math>\mathbb{n} = a b \equiv (r s)^2 \pmod{p}</math>
 
 
 
Wbrew założeniu, że <math>\mathbb{n}</math> jest liczbą niekwadratową modulo <math>p</math>.<br/>
 
&#9633;
 
{{\Spoiler}}
 
 
 
 
 
 
 
<span style="font-size: 110%; font-weight: bold;">Zadanie K16</span><br/>
 
Pokazać, że najmniejszą liczbą niekwadratową modulo <math>p</math> jest
 
 
 
:* &nbsp;liczba <math>2</math> wtedy i&nbsp;tylko wtedy, gdy <math>p = 8 k \pm 3</math>
 
:* &nbsp;liczba <math>3</math> wtedy i&nbsp;tylko wtedy, gdy <math>p = 24 k \pm 7</math>
 
:* &nbsp;liczba <math>\geqslant 5</math> wtedy i&nbsp;tylko wtedy, gdy <math>p = 24 k \pm 1</math>
 
 
 
{{Spoiler|Style = font-style: italic; font-weight: bold; color: olive; text-decoration: underline;|Show=Rozwiązanie|Hide=Ukryj rozwiązanie}}
 
Z właściwości symbolu Legendre'a (zobacz J33 p.7) wiemy, że
 
 
 
::<math>\left( {\small\frac{2}{p}} \right)_{\small{\!\! L}} \,\, =
 
\,\,
 
  \begin{cases}
 
\;\;\: 1 & \text{gdy } p \equiv 1, 7 \pmod{8} \\
 
      - 1 & \text{gdy } p \equiv 3, 5 \pmod{8} \\
 
  \end{cases}</math>
 
 
 
Wynika stąd natychmiast, dla liczb pierwszych <math>p</math> postaci <math>8 k \pm 3</math> (i tylko dla takich liczb) liczba <math>2</math> jest liczbą niekwadratową, czyli również najmniejszą liczbą niekwadratową modulo <math>p</math>.
 
 
 
Z zadania J46 wynika, że liczba <math>3</math> jest liczbą niekwadratową jedynie dla liczb pierwszych postaci <math>12 k \pm 5</math>. Zatem dla liczb pierwszych, które są jednocześnie postaci <math>p = 8 k \pm 1</math> i <math>p = 12 j \pm 5</math>, liczba <math>3</math> jest najmniejszą liczbą niekwadratową modulo <math>p</math>. Z&nbsp;czterech warunków
 
 
 
::<math>p = 8 k + 1 \quad \text{i} \quad p = 12 j + 5</math>
 
 
 
::<math>p = 8 k + 1 \quad \text{i} \quad p = 12 j + 7</math>
 
 
 
::<math>p = 8 k + 7 \quad \text{i} \quad p = 12 j + 5</math>
 
 
 
::<math>p = 8 k + 7 \quad \text{i} \quad p = 12 j + 7</math>
 
 
 
Drugi i&nbsp;trzeci nie są możliwe, bo modulo <math>4</math> otrzymujemy
 
 
 
::<math>p \equiv 1 \pmod{4} \quad \text{i} \quad p \equiv 3 \pmod{4}</math>
 
 
 
::<math>p \equiv 3 \pmod{4} \quad \text{i} \quad p \equiv 1 \pmod{4}</math>
 
 
 
a z&nbsp;pierwszego i&nbsp;czwartego mamy
 
 
 
::<math>3 p = 24 k + 3 \quad \text{i} \quad 2 p = 24 j + 10 \qquad \;\: \Longrightarrow \qquad p = 24 (k - j) - 7 \qquad \Longrightarrow \qquad p \equiv - 7 \pmod{24}</math>
 
 
 
::<math>3 p = 24 k + 21 \quad \text{i} \quad 2 p = 24 j + 14 \qquad \Longrightarrow \qquad p = 24 (k - j) + 7 \qquad \Longrightarrow \qquad p \equiv 7 \pmod{24}</math>
 
 
 
Zauważmy, że problem mogliśmy zapisać w&nbsp;postaci układu kongruencji
 
 
 
::<math>p \equiv \pm 1 \pmod{8}</math>
 
 
 
::<math>p \equiv \pm 5 \pmod{12}</math>
 
 
 
Gdyby moduły tych kongruencji były względnie pierwsze, to każdemu wyborowi znaków odpowiadałaby pewna kongruencja równoważna (zobacz J3). Widzimy, że w&nbsp;przypadku, gdy moduły nie są względnie pierwsze, kongruencja równoważna może istnieć, ale nie musi. Rozwiązując taki problem, wygodnie jest skorzystać z&nbsp;programu PARI/GP. Wystarczy wpisać
 
  
chinese(Mod(1, 8), Mod(5, 12)) = Mod(17, 24)
+
Zatem mimo zsumowania stu milionów(!) wyrazów szeregu otrzymaliśmy rezultat z&nbsp;dokładnością jednego(!) miejsca po przecinku. Co więcej, nie wiemy, jaka jest dokładność uzyskanego rezultatu. Znając oszacowanie od dołu i&nbsp;od góry, dokładność jednego miejsca po przecinku uzyskaliśmy po zsumowaniu dziesięciu(!) wyrazów szeregu.
chinese(Mod(1, 8), Mod(-5, 12)) - błąd
 
chinese(Mod(-1, 8), Mod(5, 12)) - błąd
 
chinese(Mod(-1, 8), Mod(-5, 12)) = Mod(7, 24)
 
  
Ostatni punkt zadania rozwiążemy tą metodą. Liczba większa lub równa <math>5</math> jest najmniejszą liczbą niekwadratową modulo <math>p</math> wtedy i&nbsp;tylko wtedy, gdy liczby <math>2</math> i <math>3</math> są liczbami kwadratowymi modulo <math>p</math>, co oznacza, że liczba pierwsza <math>p</math> spełnia kongruencje
+
Rozpatrywana wyżej sytuacja pokazuje, że w&nbsp;przypadku znajdowania przybliżonej wartości sumy szeregu ważniejsze od sumowania ogromnej ilości wyrazów jest posiadanie oszacowania nieskończonej reszty szeregu. Ponieważ wyznaczenie tego oszacowania na ogół nie jest proste, pokażemy jak ten problem rozwiązać przy pomocy całki oznaczonej.
  
::<math>p \equiv \pm 1 \pmod{8}</math>
 
  
::<math>p \equiv \pm 1 \pmod{12}</math>
 
  
Postępując jak wyżej, otrzymujemy
 
  
chinese(Mod(1, 8), Mod(1, 12)) = Mod(1, 24)
 
chinese(Mod(1, 8), Mod(-1, 12)) - błąd
 
chinese(Mod(-1, 8), Mod(1, 12)) - błąd
 
chinese(Mod(-1, 8), Mod(-1, 12)) = Mod(23, 24)
 
 
Co należało pokazać.<br/>
 
&#9633;
 
{{\Spoiler}}
 
  
 +
== Szeregi nieskończone i&nbsp;całka oznaczona ==
  
 +
<span style="font-size: 110%; font-weight: bold;">Twierdzenie D15</span><br/>
 +
Jeżeli funkcja <math>f(x)</math> jest ciągła, dodatnia i&nbsp;malejąca w&nbsp;przedziale <math>[m, n + 1]</math>, to prawdziwy jest następujący ciąg nierówności
  
<span style="font-size: 110%; font-weight: bold;">Twierdzenie K17</span><br/>
+
::<math>0 \leqslant \int_{m}^{n + 1} f(x) d x \leqslant \sum_{k = m}^{n} f(k) \leqslant f (m) + \int_{m}^{n} f(x) d x</math>
Dla każdej liczby pierwszej <math>p_n</math> istnieje nieskończenie wiele takich liczb pierwszych <math>q</math>, że <math>p_n</math> jest najmniejszą liczbą niekwadratową modulo <math>q</math>.
 
  
 
{{Spoiler|Style = font-style: italic; font-weight: bold; color: olive; text-decoration: underline;|Show=Dowód|Hide=Ukryj dowód}}
 
{{Spoiler|Style = font-style: italic; font-weight: bold; color: olive; text-decoration: underline;|Show=Dowód|Hide=Ukryj dowód}}
Niech <math>2, p_2, \ldots, p_{n - 1}, p_n</math> będą kolejnymi liczbami pierwszymi. Wybierzmy liczbę <math>u</math> tak, aby spełniała układ kongruencji
+
Ponieważ funkcja <math>f(x)</math> jest z&nbsp;założenia ciągła, dodatnia i&nbsp;malejąca, to zamieszczony niżej rysunek dobrze prezentuje problem.
 
 
::<math>\begin{align}
 
u & \equiv 1 \pmod{8 p_2 \cdot \ldots \cdot p_{n - 1}} \\
 
u & \equiv a \pmod{p_n} \\
 
\end{align}</math>
 
  
gdzie <math>a</math> oznacza dowolną liczbą niekwadratową modulo <math>p_n</math>. Na podstawie chińskiego twierdzenia o&nbsp;resztach (zobacz J3) powyższy układ kongruencji może być zapisany w&nbsp;postaci kongruencji równoważnej
+
::[[File: D_Szereg-i-calka-1.png|none]]
  
::<math>u \equiv c \pmod{8 p_2 \cdot \ldots \cdot p_n}</math>
+
Przedstawiona na rysunku krzywa odpowiada funkcji <math>f(x)</math>. Dla współrzędnej <math>x = k</math> zaznaczyliśmy wartość funkcji <math>f(k)</math>, a&nbsp;po lewej i&nbsp;prawej stronie tych punktów zaznaczyliśmy pasy o&nbsp;jednostkowej szerokości. Łatwo zauważamy, że
  
 +
* po lewej stronie pole pod krzywą (zaznaczone kolorem zielonym) jest większe od pola prostokąta o&nbsp;wysokości <math>f(k)</math> i&nbsp;jednostkowej szerokości
 +
* po prawej stronie pole pod krzywą (zaznaczone kolorem niebieskim) jest mniejsze od pola prostokąta o&nbsp;wysokości <math>f(k)</math> i&nbsp;jednostkowej szerokości
  
Zauważmy, że żadna z&nbsp;liczb pierwszych <math>p_k</math>, gdzie <math>1 \leqslant k \leqslant n</math> nie dzieli liczby <math>c</math>, bo mielibyśmy
+
Korzystając z&nbsp;własności całki oznaczonej, otrzymujemy ciąg nierówności
  
::<math>u \equiv 0 \pmod{p_k}</math>
+
::<math>\int_{k}^{k + 1} f(x) d x \leqslant f(k) \leqslant \int_{k - 1}^{k} f(x) d x</math>
  
wbrew wypisanemu wyżej układowi kongruencji. Zatem <math>\gcd (c, 8 p_2 \cdot \ldots \cdot p_n) = 1</math> i&nbsp;z&nbsp;twierdzenia Dirichleta (zobacz C27) wiemy, że wśród liczb <math>u</math> spełniających kongruencję <math>u \equiv c \!\! \pmod{8 p_2 \cdot \ldots \cdot p_n}</math> występuje nieskończenie wiele liczb pierwszych (bo wśród tych liczb są liczby postaci <math>8 p_2 \cdot \ldots \cdot p_n \cdot k + c</math>, gdzie <math>k \in \mathbb{Z}_+</math>). Oznaczmy przez <math>q</math> dowolną z&nbsp;tych liczb pierwszych.
+
W powyższym wzorze występują nierówności nieostre, bo rysunek przedstawia funkcję silnie malejącą, ale zgodnie z&nbsp;uczynionym założeniem funkcja <math>f(x)</math> może być funkcją słabo malejącą.
  
 +
Sumując lewą nierówność od <math>k = m</math> do <math>k = n</math>, a&nbsp;prawą od <math>k = m + 1</math> do <math>k = n</math>, dostajemy
  
Ponieważ <math>q \equiv 1 \!\! \pmod{8}</math>, to <math>\left( {\small\frac{2}{q}} \right)_{\small{\!\! L}} = 1</math> (zobacz J33), a&nbsp;dla wszystkich liczb pierwszych nieparzystych <math>p_k < p_n</math> mamy
+
::<math>\int_{m}^{n + 1} f (x) d x \leqslant \sum_{k = m}^{n} f (k)</math>
  
<div style="margin-top: 1em; margin-bottom: 1em;">
+
::<math>\sum_{k = m + 1}^{n} f (k) \leqslant \int_{m}^{n} f (x) d x</math>
::<math>\left( {\small\frac{p_k}{q}} \right)_{\small{\!\! L}} = \left( {\small\frac{q}{p_k}} \right)_{\small{\!\! L}} \cdot (- 1)^{\tfrac{q - 1}{2} \cdot \tfrac{p_k - 1}{2}} = \left( {\small\frac{q}{p_k}} \right)_{\small{\!\! L}} = \left( {\small\frac{c}{p_k}} \right)_{\small{\!\! L}} = \left( {\small\frac{1}{p_k}} \right)_{\small{\!\! L}} = 1</math>
 
</div>
 
  
bo <math>8 \mid (q - 1)</math>. Dla liczby pierwszej <math>p_n</math> jest
+
Dodając <math>f(m)</math> do obydwu stron drugiej z&nbsp;powyższych nierówności i&nbsp;łącząc je ze sobą, otrzymujemy kolejny i&nbsp;docelowy ciąg nierówności
  
<div style="margin-top: 1em; margin-bottom: 1em;">
+
::<math>0 \leqslant \int_{m}^{n + 1} f (x) d x \leqslant \sum_{k = m}^{n} f (k) \leqslant f (m) + \int_{m}^{n} f (x) d x</math><br/>
::<math>\left( {\small\frac{p_n}{q}} \right)_{\small{\!\! L}} = \left( {\small\frac{q}{p_n}} \right)_{\small{\!\! L}} \cdot (- 1)^{\tfrac{q - 1}{2} \cdot \tfrac{p_n - 1}{2}} = \left( {\small\frac{q}{p_n}} \right)_{\small{\!\! L}} = \left( {\small\frac{c}{p_n}} \right)_{\small{\!\! L}} = \left( {\small\frac{a}{p_n}} \right)_{\small{\!\! L}} = - 1</math>
 
</div>
 
 
 
Zatem wszystkie liczby pierwsze mniejsze od <math>p_n</math> są liczbami kwadratowymi modulo <math>q</math>, a&nbsp;liczba pierwsza <math>p_n</math> jest najmniejszą liczbą niekwadratową modulo <math>q</math>. Zauważmy, że <math>q</math> była dowolnie wybraną liczbą pierwszą z&nbsp;nieskończenie wielu liczb pierwszych występujących w&nbsp;ciągu arytmetycznym <math>8 p_2 \cdot \ldots \cdot p_n \cdot k + c</math>, gdzie <math>k \in \mathbb{Z}_+</math>. Co kończy dowód.<br/>
 
 
&#9633;
 
&#9633;
 
{{\Spoiler}}
 
{{\Spoiler}}
Linia 1029: Linia 518:
  
  
<span style="font-size: 110%; font-weight: bold;">Twierdzenie K18 (Sarvadaman Chowla)</span><br/>
+
<span style="font-size: 110%; font-weight: bold;">Przykład D16</span><br/>
Istnieje niekończenie wiele liczb pierwszych <math>p</math> takich, że najmniejsza liczba niekwadratowa modulo <math>p</math> jest większa od <math>{\small\frac{\log p}{2 L \log 2}}</math>, gdzie <math>L</math> jest stałą Linnika.
+
Rozważmy szereg <math>\sum_{k = 1}^{\infty} \frac{1}{k}</math>.
 
 
{{Spoiler|Style = font-style: italic; font-weight: bold; color: olive; text-decoration: underline;|Show=Dowód|Hide=Ukryj dowód}}
 
Niech <math>a = 4 P (m)</math>, gdzie <math>P(m)</math> jest iloczynem wszystkich liczb pierwszych nie większych od <math>m</math>. Z&nbsp;twierdzenia Dirichleta (zobacz C27) wiemy, że w&nbsp;ciągu arytmetycznym <math>u_k = a k + 1</math> występuje nieskończenie wiele liczb pierwszych. Niech <math>p</math> oznacza dowolną z&nbsp;nich.
 
 
 
Ponieważ <math>p \equiv 1 \!\! \pmod{8}</math>, to
 
 
 
::<math>\left( {\small\frac{2}{p}} \right)_{\small{\!\! L}} = 1</math>
 
 
 
(zobacz J33 p.7). Oczywiście <math>p \equiv 1 \!\! \pmod{4}</math>, zatem dla dowolnej liczby pierwszej nieparzystej <math>q_i \leqslant m</math> z&nbsp;twierdzenia J33 p.9 otrzymujemy
 
  
<div style="margin-top: 1em; margin-bottom: 1em;">
+
Funkcja <math>f(x) = \frac{1}{x}</math> jest ciągła, dodatnia i&nbsp;silnie malejąca w&nbsp;przedziale <math>(0, + \infty)</math>, zatem dla dowolnego <math>n \in \mathbb{Z}_+</math> prawdziwe jest oszacowanie
::<math>\left( {\small\frac{q_i}{p}} \right)_{\small{\!\! L}} = \left( {\small\frac{p}{q_i}} \right)_{\small{\!\! L}} = \left( {\small\frac{a k + 1}{q_i}} \right)_{\small{\!\! L}} = \left( {\small\frac{1}{q_i}} \right)_{\small{\!\! L}} = 1</math>
 
</div>
 
  
Wynika stąd, że najmniejsza liczba niekwadratowa modulo <math>p</math> jest większa od <math>m</math>. Wiemy też, że (zobacz A9)
+
::<math>\int_{1}^{n + 1} \frac{d x}{x} < \sum_{k = 1}^{n} \frac{1}{k} < 1 + \int_{1}^{n} \frac{d x}{x}</math>
  
::<math>a = 4 P (m) < 4 \cdot 4^m = 4^{m + 1}</math>
+
Przy obliczaniu całek oznaczonych Czytelnik może skorzystać ze strony [https://www.wolframalpha.com/input?i=integral+1%2Fx+from+1+to+n WolframAlpha].
  
Załóżmy teraz, że <math>p</math> jest najmniejszą liczbą pierwszą w&nbsp;ciągu arytmetycznym <math>u_k = a k + 1</math>, a&nbsp;liczba <math>m</math> została wybrana tak, że liczba <math>a = 4 P (m)</math> jest dostatecznie duża i&nbsp;możliwe jest skorzystanie z&nbsp;twierdzenia Linnika (zobacz C30). Dostajemy natychmiast oszacowanie
+
::<math>\log (n + 1) < \sum_{k = 1}^{n} \frac{1}{k} < 1 + \log n</math>
  
::<math>p = p_{\min} (a, 1) < a^L</math>
+
Ponieważ
  
gdzie <math>L</math> jest stałą Linnika (możemy przyjąć <math>L = 5</math>). Łącząc powyższe oszacowania, łatwo otrzymujemy oszacowanie najmniejszej liczby niekwadratowej modulo <math>p</math>
+
::<math>\log (n + 1) = \log \left( n \left( 1 + \frac{1}{n} \right) \right) = \log n + \log \left( 1 + \frac{1}{n} \right) > \log n + \frac{1}{n + 1}</math>
 
 
::<math>\mathbb{n}(p) \geqslant m + 1 > \log_4 a = {\small\frac{\log a}{\log 4}} = {\small\frac{\log a^L}{2 L \log 2}} > {\small\frac{\log p}{2 L \log 2}}</math>
 
 
 
Każdemu wyborowi innej liczby <math>m' > m</math> takiej, że <math>P(m') > P (m)</math> odpowiada inna liczba pierwsza <math>p'</math> taka, że <math>\mathbb{n}(p') > {\small\frac{\log p'}{2 L \log 2}}</math>, zatem liczb pierwszych <math>p</math> dla których najmniejsza liczba niekwadratowa modulo <math>p</math> jest większa od <math>{\small\frac{\log p}{2 L \log 2}}</math> jest nieskończenie wiele.<br/>
 
&#9633;
 
{{\Spoiler}}
 
  
 +
to dostajemy
  
 +
::<math>\frac{1}{n + 1} < \sum_{k = 1}^{n} \frac{1}{k} - \log n < 1</math>
  
<span style="font-size: 110%; font-weight: bold;">Uwaga K19</span><br/>
+
Zauważmy: nie tylko wiemy, że szereg <math>\sum_{k = 1}^{\infty} \frac{1}{k}</math> jest rozbieżny, ale jeszcze potrafimy określić, jaka funkcja tę rozbieżność opisuje! Mamy zatem podstawy, by przypuszczać, że całki umożliwią opracowanie metody, która pozwoli rozstrzygać o&nbsp;zbieżności szeregów.
W twierdzeniu K17 pokazaliśmy, że dla każdej liczby pierwszej <math>\mathbb{n}</math> istnieją takie liczby pierwsze <math>p</math>, że <math>\mathbb{n}</math> jest najmniejszą liczbą niekwadratową modulo <math>p</math>. Zatem zbiór <math>S_\mathbb{n}</math> liczb pierwszych takich, że dla każdej liczby <math>p \in S_\mathbb{n}</math> liczba <math>\mathbb{n}</math> jest najmniejszą liczbą niekwadratową modulo <math>p</math> jest zbiorem niepustym. Wynika stąd, że zbiór <math>S_\mathbb{n}</math> ma element najmniejszy i&nbsp;możemy te najmniejsze liczby pierwsze łatwo znaleźć – wystarczy w&nbsp;PARI/GP napisać proste polecenie
 
  
<span style="font-size: 90%; color:black;">'''forprime'''(n = 2, 50, '''forprime'''(p = 2, 10^10, '''if'''( A(p) == n, '''print'''(n, "  ", p); '''break'''() )))</span>
 
  
W tabeli przedstawiamy uzyskane rezultaty (zobacz też [https://oeis.org/A000229 A000229]).
 
  
::{| class="wikitable plainlinks"  style="font-size: 100%; text-align: center; margin-right: auto;"
 
|-
 
! <math>\boldsymbol{\mathbb{n}}</math>
 
| <math>2</math> || <math>3</math> || <math>5</math> || <math>7</math> || <math>11</math> || <math>13</math> || <math>17</math> || <math>19</math> || <math>23</math> || <math>29</math> || <math>31</math> || <math>37</math> || <math>41</math> || <math>43</math> || <math>47</math>
 
|-
 
! <math>\boldsymbol{p}</math>
 
| <math>3</math> || <math>7</math> || <math>23</math> || <math>71</math> || <math>311</math> || <math>479</math> || <math>1559</math> || <math>5711</math> || <math>10559</math> || <math>18191</math> || <math>31391</math> || <math>422231</math> || <math>701399</math> || <math>366791</math> || <math>3818929</math>
 
|}
 
  
 
+
<span style="font-size: 110%; font-weight: bold;">Twierdzenie D17 (kryterium całkowe zbieżności szeregów)</span><br/>
 
+
Załóżmy, że funkcja <math>f(x)</math> jest ciągła, dodatnia i&nbsp;malejąca w&nbsp;przedziale <math>[m, + \infty)</math>. Szereg <math>\sum_{k = m}^{\infty} f(k)</math> jest zbieżny lub rozbieżny w&nbsp;zależności od tego, czy funkcja pierwotna <math>F(x) = \int f (x) d x</math> ma dla <math>x \rightarrow \infty</math> granicę skończoną, czy nie.
<span style="font-size: 110%; font-weight: bold;">Uwaga K20</span><br/>
 
Z nierówności Pólyi-Winogradowa (zobacz K2) wynika natychmiast oszacowanie najmniejszej liczby niekwadratowej modulo <math>p</math>. Ponieważ najdłuższy ciąg kolejnych liczb kwadratowych modulo <math>p</math> nie może być dłuższy od <math>\left\lfloor \sqrt{p} \log p \right\rfloor</math>, to
 
 
 
::<math>\mathbb{n} (p) \leqslant \left\lfloor \sqrt{p} \log p \right\rfloor + 1 < \sqrt{p} \log p + 1</math>
 
 
 
Pokażemy, że powyższe oszacowanie można łatwo wzmocnić.
 
 
 
 
 
 
 
<span style="font-size: 110%; font-weight: bold;">Twierdzenie K21</span><br/>
 
Niech <math>p</math> będzie liczbą pierwszą nieparzystą, a <math>\mathbb{n}</math> będzie najmniejszą liczbą niekwadratową modulo <math>p</math>. Prawdziwe jest oszacowanie
 
 
 
::<math>\mathbb{n} (p) < \sqrt{p} + {\small\frac{1}{2}}</math>
 
  
 
{{Spoiler|Style = font-style: italic; font-weight: bold; color: olive; text-decoration: underline;|Show=Dowód|Hide=Ukryj dowód}}
 
{{Spoiler|Style = font-style: italic; font-weight: bold; color: olive; text-decoration: underline;|Show=Dowód|Hide=Ukryj dowód}}
Ponieważ <math>\mathbb{n} \nmid p</math>, to z&nbsp;oszacowania <math>x - 1 < \lfloor x \rfloor \leqslant x</math> wynika, że
+
Nim przejdziemy do dowodu, wyjaśnimy uczynione założenia. Założenie, że funkcja <math>f(x)</math> jest malejąca, będzie wykorzystane w&nbsp;czasie dowodu twierdzenia, ale rozważanie przypadku, gdy <math>f(x)</math> jest rosnąca, nie ma sensu, bo wtedy nie mógłby być spełniony warunek konieczny zbieżności szeregu <math>\sum_{k = m}^{\infty} f(k)</math> (zobacz twierdzenie D4).
  
::<math>{\small\frac{p}{\mathbb{n}}} - 1 < \left\lfloor {\small\frac{p}{\mathbb{n}}} \right\rfloor < {\small\frac{p}{\mathbb{n}}}</math>
+
Moglibyśmy założyć bardziej ogólnie, że funkcja jest nieujemna, ale wtedy twierdzenie obejmowałoby przypadki funkcji takich, że dla pewnego <math>x_0</math> byłoby <math>f(x_0) = 0</math>. Ponieważ z&nbsp;założenia funkcja <math>f(x)</math> jest malejąca, zatem mielibyśmy <math>f(x) = 0</math> dla <math>x \geqslant x_0</math>. Odpowiadający tej funkcji szereg <math>\sum_{k = m}^{\infty} f (k)</math> miałby dla <math>k \geqslant x_0</math> tylko wyrazy zerowe i&nbsp;byłby w&nbsp;sposób oczywisty zbieżny.
  
::<math>p < \mathbb{n} \left\lfloor {\small\frac{p}{\mathbb{n}}} \right\rfloor + \mathbb{n} < p + \mathbb{n}</math>
+
Założenie ciągłości funkcji <math>f(x)</math> ma zapewnić całkowalność funkcji <math>f(x)</math><ref name="calkowalnosc1"/>. Założenie to można osłabić<ref name="calkowalnosc2"/>, tutaj ograniczymy się tylko do podania przykładów. Niech <math>a, b \in \mathbb{R}</math>, mamy
  
Niech <math>u = \left\lfloor {\small\frac{p}{\mathbb{n}}} \right\rfloor + 1</math>, mamy
+
::<math>\int_a^b \text{sgn}(x) d x = | b | - | a |</math> <math>\qquad \qquad \int_0^a \lfloor x \rfloor d x = \frac{1}{2} \lfloor a \rfloor (2 a - \lfloor a \rfloor - 1)</math> <math>\qquad \qquad \int_{-a}^a \lfloor x \rfloor d x = - a</math>
  
::<math>0 < \mathbb{n} u - p < \mathbb{n}</math>
 
  
Liczba <math>\mathbb{n} u - p</math> musi być liczbą kwadratową modulo <math>p</math>, zatem
+
Po tych uwagach dotyczących założeń możemy przejść do właściwego dowodu. Korzystając ze wzoru udowodnionego w&nbsp;twierdzeniu D15 i&nbsp;przechodząc z <math>n</math> do nieskończoności, dostajemy
  
::<math>1 = \left( {\small\frac{\mathbb{n} u - p}{p}} \right)_{\small{\!\! L}} = \left( {\small\frac{\mathbb{n}}{p}} \right)_{\small{\!\! L}} \cdot \left( {\small\frac{u}{p}} \right)_{\small{\!\! L}} = - \left( {\small\frac{u}{p}} \right)_{\small{\!\! L}}</math>
+
::<math>0 \leqslant \int_{m}^{\infty} f(x) d x \leqslant \sum_{k = m}^{\infty} f(k) \leqslant f (m) + \int_{m}^{\infty} f(x) d x</math>
  
Ale z&nbsp;założenia <math>\mathbb{n}</math> jest najmniejszą liczbą taką, że <math>\left( {\small\frac{\mathbb{n}}{p}} \right)_{\small{\!\! L}} = - 1</math>. Wynika stąd, że musi być <math>\mathbb{n} \leqslant u</math> i&nbsp;łatwo znajdujemy, że
 
  
::<math>\mathbb{n} \leqslant \left\lfloor {\small\frac{p}{\mathbb{n}}} \right\rfloor + 1 < {\small\frac{p}{\mathbb{n}}} + 1</math>
+
'''Z drugiej nierówności wynika''', że jeżeli całka <math>\int_{m}^{\infty} f(x) d x</math> jest rozbieżna, to rosnący ciąg kolejnych całek oznaczonych <math>C_j = \int_{m}^{j} f (x) d x</math> nie może być ograniczony od góry (w&nbsp;przeciwnym wypadku całka <math>\int_{m}^{\infty} f (x) d x</math> byłby zbieżna), zatem również rosnący ciąg sum częściowych <math>F_j = \sum_{k = m}^{j} f(k)</math> nie może być ograniczony od góry, co oznacza, że szereg <math>\sum_{k = m}^{\infty} f(k)</math> jest rozbieżny.
  
::<math>\mathbb{n}^2 < p + \mathbb{n}</math>
+
'''Z trzeciej nierówności wynika''', że jeżeli całka <math>\int_{m}^{\infty} f(x) d x</math> jest zbieżna, to ciąg sum częściowych <math>F_j = \sum_{k = m}^{j} f (k)</math> jest ciągiem rosnącym i&nbsp;ograniczonym od góry. Wynika stąd, że ciąg <math>F_j</math> jest zbieżny, zatem szereg <math>\sum_{k = m}^{\infty} f(k)</math> jest zbieżny.
  
Ponieważ wypisane liczby są liczbami całkowitymi, to ostatnią nierówność możemy zapisać w&nbsp;postaci
+
Ponieważ zbieżność (rozbieżność) całki <math>\int_{m}^{\infty} f(x) d x</math> nie zależy od wyboru dolnej granicy całkowania, to wystarczy badać granicę <math>\lim_{x \to \infty} F (x)</math>, gdzie <math>F(x) = \int f (x) d x</math> jest dowolną funkcją pierwotną.<br/>
 
 
::<math>\mathbb{n}^2 \leqslant p + \mathbb{n} - 1</math>
 
 
 
Skąd otrzymujemy
 
 
 
::<math>\left( \mathbb{n} - {\small\frac{1}{2}} \right)^2 \leqslant p - {\small\frac{3}{4}}</math>
 
 
 
::<math>\mathbb{n} \leqslant {\small\frac{1}{2}} + \sqrt{p - {\small\frac{3}{4}}} < {\small\frac{1}{2}} + \sqrt{p}</math>
 
 
 
Co należało pokazać.<br/>
 
 
&#9633;
 
&#9633;
 
{{\Spoiler}}
 
{{\Spoiler}}
Linia 1132: Linia 570:
  
  
<span style="font-size: 110%; font-weight: bold;">Twierdzenie K22*</span><br/>
+
<span style="font-size: 110%; font-weight: bold;">Przykład D18</span><br/>
Niech <math>p</math> będzie liczbą pierwszą nieparzystą, a <math>\mathbb{n}</math> będzie najmniejszą liczbą niekwadratową modulo <math>p</math>. Dla <math>p \geqslant 5</math> prawdziwe jest oszacowanie<ref name="Norton1"/><ref name="Trevino1"/><ref name="Trevino2"/>
+
Przykłady zebraliśmy w&nbsp;tabeli. Przy obliczaniu całek nieoznaczonych Czytelnik może skorzystać ze strony [https://www.wolframalpha.com/input?i=integral+1%2Fsqrt%28x%29 WolframAlpha].
 
 
::<math>\mathbb{n} (p) \leqslant 1.1 \cdot p^{1 / 4} \log p</math>
 
 
 
 
 
 
 
<span style="font-size: 110%; font-weight: bold;">Uwaga K23</span><br/>
 
Liczby <math>\mathbb{n} = \mathbb{n} (p)</math> są zaskakująco małe. Średnia wartość <math>\mathbb{n} = \mathbb{n} (p)</math>, gdzie <math>p</math> są nieparzystymi liczbami pierwszymi, jest równa<ref name="Erdos1"/>
 
 
 
::<math>\lim_{x \to \infty} {\small\frac{1}{\pi (x)}} \sum_{p \leqslant x} \mathbb{n} (p) = \sum_{k = 1}^{\infty} {\small\frac{p_k}{2^k}} = 3.674643966 \ldots</math>
 
 
 
 
 
 
 
<span style="font-size: 110%; font-weight: bold;">Uwaga K24</span><br/>
 
Możemy też badać najmniejsze '''nieparzyste''' liczby niekwadratowe modulo <math>p</math>. Pokażemy, że są one również liczbami pierwszymi. W tabeli przedstawiliśmy najmniejsze '''nieparzyste''' liczby niekwadratowe modulo <math>p</math>.
 
  
 
::{| class="wikitable plainlinks"  style="font-size: 100%; text-align: center; margin-right: auto;"
 
::{| class="wikitable plainlinks"  style="font-size: 100%; text-align: center; margin-right: auto;"
 +
!
 +
! szereg <math>\sum_{k = m}^{\infty} a_k</math>
 +
! funkcja <math>f(x)</math>
 +
! całka <math>F(x) = \int f(x) d x</math>
 +
! granica <math>\lim_{x \to \infty} F(x)</math>
 +
! wynik
 
|-
 
|-
! <math>\boldsymbol{m}</math>
+
| 1. || <math>\sum_{k = 1}^{\infty} \frac{1}{k}</math> || <math>\frac{1}{x}</math> || <math>\log x</math> || <math>\infty</math> || szereg rozbieżny
| <math>5</math> || <math>7</math> || <math>9</math> || <math>11</math> || <math>13</math> || <math>15</math> || <math>17</math> || <math>19</math> || <math>21</math> || <math>23</math> || <math>25</math> || <math>27</math> || <math>29</math> || <math>31</math> || <math>33</math> || <math>35</math> || <math>37</math> || <math>39</math> || <math>41</math> || <math>43</math> || <math>45</math> || <math>47</math> || <math>49</math> || <math>51</math>
 
|-
 
! <math>\boldsymbol{\mathbb{n}_1( p )}</math>
 
| <math>3</math> || <math>3</math> || <math>-</math> || <math>7</math> || <math>5</math> || <math>-</math> || <math>3</math> || <math>3</math> || <math>-</math> || <math>5</math> || <math>-</math> || <math>-</math> || <math>3</math> || <math>3</math> || <math>-</math> || <math>-</math> || <math>5</math> || <math>-</math> || <math>3</math> || <math>3</math> || <math>-</math> || <math>5</math> || <math>-</math> || <math>-</math>
 
|}
 
 
 
 
 
 
 
<span style="font-size: 110%; font-weight: bold;">Twierdzenie K25</span><br/>
 
Dla każdej liczby pierwszej <math>p \geqslant 5</math> najmniejsza '''nieparzysta''' liczba niekwadratowa modulo <math>p</math> jest liczbą pierwszą mniejszą od <math>p</math>.
 
 
 
{{Spoiler|Style = font-style: italic; font-weight: bold; color: olive; text-decoration: underline;|Show=Dowód|Hide=Ukryj dowód}}
 
Niech <math>S \subset \{ 1, 2, \ldots, p - 1 \}</math> będzie zbiorem wszystkich '''nieparzystych''' liczb niekwadratowych modulo <math>p</math>. Z&nbsp;twierdzenia J29 wiemy, że jeżeli <math>p</math> jest liczbą pierwszą nieparzystą, to w&nbsp;zbiorze <math>\{ 1, 2, \ldots, p - 1 \}</math> jest dokładnie <math>{\small\frac{p - 1}{2}}</math> liczb kwadratowych modulo <math>p</math> i&nbsp;tyle samo liczb niekwadratowych modulo <math>p</math>. W&nbsp;zbiorze <math>\{ 1, 2, \ldots, p - 1 \}</math> mamy też dokładnie <math>{\small\frac{p - 1}{2}}</math> liczb parzystych i&nbsp;tyle samo liczb nieparzystych.
 
 
 
Wszystkie liczby parzyste nie mogą być liczbami niekwadratowymi modulo <math>p</math>, bo <math>4 = 2^2 < 5 \leqslant p</math> jest parzystą liczbą kwadratową modulo <math>p</math>, czyli wśród liczb nieparzystych musi istnieć przynajmniej jedna liczba niekwadratowa modulo <math>p</math>. Wynika stąd, że zbiór <math>S</math> nie jest zbiorem pustym, zatem ma element najmniejszy. Pokażemy, że najmniejszy element zbioru <math>S</math> jest liczbą pierwszą.
 
 
 
Niech <math>3 \leqslant \mathbb{n}_\boldsymbol{1} \leqslant p - 2</math> będzie najmniejszą '''nieparzystą''' liczbą niekwadratową modulo <math>p</math>. Wynika stąd, że każda liczba <math>a < \mathbb{n}_\boldsymbol{1}</math> musi być liczbą parzystą lub liczbą kwadratową modulo <math>p</math>. Przypuśćmy, że <math>\mathbb{n}_\boldsymbol{1}</math> jest liczbą złożoną, czyli <math>\mathbb{n}_\boldsymbol{1} = a b</math>, gdzie <math>1 < a, b < \mathbb{n}_\boldsymbol{1}</math>. Zauważmy, że żadna z&nbsp;liczb <math>a, b</math> nie może być liczbą parzystą, bo wtedy liczba <math>\mathbb{n}_\boldsymbol{1}</math> również byłaby liczbą parzystą wbrew określeniu liczby <math>\mathbb{n}_\boldsymbol{1}</math>. Zatem obie liczby <math>a, b</math> muszą być nieparzystymi liczbami kwadratowymi, co jest niemożliwe, bo
 
 
 
::<math>- 1 = \left( {\small\frac{\mathbb{n}_\boldsymbol{1}}{p}} \right)_{\small{\!\! J}} = \left( {\small\frac{a b}{p}} \right)_{\small{\!\! J}} = \left( {\small\frac{a}{p}} \right)_{\small{\!\! J}} \cdot \left( {\small\frac{b}{p}} \right)_{\small{\!\! J}}</math>
 
 
 
i jeden z&nbsp;czynników po prawej stronie musi być ujemny. Co oznacza, że jedna z&nbsp;liczb <math>a, b</math> jest nieparzystą liczbą niekwadratową modulo <math>p</math> mniejszą od <math>\mathbb{n}_\boldsymbol{1}</math> wbrew określeniu liczby <math>\mathbb{n}_\boldsymbol{1}</math>. Uzyskana sprzeczność pokazuje, że liczba <math>\mathbb{n}_\boldsymbol{1}</math> jest liczbą pierwszą. Co kończy dowód.<br/>
 
&#9633;
 
{{\Spoiler}}
 
 
 
 
 
 
 
 
 
 
 
{| style="border-spacing: 5px; border: 2px solid black; background: transparent;"
 
| &nbsp;'''B.''' Najmniejsze dodatnie liczby niekwadratowe modulo <math>m</math>
 
|}
 
 
 
<span style="font-size: 110%; font-weight: bold;">Uwaga K26</span><br/>
 
Najmniejsze liczby niekwadratowe modulo <math>m</math> są naturalnym uogólnieniem najmniejszych liczb niekwadratowych modulo <math>p .</math> W&nbsp;jednym i&nbsp;drugim przypadku liczba <math>\mathbb{n}</math> jest najmniejszą liczbą niekwadratową w&nbsp;zbiorze wszystkich liczb niekwadratowych dodatnich nie większych od <math>p</math> lub <math>m .</math> Dlatego będziemy je oznaczali również jako <math>\mathbb{n}(m) .</math>
 
 
 
 
 
 
 
<span style="font-size: 110%; font-weight: bold;">Definicja K27</span><br/>
 
Niech <math>m \in \mathbb{Z} \,</math> i <math>\, m \geqslant 3 .</math> Powiemy, że <math>\mathbb{n} (m)</math> jest najmniejszą liczbą niekwadratową modulo <math>m</math>, gdy <math>\mathbb{n}</math> jest najmniejszą liczbą dodatnią względnie pierwszą z <math>m</math> taką, że kongruencja
 
 
 
::<math>x^2 \equiv \mathbb{n} \pmod{m}</math>
 
 
 
nie ma rozwiązania.
 
 
 
 
 
 
 
<span style="font-size: 110%; font-weight: bold;">Przykład K28</span><br/>
 
W tabeli przedstawiliśmy najmniejsze liczby niekwadratowe modulo <math>p</math> i&nbsp;najmniejsze liczby niekwadratowe modulo <math>m .</math>
 
 
 
::{| class="wikitable plainlinks"  style="font-size: 100%; text-align: center; margin-right: auto;"
 
! <math>\boldsymbol{m}</math>
 
| <math>3</math> || <math>5</math> || <math>7</math> || <math>9</math> || <math>11</math> || <math>13</math> || <math>15</math> || <math>17</math> || <math>19</math> || <math>21</math> || <math>23</math> || <math>25</math> || <math>27</math> || <math>29</math> || <math>31</math> || <math>33</math> || <math>35</math> || <math>37</math> || <math>39</math> || <math>41</math> || <math>43</math> || <math>45</math> || <math>47</math> || <math>49</math> || <math>51</math>
 
 
|-
 
|-
! <math>\boldsymbol{\mathbb{n}( p )}</math>
+
| 2. || <math>\sum_{k = 1}^{\infty} \frac{1}{\sqrt{k}}</math> || <math>\frac{1}{\sqrt{x}}</math> || <math>2 \sqrt{x}</math> || <math>\infty</math> || szereg rozbieżny
| <math>2</math> || <math>2</math> || <math>3</math> || <math>-</math> || <math>2</math> || <math>2</math> || <math>-</math> || <math>3</math> || <math>2</math> || <math>-</math> || <math>5</math> || <math>-</math> || <math>-</math> || <math>2</math> || <math>3</math> || <math>-</math> || <math>-</math> || <math>2</math> || <math>-</math> || <math>3</math> || <math>2</math> || <math>-</math> || <math>5</math> || <math>-</math> || <math>-</math>
 
 
|-
 
|-
! <math>\boldsymbol{\mathbb{n}( m )}</math>
+
| 3. || <math>\sum_{k = 1}^{\infty} \frac{1}{k^2}</math> || <math>\frac{1}{x^2}</math> || <math>- \frac{1}{x}</math> || <math>0</math> || szereg zbieżny
| <math>2</math> || <math>2</math> || <math>3</math> || <math>2</math> || <math>2</math> || <math>2</math> || <math>2</math> || <math>3</math> || <math>2</math> || <math>2</math> || <math>5</math> || <math>2</math> || <math>2</math> || <math>2</math> || <math>3</math> || <math>2</math> || <math>2</math> || <math>2</math> || <math>2</math> || <math>3</math> || <math>2</math> || <math>2</math> || <math>5</math> || <math>3</math> || <math>2</math>
 
|}
 
 
 
::{| class="wikitable plainlinks"  style="font-size: 100%; text-align: center; margin-right: auto;"
 
 
|-
 
|-
! <math>\boldsymbol{m}</math>
+
| 4. || <math>\sum_{k = 2}^{\infty} \frac{1}{k \log k}</math> || <math>\frac{1}{x \log x}</math> || <math>\log \log x</math> || <math>\infty</math> || szereg rozbieżny
| <math>4</math> || <math>6</math> || <math>8</math> || <math>10</math> || <math>12</math> || <math>14</math> || <math>16</math> || <math>18</math> || <math>20</math> || <math>22</math> || <math>24</math> || <math>26</math> || <math>28</math> || <math>30</math> || <math>32</math> || <math>34</math> || <math>36</math> || <math>38</math> || <math>40</math> || <math>42</math> || <math>44</math> || <math>46</math> || <math>48</math> || <math>50</math> || <math>52</math>
 
 
|-
 
|-
! <math>\boldsymbol{\mathbb{n}( m )}</math>
+
| 5. || <math>\sum_{k = 2}^{\infty} \frac{1}{k \log^2 \! k}</math> || <math>\frac{1}{x \log^2 \! x}</math> || <math>- \frac{1}{\log x}</math> || <math>0</math> || szereg zbieżny
| <math>3</math> || <math>5</math> || <math>3</math> || <math>3</math> || <math>5</math> || <math>3</math> || <math>3</math> || <math>5</math> || <math>3</math> || <math>7</math> || <math>5</math> || <math>5</math> || <math>3</math> || <math>7</math> || <math>3</math> || <math>3</math> || <math>5</math> || <math>3</math> || <math>3</math> || <math>5</math> || <math>3</math> || <math>5</math> || <math>5</math> || <math>3</math> || <math>3</math>
 
 
|}
 
|}
  
 +
Stosując kryterium całkowe można łatwo pokazać, że szeregi
  
 +
::<math>\sum_{k = 1}^{\infty} \frac{1}{k^s}</math>
  
<span style="font-size: 110%; font-weight: bold;">Uwaga K29</span><br/>
+
::<math>\sum_{k = 2}^{\infty} \frac{1}{k \log^s \! k}</math>
Do wyszukiwania liczb <math>\mathbb{n} (m)</math> Czytelnik może wykorzystać prostą funkcję napisaną w&nbsp;PARI/GP
 
  
<span style="font-size: 90%; color:black;">B(m) =
+
są zbieżne dla <math>s > 1</math> i&nbsp;rozbieżne dla <math>s \leqslant 1</math>.
{
 
'''local'''(p, res);
 
p = 1;
 
'''while'''( p < m,
 
        p = '''nextprime'''(p + 1);
 
        '''if'''( m%p == 0, '''next'''() );
 
        res = -1;
 
        '''for'''( k = 2, '''floor'''(m/2), '''if'''( k^2%m == p, res = 1; '''break'''() ) );
 
        '''if'''( res == -1, '''return'''(p) );
 
      );
 
}</span>
 
  
Obliczenia można wielokrotnie przyspieszyć, modyfikując kod funkcji tak, aby uwzględniał pokazane niżej właściwości oraz parzystość liczby <math>m .</math> Tutaj przedstawiamy tylko przykład, który wykorzystuje część tych możliwości.
 
  
{{Spoiler|Style = font-style: italic; font-weight: bold; color: olive; text-decoration: underline;|Show=Pokaż kod|Hide=Ukryj kod}}
 
<span style="font-size: 90%; color:black;">B(m) =
 
{
 
'''local'''(p, res, t);
 
t = m%8;
 
'''if'''( t == 3 || t == 5, '''return'''(2) );
 
t = m%12;
 
'''if'''( t == 4 || t == 8, '''return'''(3) );
 
t = m%24;
 
'''if'''( t == 9 || t == 15, '''return'''(2) );
 
'''if'''( t == 10 || t == 14, '''return'''(3) );
 
t = m%30;
 
'''if'''( t == 6 || t == 12 || t == 18 || t == 24, '''return'''(5) );
 
p = 1;
 
'''while'''( p < m,
 
        p = '''nextprime'''(p + 1);
 
        '''if'''( m%p == 0, '''next'''() );
 
        res = -1;
 
        '''for'''( k = 2, '''floor'''(m/2), '''if'''( k^2%m == p, res = 1; '''break'''() ) );
 
        '''if'''( res == -1, '''return'''(p) );
 
      );
 
}</span>
 
{{\Spoiler}}
 
  
  
 +
<span style="font-size: 110%; font-weight: bold;">Twierdzenie D19</span><br/>
 +
Jeżeli funkcja <math>f(x)</math> jest ciągła, dodatnia i&nbsp;malejąca w&nbsp;przedziale <math>[m, \infty)</math> oraz
  
<span style="font-size: 110%; font-weight: bold;">Twierdzenie K30</span><br/>
+
::<math>R(m) = \int_{m}^{\infty} f(x) d x</math>
Niech <math>m \in \mathbb{Z} \,</math> i <math>\, m \geqslant 3 .</math> Jeżeli <math>\mathbb{n}</math> jest najmniejszą liczbą niekwadratową modulo <math>m</math>, to <math>\mathbb{n}</math> jest liczbą pierwszą.
 
  
{{Spoiler|Style = font-style: italic; font-weight: bold; color: olive; text-decoration: underline;|Show=Dowód|Hide=Ukryj dowód}}
+
::<math>S(m) = \sum_{k = a}^{m} f(k)</math>
Przypuśćmy, że <math>\mathbb{n} = a b</math> jest liczbą złożoną, gdzie <math>1 < a, b < \mathbb{n} .</math> Z&nbsp;założenia <math>\mathbb{n}</math> jest najmniejszą liczbą niekwadratową modulo <math>m</math>, zatem liczby <math>a, b</math> są liczbami kwadratowymi modulo <math>m .</math> Z&nbsp;definicji liczb kwadratowych muszą istnieć takie liczby <math>r, s</math>, że
 
  
::<math>r^2 \equiv a \pmod{m}</math>
+
gdzie <math>a < m</math>, to prawdziwe jest następujące oszacowanie sumy szeregu nieskończonego <math>\sum_{k = a}^{\infty} f (k)</math>
  
::<math>s^2 \equiv b \pmod{m}</math>
+
::<math>S(m) + R(m) - f(m) \leqslant \sum_{k = a}^{\infty} f(k) \leqslant S(m) + R(m)</math>
  
Skąd wynika, że
+
{{Spoiler|Style = font-style: italic; font-weight: bold; color: olive; text-decoration: underline;|Show=Dowód|Hide=Ukryj dowód}}
 +
Korzystając ze wzoru udowodnionego w&nbsp;twierdzeniu D15 i&nbsp;przechodząc z <math>n</math> do nieskończoności, dostajemy
  
::<math>\mathbb{n} = a b \equiv (r s)^2 \pmod{m}</math>
+
::<math>\int_{m}^{\infty} f(x) d x \leqslant \sum_{k = m}^{\infty} f(k) \leqslant f(m) + \int_{m}^{\infty} f(x) d x</math>
  
Wbrew założeniu, że <math>\mathbb{n}</math> jest liczbą niekwadratową modulo <math>m .</math><br/>
+
Czyli
&#9633;
 
{{\Spoiler}}
 
 
 
 
 
 
 
<span style="font-size: 110%; font-weight: bold;">Zadanie K31</span><br/>
 
Niech <math>m \in \mathbb{Z}_+ \,</math> i <math>\, \mathbb{n} (m)</math> będzie najmniejszą liczbą niekwadratową modulo <math>m .</math> Pokazać, że jeżeli <math>m = 8 k \pm 3</math>, to <math>\mathbb{n} (m) = 2 .</math>
 
 
 
{{Spoiler|Style = font-style: italic; font-weight: bold; color: olive; text-decoration: underline;|Show=Rozwiązanie|Hide=Ukryj rozwiązanie}}
 
Z twierdzenia J41 wiemy, że <math>\left( {\small\frac{2}{m}} \right)_{\small{\!\! J}} = - 1</math>, gdy <math>m = 8 k \pm 3 .</math> Wynika stąd, że <math>2</math> jest liczbą niekwadratową modulo <math>m</math>, a&nbsp;jeśli tak, to musi być najmniejszą liczbą niekwadratową modulo <math>m .</math> Co należało pokazać.<br/>
 
&#9633;
 
{{\Spoiler}}
 
 
 
 
 
 
 
<span style="font-size: 110%; font-weight: bold;">Zadanie K32</span><br/>
 
Niech <math>m \in \mathbb{Z}_+ \,</math> i <math>\, \mathbb{n} (m)</math> będzie najmniejszą liczbą niekwadratową modulo <math>m .</math> Pokazać, że jeżeli spełniony jest jeden z&nbsp;warunków
 
 
 
:*&nbsp;&nbsp;<math>4 \mid m \;</math> i <math>\; \gcd (3, m) = 1</math>
 
:*&nbsp;&nbsp;<math>m = 12 k \pm 4</math>
 
 
 
to <math>\mathbb{n} (m) = 3 .</math>
 
 
 
{{Spoiler|Style = font-style: italic; font-weight: bold; color: olive; text-decoration: underline;|Show=Rozwiązanie|Hide=Ukryj rozwiązanie}}
 
Zauważmy, że <math>2</math> nie może być najmniejszą liczbą niekwadratową modulo <math>m</math>, bo <math>2 \mid m .</math> Rozważmy kongruencję
 
  
::<math>x^2 \equiv 3 \pmod{m}</math>
+
::<math>R(m) \leqslant \sum_{k = m}^{\infty} f(k) \leqslant f(m) + R (m)</math>
  
Z założenia <math>4 \mid m</math>, co nie wyklucza możliwości, że również <math>8 \mid m .</math> Ponieważ <math>4 \nmid (3 - 1)</math> i <math>8 \nmid (3 - 1)</math>, to z&nbsp;twierdzenia J55 wynika, że kongruencja <math>x^2 \equiv 3 \!\! \pmod{m}</math> nie ma rozwiązania. Jeśli tylko <math>3 \nmid m</math>, to <math>\mathbb{n} (m) = 3 .</math> W&nbsp;pierwszym punkcie jest to założone wprost, w&nbsp;drugim łatwo widzimy, że <math>3 \nmid (12 k \pm 4) .</math>
+
Odejmując od każdej ze stron nierówności liczbę <math>f(m)</math> i&nbsp;dodając do każdej ze stron nierówności sumę skończoną <math>S(m) = \sum_{k = a}^{m} f(k)</math>, otrzymujemy
  
Można też zauważyć, że żądanie, aby <math>\gcd (3, m) = 1</math>, prowadzi do dwóch układów kongruencji
+
::<math>S(m) + R (m) - f(m) \leqslant \sum_{k = a}^{\infty} f(k) \leqslant S(m) + R (m)</math>
 
 
::<math>\begin{align}
 
m &\equiv 0 \pmod{4} \\
 
m &\equiv 1 \pmod{3} \\
 
\end{align}</math>
 
 
 
oraz
 
 
 
::<math>\begin{align}
 
m &\equiv 0 \pmod{4} \\
 
m &\equiv 2 \pmod{3} \\
 
\end{align}</math>
 
 
 
którym, na mocy chińskiego twierdzenia o&nbsp;resztach, odpowiadają dwie kongruencje równoważne
 
 
 
::<math>m \equiv \pm 4 \pmod{12}</math>
 
  
 
Co należało pokazać.<br/>
 
Co należało pokazać.<br/>
Linia 1336: Linia 633:
  
  
<span style="font-size: 110%; font-weight: bold;">Zadanie K33</span><br/>
+
<span style="font-size: 110%; font-weight: bold;">Przykład D20</span><br/>
Niech <math>m = 24 k \pm 10 .</math> Pokazać, że <math>\mathbb{n} (m) = 3 .</math>
+
Twierdzenie D19 umożliwia określenie, z&nbsp;jaką dokładnością została wyznaczona suma szeregu. Wyznaczmy sumę szeregu <math>\sum_{k = 1}^{\infty} \frac{1}{(k + 1) \sqrt{k}}</math>. Mamy
  
{{Spoiler|Style = font-style: italic; font-weight: bold; color: olive; text-decoration: underline;|Show=Rozwiązanie|Hide=Ukryj rozwiązanie}}
+
::<math>S(m) = \sum_{k = 1}^{m} \frac{1}{(k + 1) \sqrt{k}}</math>
Zapiszmy <math>m</math> w&nbsp;postaci <math>m = 2 m'</math>, gdzie <math>m' = 12 k \pm 5 .</math> Gdyby kongruencja
 
  
::<math>x^2 \equiv 3 \pmod{2 m'}</math>
+
::<math>\int \frac{d x}{(x + 1) \sqrt{x}} = 2 \text{arctg} \left( \sqrt{x} \right)</math>
  
miała rozwiązanie, to również kongruencja
+
::<math>R(m) = \int_{m}^{\infty} \frac{d x}{(x + 1) \sqrt{x}} = \pi - 2 \text{arctg} \left( \sqrt{m} \right)</math>
  
::<math>x^2 \equiv 3 \pmod{m'}</math>
+
Zatem
 
 
miałaby rozwiązanie, ale jest to niemożliwe, bo <math>\left( {\small\frac{3}{m'}} \right)_{\small{\!\! J}} = - 1</math> (zobacz J46), czyli <math>3</math> jest liczbą niekwadratową modulo <math>m' .</math> Ponieważ <math>2 \mid m</math>, to <math>2</math> nie może być najmniejszą liczbą niekwadratową modulo <math>m .</math> Wynika stąd, że <math>\mathbb{n} (m) = 3 .</math><br/>
 
&#9633;
 
{{\Spoiler}}
 
  
 +
::<math>S(m) + R (m) - f (m) \leqslant \sum_{k = 1}^{\infty} \frac{1}{(k + 1) \sqrt{k}} \leqslant S (m) + R (m)</math>
  
 +
Dla kolejnych wartości <math>m</math> otrzymujemy
  
<span style="font-size: 110%; font-weight: bold;">Twierdzenie K34</span><br/>
+
::{| class="wikitable plainlinks"  style="font-size: 100%; text-align: center; margin-right: auto;"
Niech <math>m \in \mathbb{Z}_+ \;</math> i <math>\; S_2 = \{ 3, 5, 11, 13, 19, 29, 37, 43, \ldots \}</math> będzie zbiorem liczb pierwszych <math>p</math> takich, że <math>\left( {\small\frac{2}{p}} \right)_{\small{\!\! J}} = - 1 .</math> Jeżeli <math>m</math> jest liczbą nieparzystą podzielną przez <math>p \in S_2</math>, to <math>\mathbb{n} (m) = 2 .</math>
+
! <math>m</math>
 
+
! <math>S(m) + R(m) - f(m)</math>
{{Spoiler|Style = font-style: italic; font-weight: bold; color: olive; text-decoration: underline;|Show=Dowód|Hide=Ukryj dowód}}
+
! <math>S(m) + R(m)</math>
Z założenia <math>p \mid m \;</math> i <math>\; \left( {\small\frac{2}{p}} \right)_{\small{\!\! J}} = - 1 .</math> Zatem kongruencja
 
 
 
::<math>x^2 \equiv 2 \pmod{m}</math>
 
 
 
nie ma rozwiązania (zobacz J55). Ponieważ <math>2 \nmid m</math>, to <math>\mathbb{n} (m) = 2 .</math>
 
 
 
Uwaga: zbiór <math>S_2</math> tworzą liczby pierwsze postaci <math>8 k \pm 3</math> (zobacz J41).<br/>
 
&#9633;
 
{{\Spoiler}}
 
 
 
 
 
 
 
<span style="font-size: 110%; font-weight: bold;">Twierdzenie K35</span><br/>
 
Niech <math>m \in \mathbb{Z}_+ \;</math> i <math>\; S_3 = \{ 5, 7, 17, 19, 29, 31, 41, 43, \ldots \}</math> będzie zbiorem liczb pierwszych <math>p</math> takich, że <math>\left( {\small\frac{3}{p}} \right)_{\small{\!\! J}} = - 1 .</math> Jeżeli <math>m</math> jest liczbą parzystą niepodzielną przez <math>3</math> i&nbsp;podzielną przez <math>p \in S_3</math>, to <math>\mathbb{n} (m) = 3 .</math>
 
 
 
{{Spoiler|Style = font-style: italic; font-weight: bold; color: olive; text-decoration: underline;|Show=Dowód|Hide=Ukryj dowód}}
 
Z założenia <math>p \mid m \;</math> i <math>\; \left( {\small\frac{3}{p}} \right)_{\small{\!\! J}} = - 1 .</math> Zatem kongruencja
 
 
 
::<math>x^2 \equiv 3 \pmod{m}</math>
 
 
 
nie ma rozwiązania (zobacz J55). Ponieważ <math>2 \mid m</math> i <math>3 \nmid m</math>, to <math>\mathbb{n} (m) = 3 .</math>
 
 
 
Uwaga: zbiór <math>S_3</math> tworzą liczby pierwsze postaci <math>12 k \pm 5</math> (zobacz J46).<br/>
 
&#9633;
 
{{\Spoiler}}
 
 
 
 
 
 
 
<span style="font-size: 110%; font-weight: bold;">Twierdzenie K36</span><br/>
 
Jeżeli <math>m</math> jest liczbą dodatnią podzielną przez <math>6</math> i&nbsp;niepodzielną przez <math>5</math>, to <math>\mathbb{n} (m) = 5 .</math>
 
 
 
{{Spoiler|Style = font-style: italic; font-weight: bold; color: olive; text-decoration: underline;|Show=Dowód|Hide=Ukryj dowód}}
 
Z założenia <math>3 \mid m \;</math> i <math>\; \left( {\small\frac{5}{3}} \right)_{\small{\!\! J}} = \left( {\small\frac{2}{3}} \right)_{\small{\!\! J}} = - 1 .</math> Zatem kongruencja
 
 
 
::<math>x^2 \equiv 5 \pmod{m}</math>
 
 
 
nie ma rozwiązania (zobacz J55). Ponieważ <math>2 \mid m</math>, <math>3 \mid m</math> i <math>5 \nmid m</math>, to <math>\mathbb{n} (m) = 5 .</math><br/>
 
&#9633;
 
{{\Spoiler}}
 
 
 
 
 
 
 
<span style="font-size: 110%; font-weight: bold;">Twierdzenie K37</span><br/>
 
Niech <math>m \in \mathbb{Z}_+</math> i <math>p \geqslant 5</math> będzie liczbą pierwszą. Jeżeli iloczyn wszystkich liczb pierwszych mniejszych od <math>p</math> dzieli <math>m</math> i <math>p \nmid m</math>, to <math>\mathbb{n} (m) = p</math>.
 
 
 
{{Spoiler|Style = font-style: italic; font-weight: bold; color: olive; text-decoration: underline;|Show=Dowód|Hide=Ukryj dowód}}
 
Z twierdzenia K69 wiemy, że istnieje liczba pierwsza nieparzysta <math>q < p</math> taka, że <math>\left( {\small\frac{p}{q}} \right)_{\small{\!\! J}} = - 1 .</math> Z&nbsp;założenia <math>q \mid m</math>, zatem kongruencja
 
 
 
::<math>x^2 \equiv p \pmod{m}</math>
 
 
 
nie ma rozwiązania (zobacz J55). Ponieważ wszystkie liczby pierwsze mniejsze od <math>p</math> dzielą <math>m</math>, to <math>\mathbb{n} (m) = p</math>. Co należało pokazać.<br/>
 
&#9633;
 
{{\Spoiler}}
 
 
 
 
 
 
 
<span style="font-size: 110%; font-weight: bold;">Zadanie K38</span><br/>
 
Pokazać, że podanym w&nbsp;pierwszej kolumnie postaciom liczby <math>m</math> odpowiadają wymienione w&nbsp;drugiej kolumnie wartości <math>\mathbb{n} (m) .</math>
 
 
 
::{| class="wikitable plainlinks"  style="font-size: 90%; text-align: left; margin-right: auto;"
 
 
|-
 
|-
! Postać liczby <math>\boldsymbol{m}</math> || <math>\boldsymbol{𝕟(m)}</math> || Uwagi
+
| <math>10^1</math> || <math>1.84</math> || <math>1.87</math>
 
|-
 
|-
| <math>m=24k \pm 9</math> || style="text-align:center;" | <math>2</math> || rowspan="3" style="text-align:center;" | K34
+
| <math>10^2</math> || <math>1.85</math> || <math>1.86</math>
 
|-
 
|-
| <math>m=120k \pm 25</math> || style="text-align:center;" | <math>2</math>
+
| <math>10^3</math> || <math>1.86000</math> || <math>1.86004</math>
 
|-
 
|-
| <math>m=120k \pm 55</math> || style="text-align:center;" | <math>2</math>
+
| <math>10^4</math> || <math>1.860024</math> || <math>1.860025</math>
 
|-
 
|-
| <math>m=120k \pm 50</math> || style="text-align:center;" | <math>3</math> || style="text-align:center;" | K35
+
| <math>10^5</math> || <math>1.86002506</math> || <math>1.86002509</math>
 
|-
 
|-
| <math>m=30k \pm 6</math> || style="text-align:center;" | <math>5</math> || rowspan="2" style="text-align:center;" | K36,<math>\;</math>K37
+
| <math>10^6</math> || <math>1.860025078</math> || <math>1.860025079</math>
 
|-
 
|-
| <math>m=30k \pm 12</math> || style="text-align:center;" | <math>5</math>
+
| <math>10^7</math> || <math>1.86002507920</math> || <math>1.86002507923</math>
 
|-
 
|-
| <math>m=210k \pm 30</math> || style="text-align:center;" | <math>7</math> || rowspan="3" style="text-align:center;" | K37
+
| <math>10^8</math> || <math>1.860025079220</math> || <math>1.860025079221</math>
 
|-
 
|-
| <math>m=210k \pm 60</math> || style="text-align:center;" | <math>7</math>  
+
| <math>10^9</math> || <math>1.8600250792211</math> || <math>1.8600250792212</math>
 
|-
 
|-
| <math>m=210k \pm 90</math> || style="text-align:center;" | <math>7</math>
 
 
|}
 
|}
  
  
 +
W programie PARI/GP wystarczy napisać:
  
<span style="font-size: 110%; font-weight: bold;">Twierdzenie K39</span><br/>
+
f(k) = 1.0/(k+1)/sqrt(k)
Niech <math>m</math> będzie liczbą nieparzystą, a <math>\mathbb{n} (m)</math> będzie najmniejszą liczbą niekwadratową modulo <math>m .</math> Mamy
+
S(m) = sum( k = 1, m, f(k) )
 +
R(m) = Pi - 2*atan( sqrt(m) )
 +
for(j=1, 9, m=10^j; suma=S(m); reszta=R(m); print( "j= ", j, a= ", suma + reszta - f(m), "  b= ", suma + reszta ))
  
::<math>\begin{array}{lll}
 
  \mathbb{n} (2 m) >\mathbb{n} (m) &  & \text{gdy} \;\; \mathbb{n} (m) = 2 \\
 
  \mathbb{n} (2 m) =\mathbb{n} (m) &  & \text{gdy} \;\; \mathbb{n} (m) > 2 \\
 
\end{array}</math>
 
  
{{Spoiler|Style = font-style: italic; font-weight: bold; color: olive; text-decoration: underline;|Show=Dowód|Hide=Ukryj dowód}}
 
  
'''Punkt 1.'''
 
  
W przypadku, gdy <math>\mathbb{n} (m) = 2</math>, mamy <math>\mathbb{n} (2 m) > 2 = \mathbb{n} (m)</math>, bo <math>\mathbb{n} (2 m)</math> musi być liczbą względnie pierwszą z <math>2 m .</math>
+
Prostym wnioskiem z&nbsp;twierdzenia D15 jest następujące<br/>
 +
<span style="font-size: 110%; font-weight: bold;">Twierdzenie D21</span><br/>
 +
Niech <math>f(x)</math> będzie funkcją ciągłą, dodatnią i&nbsp;malejącą w&nbsp;przedziale <math>[m, + \infty)</math>. Jeżeli przy wyliczaniu sumy szeregu nieskończonego <math>\sum_{k = a}^{\infty} f (k)</math> (gdzie <math>a < m</math>) zastąpimy sumę <math>\sum_{k = m}^{\infty} f (k)</math> całką <math>\int_{m}^{\infty} f (x) d x</math>, to błąd wyznaczenia sumy szeregu nie przekroczy <math>f(m)</math>.
  
'''Punkt 2.'''
+
{{Spoiler|Style = font-style: italic; font-weight: bold; color: olive; text-decoration: underline;|Show=Dowód|Hide=Ukryj dowód}}
 +
Korzystając ze wzoru z&nbsp;twierdzenia D15 i&nbsp;przechodząc z <math>n</math> do nieskończoności, otrzymujemy
  
Z definicji najmniejszej liczby niekwadratowej modulo <math>m</math> wiemy, że kongruencja
+
::<math>\int_{m}^{\infty} f(x) d x \leqslant \sum_{k = m}^{\infty} f(k) \leqslant f(m) + \int_{m}^{\infty} f(x) d x</math>
  
::<math>x^2 \equiv \mathbb{n} (m) \pmod{m}</math>
+
Dodając do każdej ze stron nierówności wyrażenie <math>- f(m) + \sum_{k = a}^{m} f(k)</math>, dostajemy
  
nie ma rozwiązania. Oznacza to, że istnieje liczba pierwsza nieparzysta <math>p</math> taka, że <math>p \mid m \;</math> i <math>\; \left( {\small\frac{\mathbb{n} (m)}{p}} \right)_{\small{\!\! J}} = - 1 .</math> Ponieważ <math>p \mid 2 m</math>, to wynika stąd natychmiast, że kongruencja
+
::<math>- f(m) + \sum_{k = a}^{m} f(k) + \int_{m}^{\infty} f(x) d x \leqslant \sum_{k = a}^{\infty} f(k) \leqslant \sum_{k = a}^{m} f(k) + \int_{m}^{\infty} f(x) d x</math>
  
::<math>x^2 \equiv \mathbb{n} (m) \pmod{2 m}</math>
+
Skąd wynika natychmiast
  
również nie ma rozwiązania (zobacz J55).
+
::<math>- f(m) \leqslant \sum_{k = a}^{\infty} f(k) - \left( \sum_{k = a}^{m} f(k) + \int_{m}^{\infty} f(x) d x \right) \leqslant 0 < f(m)</math>
  
Zatem <math>\mathbb{n} (2 m) \leqslant \mathbb{n} (m) .</math> Niech <math>q</math> będzie liczbą pierwszą taką, że <math>2 < q <\mathbb{n} (m) .</math> Kongruencję
+
Czyli
  
::<math>x^2 \equiv q \pmod{2 m} \qquad \qquad (1)</math>
+
::<math>\left| \sum_{k = a}^{\infty} f(k) - \left( \sum_{k = a}^{m} f(k) + \int_{m}^{\infty} f(x) d x \right) \right| \leqslant f(m)</math>
  
możemy zapisać w&nbsp;postaci układu kongruencji równoważnych (zobacz J1)
+
Co kończy dowód.<br/>
 
 
::<math>\begin{align}
 
x^2 & \equiv q \pmod{m} \qquad \qquad \;\: (2) \\
 
x^2 & \equiv q \pmod{2} \qquad \qquad \;\;\,\, (3) \\
 
\end{align}</math>
 
 
 
Z definicji <math>q</math> jest liczbą kwadratową modulo <math>m</math>, zatem kongruencja <math>(2)</math> ma rozwiązanie – oznaczmy to rozwiązanie przez <math>x_0 .</math> Łatwo zauważamy, że liczba
 
 
 
::<math>x'_0 =
 
  \begin{cases}
 
  \;\;\;\; x_0 & \text{gdy} \quad x_0 \equiv 1 \pmod{2} \\
 
  x_0 + m & \text{gdy} \quad x_0 \equiv 0 \pmod{2} \\
 
  \end{cases}</math>
 
 
 
jest rozwiązaniem układu kongruencji <math>(2)</math> i <math>(3)</math>, a&nbsp;tym samym kongruencja <math>(1)</math> ma rozwiązanie dla każdego <math>2 < q <\mathbb{n} (m) .</math> Wynika stąd, że <math>\mathbb{n} (2 m) =\mathbb{n} (m) .</math><br/>
 
 
&#9633;
 
&#9633;
 
{{\Spoiler}}
 
{{\Spoiler}}
Linia 1493: Linia 711:
  
  
<span style="font-size: 110%; font-weight: bold;">Twierdzenie K40</span><br/>
+
<span style="font-size: 110%; font-weight: bold;">Twierdzenie D22</span><br/>
Niech <math>m</math> będzie liczbą nieparzystą, a <math>\mathbb{n} (m)</math> będzie najmniejszą liczbą niekwadratową modulo <math>m .</math> Mamy
+
Niech <math>f(x)</math> będzie funkcją ciągłą, dodatnią i&nbsp;malejącą w&nbsp;przedziale <math>[m, + \infty)</math>. Jeżeli szereg <math>\sum_{k = m}^{\infty} f (k)</math> jest zbieżny, to dla każdego <math>n \geqslant m</math> prawdziwe jest następujące oszacowanie sumy częściowej szeregu <math>S(n)</math>
  
::<math>\begin{array}{lllll}
+
::<math>S(n) = \sum_{k = m}^{n} f (k) \leqslant C - B \int_{n}^{\infty} f (x) d x</math>
  \mathbb{n} (4 m) \geqslant 5 & & \mathbb{n} (m) = 2        & & \text{gdy } \;\; 3 \mid m \\
 
  \mathbb{n} (4 m) = 3        & & \mathbb{n} (m) \geqslant 2 & & \text{gdy } \;\; 3 \nmid m \\
 
\end{array}</math>
 
  
{{Spoiler|Style = font-style: italic; font-weight: bold; color: olive; text-decoration: underline;|Show=Dowód|Hide=Ukryj dowód}}
+
gdzie <math>B</math> oraz <math>C</math> są dowolnymi stałymi spełniającymi nierówności
  
'''Punkt 1.'''
+
::<math>B \geqslant 1</math>
  
Z twierdzenia K34 wynika, że w&nbsp;przypadku, gdy <math>3 \mid m</math>, to <math>\mathbb{n} (m) = 2 .</math> Ponieważ <math>2 \mid 4 m</math> i <math>3 \mid 4 m</math>, to <math>\mathbb{n} (4 m) \geqslant 5 .</math>
+
::<math>C \geqslant f (m) + B \int_{m}^{\infty} f (x) d x</math>
  
'''Punkt 2.'''
+
{{Spoiler|Style = font-style: italic; font-weight: bold; color: olive; text-decoration: underline;|Show=Dowód|Hide=Ukryj dowód}}
 
+
Z twierdzenia D15 mamy
Ponieważ <math>m</math> jest liczbą nieparzystą, to <math>8 \nmid 4 m</math>, ale <math>4 \mid 4 m \;</math> i <math>\; 4 \nmid (3 - 1)</math>, zatem z&nbsp;twierdzenia J55 wynika, że kongruencja
 
 
 
::<math>x^2 \equiv 3 \pmod{4 m}</math>
 
 
 
nie ma rozwiązania. Ponieważ <math>2 \mid 4 m \;</math> i <math>\; 3 \nmid 4 m</math>, to <math>\mathbb{n} (4 m) = 3 .</math><br/>
 
&#9633;
 
{{\Spoiler}}
 
 
 
 
 
 
 
<span style="font-size: 110%; font-weight: bold;">Twierdzenie K41</span><br/>
 
Niech <math>p</math> będzie liczbą pierwszą nieparzystą. Jeżeli <math>a</math> jest liczbą niekwadratową modulo <math>p \,</math> i <math>\, p \mid m</math>, to <math>a</math> jest liczbą niekwadratową modulo <math>m .</math>
 
  
{{Spoiler|Style = font-style: italic; font-weight: bold; color: olive; text-decoration: underline;|Show=Dowód|Hide=Ukryj dowód}}
+
::<math>S(n) = \sum_{k = m}^{n} f (k) \leqslant f (m) + \int_{m}^{n} f (x) d x \leqslant</math>
Wiemy, że liczba <math>a</math> jest liczbą kwadratową modulo <math>m</math> wtedy i&nbsp;tylko wtedy, gdy kongruencja
 
  
::<math>x^2 \equiv a \pmod{m}</math>
+
:::::::<math>\;\! \leqslant f (m) + B \int_{m}^{n} f (x) d x =</math>
  
ma rozwiązanie. Przypuśćmy, że liczba <math>a</math> jest liczbą kwadratową modulo <math>m .</math> Zatem istnieje taka liczba <math>k \in \mathbb{Z}</math>, że
+
:::::::<math>\;\! = f (m) + B \int_{m}^{n} f (x) d x - B \int_{m}^{\infty} f (x) d x + B \int_{m}^{\infty} f (x) d x =</math>
  
::<math>k^2 \equiv a \pmod{m}</math>
+
:::::::<math>\;\! = f (m) + B \int_{m}^{n} f (x) d x - B \int^n_m f (x) d x - B \int_{n}^{\infty} f (x) d x + B \int_{m}^{\infty} f (x) d x =</math>
  
Ponieważ z&nbsp;założenia <math>p \mid m</math>, to prawdziwa jest też kongruencja
+
:::::::<math>\;\! = f (m) - B \int_{n}^{\infty} f (x) d x + B \int_{m}^{\infty} f (x) d x =</math>
  
::<math>k^2 \equiv a \pmod{p}</math>
+
:::::::<math>\;\! = \left[ f (m) + B \int_{m}^{\infty} f (x) d x \right] - B \int_{n}^{\infty} f (x) d x \leqslant</math>
  
co przeczy założeniu, że liczba <math>a</math> jest liczbą niekwadratową modulo <math>p .</math><br/>
+
:::::::<math>\;\! \leqslant C - B \int_{n}^{\infty} f (x) d x</math><br/>
 
&#9633;
 
&#9633;
 
{{\Spoiler}}
 
{{\Spoiler}}
Linia 1541: Linia 743:
  
  
<span style="font-size: 110%; font-weight: bold;">Twierdzenie K42</span><br/>
+
<span style="font-size: 110%; font-weight: bold;">Uwaga D23</span><br/>
Niech <math>m \geqslant 3</math> będzie liczbą nieparzystą. Jeżeli liczba <math>\mathbb{n} = \mathbb{n} (m)</math> jest najmniejszą liczbą niekwadratową modulo <math>m</math>, to istnieje taki dzielnik pierwszy <math>p</math> liczby <math>m</math>, że <math>\mathbb{n}</math> jest najmniejszą liczbą niekwadratową modulo <math>p .</math>
+
Niech <math>f(x)</math> będzie funkcją ciągłą, dodatnią i&nbsp;malejącą w&nbsp;przedziale <math>[m, \infty)</math>. Rozważmy szereg <math>\sum_{k = m}^{\infty} f (k)</math>. Zauważmy, że:
 
 
{{Spoiler|Style = font-style: italic; font-weight: bold; color: olive; text-decoration: underline;|Show=Dowód|Hide=Ukryj dowód}}
 
Przypuśćmy, że taki dzielnik pierwszy nie istnieje. Zatem mamy zbiór dzielników pierwszych liczby <math>m</math>: <math>\{ p_1, \ldots, p_s \}</math> i&nbsp;powiązany z&nbsp;dzielnikami pierwszymi <math>p_k</math> zbiór najmniejszych liczb niekwadratowych modulo <math>p_k</math>: <math>\{ \mathbb{n}_1, \ldots, \mathbb{n}_s \}</math>, z&nbsp;których każda jest liczbą niekwadratową modulo <math>m</math> (zobacz K41). Wynika stąd, że liczba <math>\mathbb{n} = \mathbb{n} (m)</math> musi być mniejsza od każdej z&nbsp;liczb <math>\mathbb{n}_k .</math>
 
 
 
Z definicji liczba <math>\mathbb{n} = \mathbb{n} (m)</math> jest liczbą niekwadratową modulo <math>m</math>, co oznacza, że kongruencja
 
 
 
::<math>x^2 \equiv \mathbb{n} \pmod{m}</math>
 
  
nie ma rozwiązania. Niech <math>m = p^{\alpha_1}_1 \cdot \ldots \cdot p^{\alpha_s}_s .</math> Zatem przynajmniej jedna z&nbsp;kongruencji
+
* korzystając z&nbsp;całkowego kryterium zbieżności, możemy łatwo zbadać, czy szereg <math>\sum_{k = m}^{\infty} f (k)</math> jest zbieżny
 +
* jeżeli szereg jest zbieżny, to ponownie wykorzystując całki (twierdzenie D22), możemy znaleźć oszacowanie sumy częściowej szeregu <math>S(n) = \sum_{k = m}^{n} f(k)</math>
  
::<math>x^2 \equiv \mathbb{n} \pmod{p^{\alpha_k}_k}</math>
+
Jednak dysponując już oszacowaniem sumy częściowej szeregu <math>S(n) = \sum_{k = m}^{n} f(k)</math>, możemy udowodnić jego poprawność przy pomocy indukcji matematycznej, a&nbsp;stąd łatwo pokazać zbieżność szeregu <math>\sum_{k = m}^{\infty} f(k)</math>. Zauważmy, że wybór większego <math>B</math> ułatwia dowód indukcyjny. Stałą <math>C</math> najlepiej zaokrąglić w&nbsp;górę do wygodnej dla nas wartości.
  
musi nie mieć rozwiązania (zobacz J11). Z&nbsp;twierdzenia J49 wiemy, że wtedy kongruencja
 
  
::<math>x^2 \equiv \mathbb{n} \pmod{p_k}</math>
+
Czasami potrzebujemy takiego uproszczenia problemu, aby udowodnić zbieżność szeregów bez odwoływania się do całek. Zauważmy, że Czytelnik nawet nie musi znać całek – wystarczy, że policzy je przy pomocy programów, które potrafią to robić (np. WolframAlpha). Kiedy już znajdziemy oszacowanie sumy częściowej szeregu, nie musimy wyjaśniać, w&nbsp;jaki sposób je znaleźliśmy – wystarczy udowodnić, że jest ono poprawne, a&nbsp;do tego wystarczy indukcja matematyczna.
  
również nie ma rozwiązania. Zatem <math>\mathbb{n}</math> jest liczbą niekwadratową modulo <math>p_k \,</math> i <math>\, \mathbb{n} < \mathbb{n}_k</math>, co przeczy definicji liczby <math>\mathbb{n}_k .</math><br/>
+
Zamieszczonej niżej zadania pokazują, jak wykorzystać w&nbsp;tym celu twierdzenie D22.
&#9633;
 
{{\Spoiler}}
 
  
  
  
<span style="font-size: 110%; font-weight: bold;">Twierdzenie K43</span><br/>
+
<span style="font-size: 110%; font-weight: bold;">Zadanie D24</span><br/>
Niech <math>m \geqslant 3</math> będzie liczbą nieparzystą. Jeżeli <math>m = p^{\alpha_1}_1 \cdot \ldots \cdot p^{\alpha_s}_s</math>, to
+
Korzystając z&nbsp;twierdzenia D22, znaleźć oszacowania sumy częściowej szeregów
  
::<math>\mathbb{n}(m) = \min ( \mathbb{n} (p_1), \ldots, \mathbb{n} (p_s) )</math>
+
::<math>\sum_{k = 1}^{\infty} \frac{1}{k^2} \qquad</math> oraz <math>\qquad \sum_{k = 2}^{\infty} \frac{1}{k (\log k)^2}</math>
  
gdzie <math>\mathbb{n}(m)</math> jest najmniejszą liczbą kwadratową modulo <math>m</math>, a <math>\mathbb{n}(p_k)</math> są najmniejszymi liczbami kwadratowymi modulo <math>p_k .</math>
+
{{Spoiler|Style = font-style: italic; font-weight: bold; color: olive; text-decoration: underline;|Show=Rozwiązanie|Hide=Ukryj rozwiązanie}}
 
+
Rozważmy szereg <math>\sum_{k = 1}^{\infty} \frac{1}{k^2}</math>. Funkcja <math>f(x) = \frac{1}{x^2}</math> jest funkcją ciągłą, dodatnią i&nbsp;malejącą w&nbsp;przedziale <math>(0, + \infty)</math>. Dla <math>n > 0</math> jest
{{Spoiler|Style = font-style: italic; font-weight: bold; color: olive; text-decoration: underline;|Show=Dowód|Hide=Ukryj dowód}}
 
Twierdzenie to jest prostym wnioskiem z&nbsp;twierdzenia K42, ale musimy jeszcze pokazać, że <math>\gcd (\mathbb{n} (m), m) = 1 .</math> Przypuśćmy, że <math>p_k |\mathbb{n} (m)</math> dla pewnego <math>1 \leqslant k \leqslant s .</math> Ponieważ <math>\mathbb{n} (m)</math> jest liczbą pierwszą, to musi być <math>\mathbb{n} (m) = p_k</math>, ale wtedy
 
  
::<math>\mathbb{n} (p_k) < p_k =\mathbb{n} (m) \leqslant \mathbb{n} (p_k)</math>
+
::<math>\int_{n}^{\infty} \frac{1}{x^2} d x = \frac{1}{n} \qquad</math> (zobacz: [https://www.wolframalpha.com/input/?i=int+1%2Fx%5E2%2C+x%3Dn%2C+infinity WolframAlpha])
  
Otrzymana sprzeczność dowodzi, że <math>\mathbb{n} (m)</math> jest względnie pierwsza z&nbsp;każdą z&nbsp;liczb pierwszych <math>p_i</math>, gdzie <math>1 \leqslant i \leqslant s .</math> Co kończy dowód.<br/>
+
::<math>C \geqslant 1 + \int_{1}^{\infty} \frac{1}{x^2} d x = 2</math>
&#9633;
 
{{\Spoiler}}
 
  
 +
Zatem
  
 +
::<math>\sum_{k = 1}^{n} \frac{1}{k^2} \leqslant 2 - \frac{1}{n}</math>
  
<span style="font-size: 110%; font-weight: bold;">Twierdzenie K44</span><br/>
 
Niech <math>m \geqslant 3</math> będzie liczbą nieparzystą, a <math>\mathbb{n}(m)</math> jest najmniejszą liczbą niekwadratową modulo <math>m .</math> Prawdziwe są oszacowania
 
  
::<math>\mathbb{n}(m) < \sqrt{m} + {\small\frac{1}{2}} \qquad \qquad \qquad \;\;\, \text{dla } m \geqslant 3</math>
+
Rozważmy szereg <math>\sum_{k = 2}^{\infty} \frac{1}{k (\log k)^2}</math>. Funkcja <math>f(x) = \frac{1}{x (\log x)^2}</math> jest funkcją ciągłą, dodatnią i&nbsp;malejącą w&nbsp;przedziale <math>(1, + \infty)</math>. Dla <math>n > 1</math> jest
  
::<math>\mathbb{n}(m) \leqslant 1.1 \cdot m^{1 / 4} \log m \qquad \qquad \text{dla } m \geqslant 5</math>
+
::<math>\int_{n}^{\infty} \frac{1}{x (\log x)^2} d x = \frac{1}{\log n} \qquad</math> (zobacz: [https://www.wolframalpha.com/input/?i=int+1%2F%28x*%28log%28x%29%29%5E2%29%2C+x%3Dn%2C+infinity WolframAlpha])
  
{{Spoiler|Style = font-style: italic; font-weight: bold; color: olive; text-decoration: underline;|Show=Dowód|Hide=Ukryj dowód}}
+
::<math>C \geqslant \frac{1}{2 \cdot (\log 2)^2} + \int_{2}^{\infty} \frac{1}{x (\log x)^2} d x = \frac{1}{2 \cdot (\log 2)^2} + \frac{1}{\log 2} = 2.483379 \ldots</math>
Niech <math>p</math> będzie dzielnikiem pierwszym liczby <math>m</math> takim, że <math>\mathbb{n}(m) = \mathbb{n} (p)</math> (z twierdzenia K42 wiemy, że taki dzielnik istnieje). Jeżeli prawdziwe jest oszacowanie <math>\mathbb{n}(p) < F (p)</math>, gdzie <math>F(x)</math> jest funkcją rosnącą, to
 
  
::<math>\mathbb{n}(m) = \mathbb{n} (p) < F (p) \leqslant F (m)</math>
+
Przyjmijmy <math>C = 2.5</math>, zatem
  
Podane w&nbsp;twierdzeniu oszacowania wynikają natychmiast z&nbsp;twierdzeń K21 i&nbsp;K22.<br/>
+
::<math>\sum_{k = 2}^{n} \frac{1}{k (\log k)^2} < 2.5 - \frac{1}{\log n}</math><br/>
 
&#9633;
 
&#9633;
 
{{\Spoiler}}
 
{{\Spoiler}}
Linia 1601: Linia 789:
  
  
<span style="font-size: 110%; font-weight: bold;">Uwaga K45</span><br/>
+
<span style="font-size: 110%; font-weight: bold;">Zadanie D25</span><br/>
Liczby <math>\mathbb{n} (m)</math> są zaskakująco małe. Średnia wartość <math>\mathbb{n} = \mathbb{n} (m)</math> wynosi<ref name="Pollack1"/>
+
Stosując indukcję matematyczną, udowodnić prawdziwość oszacowania <math>\sum_{k = 1}^{n} \frac{1}{k^2} \leqslant 2 - \frac{1}{n}</math> i&nbsp;udowodnić, że szereg <math>\sum_{k = 1}^{\infty} \frac{1}{k^2}</math> jest zbieżny.
  
::<math>\lim_{x \to \infty} {\small\frac{1}{x}} \sum_{m \leqslant x} \mathbb{n} (m) = 2 + \sum_{k = 3}^{\infty} {\small\frac{p_k - 1}{p_1 \cdot \ldots \cdot p_{k - 1}}} = 2.920050977 \ldots</math>
+
{{Spoiler|Style = font-style: italic; font-weight: bold; color: olive; text-decoration: underline;|Show=Rozwiązanie|Hide=Ukryj rozwiązanie}}
 +
Indukcja matematyczna. Łatwo zauważamy, że oszacowanie jest prawdziwe dla <math>n = 1</math>. Zakładając, że oszacowanie jest prawdziwe dla <math>n</math>, otrzymujemy dla <math>n + 1</math>
  
 +
::<math>\sum_{k = 1}^{n + 1} \frac{1}{k^2} = \sum_{k = 1}^{n} \frac{1}{k^2} + \frac{1}{(n + 1)^2} \leqslant</math>
  
 +
::::<math>\;\: \leqslant 2 - \frac{1}{n} + \frac{1}{(n + 1)^2} \leqslant</math>
  
 +
::::<math>\;\: \leqslant 2 - \frac{1}{n + 1} + \left( \frac{1}{n + 1} - \frac{1}{n} + \frac{1}{(n + 1)^2} \right) =</math>
  
 +
::::<math>\;\: = 2 - \frac{1}{n + 1} - \frac{1}{n (n + 1)^2} <</math>
  
{| style="border-spacing: 5px; border: 2px solid black; background: transparent;"
+
::::<math>\;\: < 2 - \frac{1}{n + 1}</math>
| &nbsp;'''C.''' Najmniejsze dodatnie liczby niekwadratowe <math>a</math> takie, że <math>\left( {\small\frac{a}{m}} \right)_{\small{\!\! J}} = - 1</math>&nbsp;
 
|}
 
  
<span style="font-size: 110%; font-weight: bold;">Przykład K46</span><br/>
+
Co kończy dowód indukcyjny. Zatem dla <math>n \geqslant 1</math> mamy
W tabeli przedstawiliśmy najmniejsze liczby niekwadratowe modulo <math>p</math>, najmniejsze liczby niekwadratowe modulo <math>m</math> i&nbsp;najmniejsze dodatnie liczby niekwadratowe <math>a</math> takie, że <math>\left( {\small\frac{a}{m}} \right)_{\small{\!\! J}} = - 1</math>.
 
  
::{| class="wikitable plainlinks"  style="font-size: 100%; text-align: center; margin-right: auto;"
+
::<math>S(n) = \sum_{k = 1}^{n} \frac{1}{k^2} \leqslant 2 - \frac{1}{n} < 2</math>
! <math>\boldsymbol{m}</math>
 
| <math>3</math> || <math>5</math> || <math>7</math> || <math>9</math> || <math>11</math> || <math>13</math> || <math>15</math> || <math>17</math> || <math>19</math> || <math>21</math> || <math>23</math> || <math>25</math> || <math>27</math> || <math>29</math> || <math>31</math> || <math>33</math> || <math>35</math> || <math>37</math> || <math>39</math> || <math>41</math> || <math>43</math> || <math>45</math> || <math>47</math> || <math>49</math> || <math>51</math>
 
|-
 
!  <math>\boldsymbol{\mathbb{n}( p )}</math>
 
| <math>2</math> || <math>2</math> || <math>3</math> || <math>-</math> || <math>2</math> || <math>2</math> || <math>-</math> || <math>3</math> || <math>2</math> || <math>-</math> || <math>5</math> || <math>-</math> || <math>-</math> || <math>2</math> || <math>3</math> || <math>-</math> || <math>-</math> || <math>2</math> || <math>-</math> || <math>3</math> || <math>2</math> || <math>-</math> || <math>5</math> || <math>-</math> || <math>-</math>
 
|-
 
!  <math>\boldsymbol{\mathbb{n}( m )}</math>
 
| <math>2</math> || <math>2</math> || <math>3</math> || <math>2</math> || <math>2</math> || <math>2</math> || <math>2</math> || <math>3</math> || <math>2</math> || <math>2</math> || <math>5</math> || <math>2</math> || <math>2</math> || <math>2</math> || <math>3</math> || <math>2</math> || <math>2</math> || <math>2</math> || <math>2</math> || <math>3</math> || <math>2</math> || <math>2</math> || <math>5</math> || <math>3</math> || <math>2</math>
 
|-
 
!  <math>\boldsymbol{c( m )}</math>
 
| <math>2</math> || <math>2</math> || <math>3</math> || <math>-</math> || <math>2</math> || <math>2</math> || <math>7</math> || <math>3</math> || <math>2</math> || <math>2</math> || <math>5</math> || <math>-</math> || <math>2</math> || <math>2</math> || <math>3</math> || <math>5</math> || <math>2</math> || <math>2</math> || <math>7</math> || <math>3</math> || <math>2</math> || <math>2</math> || <math>5</math> || <math>-</math> || <math>2</math>
 
|}
 
  
 +
Czyli ciąg sum częściowych <math>S(n) = \sum_{k = 1}^{n} \frac{1}{k^2}</math> szeregu <math>\sum_{k = 1}^{\infty} \frac{1}{k^2}</math> jest rosnący i&nbsp;ograniczony od góry, a&nbsp;zatem zbieżny. Co oznacza, że szereg jest zbieżny.<br/>
 +
&#9633;
 +
{{\Spoiler}}
  
  
<span style="font-size: 110%; font-weight: bold;">Uwaga K47</span><br/>
 
Do wyszukiwania liczb <math>c = c (m)</math> Czytelnik może wykorzystać prostą funkcję napisaną w&nbsp;PARI/GP
 
  
<span style="font-size: 90%; color:black;">C(m) =
+
<span style="font-size: 110%; font-weight: bold;">Zadanie D26</span><br/>
{
+
Stosując indukcję matematyczną, udowodnić prawdziwość oszacowania <math>\sum_{k = 2}^{n} \frac{1}{k (\log k)^2} < 2.5 - \frac{1}{\log n}</math> i&nbsp;udowodnić, że szereg <math>\sum_{k = 2}^{\infty} \frac{1}{k (\log k)^2}</math> jest zbieżny.
'''if'''( m%2 == 0, '''return'''(0) );
 
'''if'''( '''issquare'''(m), '''return'''(0) );
 
'''forprime'''(p = 2, m, '''if'''( jacobi(p, m) == -1, '''return'''(p) ));
 
}</span>
 
  
 +
{{Spoiler|Style = font-style: italic; font-weight: bold; color: olive; text-decoration: underline;|Show=Rozwiązanie|Hide=Ukryj rozwiązanie}}
 +
Indukcja matematyczna. Łatwo sprawdzamy, że oszacowanie jest prawdziwe dla <math>n = 2</math>
  
 +
::<math>\sum_{k = 2}^{2} \frac{1}{k (\log k)^2} \approx 1.040684 < 2.5 - \frac{1}{\log 2} \approx 1.05730</math>
  
<span style="font-size: 110%; font-weight: bold;">Uwaga K48</span><br/>
+
Zakładając, że oszacowanie jest prawdziwe dla <math>n</math>, otrzymujemy dla <math>n + 1</math>
Najmniejsze dodatnie liczby niekwadratowe <math>a</math> takie, że <math>\left( {\small\frac{a}{m}} \right)_{\small{\!\! J}} = - 1</math> oznaczyliśmy jako <math>c(m)</math>. Zauważmy, że są to liczby inne od <math>\mathbb{n}(p)</math> i <math>\mathbb{n}(m)</math>. Wystarczy zwrócić uwagę na występujące w&nbsp;tabeli liczby <math>\mathbb{n}(p)</math>, <math>\mathbb{n}(m)</math> i <math>c(m)</math> dla <math>m = 15, 33, 39</math>. Różnice wynikają z&nbsp;innej definicji liczb <math>c(m)</math> – jeżeli liczba <math>a</math> jest liczbą niekwadratową modulo <math>m</math>, to symbol Jacobiego <math>\left( {\small\frac{a}{m}} \right)_{\small{\!\! J}}</math> nie musi być równy <math>- 1</math>. I&nbsp;tak czasami bywa, co bardzo dobrze pokazuje powyższa tabela.
 
  
Ponieważ <math>c(m)</math> nie zawsze będzie najmniejszą liczbą kwadratową modulo <math>m</math>, to mamy natychmiast oszacowanie: <math>c(m) \geqslant \mathbb{n} (m)</math> (poza przypadkami, gdy <math>m = n^2</math>).
+
::<math>\sum_{k = m}^{n + 1} \frac{1}{k (\log k)^2} = \sum_{k = m}^{n} \frac{1}{k (\log k)^2} + \frac{1}{(n + 1) \cdot (\log (n + 1))^2} <</math>
  
Dla <math>c(m)</math> nie są prawdziwe oszacowania podane w&nbsp;twierdzeniu K21. Łatwo zauważamy, że
+
::::::<math>\;\: < 2.5 - \frac{1}{\log n} + \frac{1}{(n + 1) \cdot (\log (n + 1))^2} =</math>
  
::<math>c = c (15) = 7 > \sqrt{15} + {\small\frac{1}{2}} \approx 4.37</math>
+
::::::<math>\;\: = 2.5 - \frac{1}{\log (n + 1)} + \left( \frac{1}{\log (n + 1)} - \frac{1}{\log n} + \frac{1}{(n + 1) \cdot (\log (n + 1))^2} \right) =</math>
  
::<math>c = c (39) = 7 > \sqrt{39} + {\small\frac{1}{2}} \approx 6.74</math>
+
::::::<math>\;\: = 2.5 - \frac{1}{\log (n + 1)} + \frac{1}{\log (n + 1)} \left( 1 - \frac{\log (n + 1)}{\log n} + \frac{1}{(n + 1) \cdot \log (n + 1)} \right) =</math>
  
::<math>c = c (105) = 11 > \sqrt{105} + {\small\frac{1}{2}} \approx 10.75</math>
+
::::::<math>\;\: = 2.5 - \frac{1}{\log (n + 1)} + \frac{1}{\log (n + 1)} \left( 1 - \frac{\log \left( n \left( 1 + \frac{1}{n} \right) \right)}{\log n} + \frac{1}{(n + 1) \cdot \log (n + 1)} \right) =</math>
  
::<math>c = c (231) = 17 > \sqrt{231} + {\small\frac{1}{2}} \approx 15.7</math>
+
::::::<math>\;\: = 2.5 - \frac{1}{\log (n + 1)} + \frac{1}{\log (n + 1)} \left( 1 - 1 - \frac{\log \left( 1 + \frac{1}{n} \right)}{\log n} + \frac{1}{(n + 1) \cdot \log (n + 1)} \right) <</math>
  
Nie ma więcej takich przypadków dla <math>m < 10^9</math>.
+
::::::<math>\;\: < 2.5 - \frac{1}{\log (n + 1)} + \frac{1}{\log (n + 1)} \left( - \frac{1}{(n + 1) \log n} + \frac{1}{(n + 1) \cdot \log (n + 1)} \right) <</math>
  
 +
::::::<math>\;\: < 2.5 - \frac{1}{\log (n + 1)}</math>
  
 +
Co kończy dowód indukcyjny. Zatem dla <math>n \geqslant 2</math> mamy
  
<span style="font-size: 110%; font-weight: bold;">Twierdzenie K49</span><br/>
+
::<math>S(n) = \sum_{k = 2}^{n} \frac{1}{k (\log k)^2} < 2.5 - \frac{1}{\log n} < 2.5</math>
Niech <math>c, m \in \mathbb{Z}_+</math> i&nbsp;niech <math>m \geqslant 3</math> będzie liczbą nieparzystą, a <math>c</math> będzie najmniejszą liczbą taką, że <math>\left( {\small\frac{c}{m}} \right)_{\small{\!\! J}} = - 1</math>. Liczba <math>c</math> musi być liczbą pierwszą.
 
  
{{Spoiler|Style = font-style: italic; font-weight: bold; color: olive; text-decoration: underline;|Show=Dowód|Hide=Ukryj dowód}}
+
Czyli ciąg sum częściowych <math>S(n)</math> szeregu <math>\sum_{k = 2}^{\infty} \frac{1}{k (\log k)^2}</math> jest rosnący i&nbsp;ograniczony od góry, a&nbsp;zatem zbieżny. Co oznacza, że szereg jest zbieżny.<br/>
Przypuśćmy, że <math>c = a b</math> jest liczbą złożoną, gdzie <math>1 < a, b < c</math>. Mamy
 
 
 
::<math>- 1 = \left( {\small\frac{c}{m}} \right)_{\small{\!\! J}} = \left( {\small\frac{a b}{m}} \right)_{\small{\!\! J}} = \left( {\small\frac{a}{m}} \right)_{\small{\!\! J}}</math><math>\left( {\small\frac{b}{m}} \right)_{\small{\!\! J}}</math>
 
 
 
Zatem jeden z&nbsp;czynników po prawej stronie musi być równy <math>- 1</math> wbrew definicji liczby <math>c</math>.<br/>
 
 
&#9633;
 
&#9633;
 
{{\Spoiler}}
 
{{\Spoiler}}
Linia 1680: Linia 853:
  
  
== Liczby pierwsze postaci <math>x^2 + n y^2</math> ==
+
== Szeregi nieskończone i&nbsp;liczby pierwsze ==
  
<span style="font-size: 110%; font-weight: bold;">Przykład K50</span><br/>
+
<span style="font-size: 110%; font-weight: bold;">Twierdzenie D27</span><br/>
Przedstawiamy wszystkie rozkłady liczb naturalnych nie większych od <math>85</math> na sumę postaci <math>x^2 + y^2</math>, gdzie <math>x, y \in \mathbb{N}_0</math>. Rozkłady różniące się jedynie kolejnością liczb <math>x , y</math> nie zostały uwzględnione.
+
Następujące szeregi są zbieżne
  
{| class="wikitable plainlinks"  style="font-size: 70%; text-align: center; margin-right: auto;"
+
::{| class="wikitable plainlinks"  style="font-size: 100%; text-align: left; margin-right: auto;"
 
|-
 
|-
! <math>\boldsymbol{n}</math>
+
| 1. <math>\quad \sum_{k = 1}^{\infty} \frac{(- 1)^{k + 1}}{p_k} = 0.269605966 \ldots</math>
| <math>1</math> || style="background-color: #99cc66" | <math>2</math> || <math>4</math> || style="background-color: #99cc66" | <math>5</math> || <math>8</math> || <math>9</math> || <math>10</math> || style="background-color: #99cc66" | <math>13</math> || <math>16</math> || style="background-color: #99cc66" | <math>17</math> || <math>18</math> || <math>20</math> || <math>25</math> || <math>26</math> || style="background-color: #99cc66" | <math>29</math> || <math>32</math> || <math>34</math> || <math>36</math> || style="background-color: #99cc66" | <math>37</math> || <math>40</math> || style="background-color: #99cc66" | <math>41</math> || <math>45</math> || <math>49</math> || <math>50</math> || <math>52</math> || style="background-color: #99cc66" | <math>53</math> || <math>58</math> ||style="background-color: #99cc66" | <math>61</math> || <math>64</math> || <math>65</math> || <math>68</math> || <math>72</math> || style="background-color: #99cc66" | <math>73</math> || <math>74</math> || <math>80</math> || <math>81</math> || <math>82</math> || <math>85</math>
+
|  
 
|-
 
|-
! <math>\boldsymbol{x,y}</math>
+
| 2. <math>\quad \sum_{p \geqslant 2} \frac{1}{p^2} = 0.452247420041 \ldots</math>
| <math>1,0</math> || <math>1,1</math> || <math>2,0</math> || <math>2,1</math> || <math>2,2</math> || <math>3,0</math> || <math>3,1</math> || <math>3,2</math> || <math>4,0</math> || <math>4,1</math> || <math>3,3</math> || <math>4,2</math> || <math>5,0</math> || <math>5,1</math> || <math>5,2</math> || <math>4,4</math> || <math>5,3</math> || <math>6,0</math> || <math>6,1</math> || <math>6,2</math> || <math>5,4</math> || <math>6,3</math> || <math>7,0</math> || <math>7,1</math> || <math>6,4</math> || <math>7,2</math> || <math>7,3</math> || <math>6,5</math> || <math>8,0</math> || <math>8,1</math> || <math>8,2</math> || <math>6,6</math> || <math>8,3</math> || <math>7,5</math> || <math>8,4</math> || <math>9,0</math> || <math>9,1</math> || <math>9,2</math>
+
| [https://oeis.org/A085548 A085548]
 
|-
 
|-
! <math>\boldsymbol{x,y}</math>  
+
| 3. <math>\quad \sum_{p \geqslant 2} \frac{1}{(p - 1)^2} = 1.375064994748 \ldots</math>
| <math></math> || <math></math> || <math></math> || <math></math> || <math></math> || <math></math> || <math></math> || <math></math> || <math></math> || <math></math> || <math></math> || <math></math> || <math>4,3</math> || <math></math> || <math></math> || <math></math> || <math></math> || <math></math> || <math></math> || <math></math> || <math></math> || <math></math> || <math></math> || <math>5,5</math> || <math></math> || <math></math> || <math></math> || <math></math> || <math></math> || <math>7,4</math> || <math></math> || <math></math> || <math></math> || <math></math> || <math></math> || <math></math> || <math></math> || <math>7,6</math>
+
| [https://oeis.org/A086242 A086242]
 +
|-
 +
| 4. <math>\quad \sum_{p \geqslant 2} \frac{1}{p (p - 1)} = 0.773156669049 \ldots</math>
 +
| [https://oeis.org/A136141 A136141]
 
|}
 
|}
  
Zauważmy, że liczba złożona <math>21</math> nie ma rozkładu na sumę kwadratów, a&nbsp;liczba złożona <math>65</math> ma dwa takie rozkłady. Obie liczby są postaci <math>4 k + 1</math>.
+
{{Spoiler|Style = font-style: italic; font-weight: bold; color: olive; text-decoration: underline;|Show=Dowód|Hide=Ukryj dowód}}
 +
'''Punkt 1.'''<br/>
 +
Szereg jest szeregiem naprzemiennym i&nbsp;jego zbieżność wynika z&nbsp;twierdzenia D5.
  
 +
'''Punkt 2.'''<br/>
 +
Szereg jest zbieżny, bo sumy częściowe tego szeregu tworzą ciąg rosnący i&nbsp;ograniczony
  
 +
::<math>\sum_{p \leqslant n} \frac{1}{p^2} < \sum_{k = 2}^{\infty} \frac{1}{k^2} < \frac{\pi^2}{6}</math>
  
<span style="font-size: 110%; font-weight: bold;">Przykład K51</span><br/>
+
'''Punkt 3.'''<br/>
Przedstawiamy wszystkie rozkłady liczb naturalnych nie większych od <math>73</math> na sumę postaci <math>x^2 + 2 y^2</math>, gdzie <math>x, y \in \mathbb{N}_0</math>.
+
Szereg jest zbieżny, bo sumy częściowe tego szeregu tworzą ciąg rosnący i&nbsp;ograniczony
  
{| class="wikitable plainlinks"  style="font-size: 70%; text-align: center; margin-right: auto;"
+
::<math>\sum_{p \leqslant n} \frac{1}{(p - 1)^2} < \sum_{j = 2}^{\infty} \frac{1}{(j - 1)^2} = \sum_{k = 1}^{\infty} \frac{1}{k^2} = \frac{\pi^2}{6}</math>
|-
 
! <math>\boldsymbol{n}</math>
 
| <math>1</math> || style="background-color: #99cc66" | <math>2</math> || style="background-color: #99cc66" | <math>3</math> || <math>4</math> || <math>6</math> || <math>8</math> || <math>9</math> || style="background-color: #99cc66" | <math>11</math> || <math>12</math> || <math>16</math> || style="background-color: #99cc66" | <math>17</math> || <math>18</math> || style="background-color: #99cc66" | <math>19</math> || <math>22</math> || <math>24</math> || <math>25</math> || <math>27</math> || <math>32</math> || <math>33</math> || <math>34</math> || <math>36</math> || <math>38</math> || style="background-color: #99cc66" | <math>41</math> || style="background-color: #99cc66" | <math>43</math> || <math>44</math> || <math>48</math> || <math>49</math> || <math>50</math> || <math>51</math> || <math>54</math> || <math>57</math> || style="background-color: #99cc66" | <math>59</math> || <math>64</math> || <math>66</math> || style="background-color: #99cc66" | <math>67</math> || <math>68</math> || <math>72</math> || style="background-color: #99cc66" | <math>73</math>
 
|-
 
! <math>\boldsymbol{x,y}</math>
 
| <math>1,0</math> || <math>0,1</math> || <math>1,1</math> || <math>2,0</math> || <math>2,1</math> || <math>0,2</math> || <math>3,0</math> || <math>3,1</math> || <math>2,2</math> || <math>4,0</math> || <math>3,2</math> || <math>4,1</math> || <math>1,3</math> || <math>2,3</math> || <math>4,2</math> || <math>5,0</math> || <math>5,1</math> || <math>0,4</math> || <math>5,2</math> || <math>4,3</math> || <math>6,0</math> || <math>6,1</math> || <math>3,4</math> || <math>5,3</math> || <math>6,2</math> || <math>4,4</math> || <math>7,0</math> || <math>0,5</math> || <math>7,1</math> || <math>6,3</math> || <math>7,2</math> || <math>3,5</math> || <math>8,0</math> || <math>8,1</math> || <math>7,3</math> || <math>6,4</math> || <math>8,2</math> || <math>1,6</math>
 
|-
 
! <math>\boldsymbol{x,y}</math>
 
| <math></math> || <math></math> || <math></math> || <math></math> || <math></math> || <math></math> || <math>1,2</math> || <math></math> || <math></math> || <math></math> || <math></math> || <math>0,3</math> || <math></math> || <math></math> || <math></math> || <math></math> || <math>3,3</math> || <math></math> || <math>1,4</math> || <math></math> || <math>2,4</math> || <math></math> || <math></math> || <math></math> || <math></math> || <math></math> || <math></math> || <math></math> || <math>1,5</math> || <math>2,5</math> || <math>5,4</math> || <math></math> || <math></math> || <math>4,5</math> || <math></math> || <math></math> || <math>0,6</math> || <math></math>
 
|}
 
  
Zauważmy, że liczba złożona <math>65</math> nie ma rozkładu na sumę postaci <math>x^2 + 2 y^2</math>, a&nbsp;liczba złożona <math>33</math> ma dwa takie rozkłady. Obie liczby są postaci <math>8 k + 1</math>.
+
'''Punkt 4.'''<br/>
 +
Zbieżność wzoru wynika z&nbsp;kryterium porównawczego, bo dla każdego <math>p \geqslant 2</math> jest
  
Zauważmy też, że liczba złożona <math>35</math> nie ma rozkładu na sumę postaci <math>x^2 + 2 y^2</math>, a&nbsp;liczba złożona <math>27</math> ma dwa takie rozkłady. Obie liczby są postaci <math>8 k + 3</math>.
+
::<math>0 < \frac{1}{p (p - 1)} < \frac{1}{(p - 1)^2}</math><br/>
 +
&#9633;
 +
{{\Spoiler}}
  
  
  
<span style="font-size: 110%; font-weight: bold;">Przykład K52</span><br/>
+
<span style="font-size: 110%; font-weight: bold;">Twierdzenie D28</span><br/>
Przedstawiamy wszystkie rozkłady liczb naturalnych nie większych od <math>103</math> na sumę postaci <math>x^2 + 3 y^2</math>, gdzie <math>x, y \in \mathbb{N}_0</math>.
+
Następujące szeregi są zbieżne
  
{| class="wikitable plainlinks"  style="font-size: 70%; text-align: center; margin-right: auto;"
+
::{| class="wikitable plainlinks"  style="font-size: 100%; text-align: left; margin-right: auto;"
 
|-
 
|-
! <math>\boldsymbol{n}</math>
+
| 1. <math>\quad \sum_{p \geqslant 2} \frac{1}{p \log p} = 1.636616323351 \ldots</math>
| <math>1</math> || style="background-color: #99cc66" | <math>3</math> || <math>4</math> || style="background-color: #99cc66" | <math>7</math> || <math>9</math> || <math>12</math> || style="background-color: #99cc66" | <math>13</math> || <math>16</math> || style="background-color: #99cc66" | <math>19</math> || <math>21</math> || <math>25</math> || <math>27</math> || <math>28</math> || style="background-color: #99cc66" | <math>31</math> || <math>36</math> || style="background-color: #99cc66" | <math>37</math> || <math>39</math> || style="background-color: #99cc66" | <math>43</math> || <math>48</math> || <math>49</math> || <math>52</math> || <math>57</math> || style="background-color: #99cc66" | <math>61</math> || <math>63</math> || <math>64</math> || style="background-color: #99cc66" | <math>67</math> || style="background-color: #99cc66" | <math>73</math> || <math>75</math> || <math>76</math> || style="background-color: #99cc66" | <math>79</math> || <math>81</math> || <math>84</math> || <math>91</math> || <math>93</math> || style="background-color: #99cc66" | <math>97</math> || <math>100</math> || style="background-color: #99cc66" | <math>103</math>
+
| [https://oeis.org/A137245 A137245]
 
|-
 
|-
! <math>\boldsymbol{x,y}</math>
+
| 2. <math>\quad \sum_{p \geqslant 2} \frac{1}{p^2 \log p} = 0.507782187859 \ldots</math>
| <math>1,0</math> || <math>0,1</math> || <math>2,0</math> || <math>2,1</math> || <math>3,0</math> || <math>3,1</math> || <math>1,2</math> || <math>4,0</math> || <math>4,1</math> || <math>3,2</math> || <math>5,0</math> || <math>0,3</math> || <math>5,1</math> || <math>2,3</math> || <math>6,0</math> || <math>5,2</math> || <math>6,1</math> || <math>4,3</math> || <math>6,2</math> || <math>7,0</math> || <math>7,1</math> || <math>3,4</math> || <math>7,2</math> || <math>6,3</math> || <math>8,0</math> || <math>8,1</math> || <math>5,4</math> || <math>0,5</math> || <math>8,2</math> || <math>2,5</math> || <math>9,0</math> || <math>9,1</math> || <math>8,3</math> || <math>9,2</math> || <math>7,4</math> || <math>10,0</math> || <math>10,1</math>
+
| [https://oeis.org/A221711 A221711]
 
|-
 
|-
! <math>\boldsymbol{x,y}</math>
+
| 3. <math>\quad \sum_{p \geqslant 2} \frac{\log p}{p (p - 1)} = 0.755366610831 \ldots</math>
| <math></math> || <math></math> || <math>1,1</math> || <math></math> || <math></math> || <math>0,2</math> || <math></math> || <math>2,2</math> || <math></math> || <math></math> || <math></math> || <math></math> || <math>4,2</math> || <math></math> || <math>3,3</math> || <math></math> || <math></math> || <math></math> || <math>0,4</math> || <math>1,4</math> || <math>5,3</math> || <math></math> || <math></math> || <math></math> || <math>4,4</math> || <math></math> || <math></math> || <math></math> || <math>7,3</math> || <math></math> || <math></math> || <math>6,4</math> || <math>4,5</math> || <math></math> || <math></math> || <math>5,5</math> || <math></math>
+
| [https://oeis.org/A138312 A138312]
 
|-
 
|-
! <math>\boldsymbol{x,y}</math>  
+
| 4. <math>\quad \sum_{p \geqslant 2} \frac{\log p}{p^2} = 0.493091109368 \ldots</math>
| <math></math> || <math></math> || <math></math> || <math></math> || <math></math> || <math></math> || <math></math> || <math></math> || <math></math> || <math></math> || <math></math> || <math></math> || <math>1,3</math> || <math></math> || <math></math> || <math></math> || <math></math> || <math></math> || <math></math> || <math></math> || <math>2,4</math> || <math></math> || <math></math> || <math></math> || <math></math> || <math></math> || <math></math> || <math></math> || <math>1,5</math> || <math></math> || <math></math> || <math>3,5</math> || <math></math> || <math></math> || <math></math> || <math></math> || <math></math>
+
| [https://oeis.org/A136271 A136271]
 
|}
 
|}
 
Zauważmy, że liczba złożona <math>55</math> nie ma rozkładu na sumę postaci <math>x^2 + 3 y^2</math>, a&nbsp;liczba złożona <math>91</math> ma dwa takie rozkłady. Obie liczby są postaci <math>6 k + 1</math>.
 
 
 
 
<span style="font-size: 110%; font-weight: bold;">Twierdzenie K53</span><br/>
 
Jeżeli liczba nieparzysta postaci <math>Q = x^2 + n y^2</math>, gdzie <math>n \in \{ 1, 2, 3 \}</math>, ma dwa różne takie przedstawienia w&nbsp;liczbach całkowitych dodatnich, to jest liczbą złożoną.
 
  
 
{{Spoiler|Style = font-style: italic; font-weight: bold; color: olive; text-decoration: underline;|Show=Dowód|Hide=Ukryj dowód}}
 
{{Spoiler|Style = font-style: italic; font-weight: bold; color: olive; text-decoration: underline;|Show=Dowód|Hide=Ukryj dowód}}
W dowodzie wyróżniliśmy miejsca, które wymagają oddzielnej analizy ze względu na wartość liczby <math>n</math>.
+
'''Punkt 1.'''<br/>
 
+
Zbieżność tego szeregu udowodniliśmy w&nbsp;twierdzeniu B39, ale obecnie potrafimy uzyskać rezultat znacznie łatwiej. Zauważmy, że rozpatrywaną sumę możemy zapisać w&nbsp;postaci
Niech
 
 
 
::<math>Q = x^2 + n y^2 = a^2 + n b^2</math>
 
 
 
<div style="border: thin solid black; padding-top: 0em; margin-top: 0.5em; padding-bottom: 0em; margin-bottom: 0.5em;">
 
<math>\boldsymbol{n = 1}</math>
 
 
 
Z założenia <math>Q</math> jest liczbą nieparzystą, zatem liczby występujące w&nbsp;rozkładach <math>x^2 + y^2 = a^2 + b^2</math> muszą mieć przeciwną parzystość. Nie zmniejszając ogólności, możemy założyć, że liczby <math>x, a</math> są nieparzyste, a&nbsp;liczby <math>y, b</math> parzyste.
 
 
 
<math>\boldsymbol{n = 2}</math>
 
 
 
Z założenia <math>Q</math> jest liczbą nieparzystą, zatem liczby <math>x, a</math> występująca w&nbsp;rozkładach <math>x^2 + 2 y^2 = a^2 + 2 b^2</math> muszą być nieparzyste. Pokażemy, że liczby <math>y, b</math> muszą mieć taką samą parzystość. Przypuśćmy, że <math>y</math> jest parzysta, a <math>b</math> nieparzysta, wtedy modulo <math>4</math> dostajemy
 
 
 
::<math>1 + 2 \cdot 0 \equiv 1 + 2 \cdot 1 \!\! \pmod{4}</math>
 
 
 
Co jest niemożliwe.
 
 
 
<math>\boldsymbol{n = 3}</math>
 
 
 
Z założenia <math>Q</math> jest liczbą nieparzystą, zatem liczby występujące w&nbsp;rozkładach <math>x^2 + 3 y^2 = a^2 + 3 b^2</math> muszą mieć przeciwną parzystość. Pokażemy, że liczby <math>x, a</math> muszą mieć taką samą parzystość. Gdyby liczba <math>x</math> była nieparzysta, a&nbsp;liczba <math>a</math> parzysta, to modulo <math>4</math> mielibyśmy
 
  
::<math>1 + 3 \cdot 0 \equiv 0 + 3 \cdot 1 \!\! \pmod{4}</math>
+
::<math>\sum_{p \geqslant 2} \frac{1}{p \log p} = \sum_{k = 1}^{\infty} \frac{1}{p_k \log p_k} = \frac{1}{2 \log 2} + \sum_{k = 2}^{\infty} \frac{1}{p_k \log p_k}</math>
  
Co jest niemożliwe.
+
Wyrażenie w&nbsp;mianowniku ułamka możemy łatwo oszacować. Z&nbsp;twierdzenia A1 mamy (<math>a = 0.72</math>)
</div>
 
Z powyższego zestawienia wynika, że liczby <math>x, a</math> i liczby <math>y, b</math> mają taką samą parzystość. Mamy
 
  
::<math>x^2 - a^2 = n (b^2 - y^2)</math>
+
::<math>p_k \log p_k > a \cdot k \log k \cdot \log (a \cdot k \log k) =</math>
  
::<math>(x - a) (x + a) = n (b - y) (b + y)</math>
+
::::<math>\;\;\:\, = a \cdot k \log k \cdot (\log a + \log k + \log \log k) =</math>
  
Niech <math>f = \gcd (x - a, b - y)</math>, zatem <math>f</math> jest liczbą parzystą i
+
::::<math>\;\;\:\, = a \cdot k \cdot (\log k)^2 \cdot \left( 1 + \frac{\log a + \log \log k}{\log k} \right)</math>
  
::<math>x - a = f r , \qquad \qquad b - y = f s , \qquad \qquad \gcd (r, s) = 1</math>
+
Ponieważ dla <math>k > \exp \left( \tfrac{1}{a} \right) = 4.01039 \ldots</math> jest
  
Czyli
+
::<math>\log a + \log \log k > 0</math>
  
::<math>r(x + a) = n s (y + b)</math>
+
to dla <math>k \geqslant 5</math> prawdziwe jest oszacowanie
  
ale liczby <math>r, s</math> są względnie pierwsze, zatem <math>s \mid (x + a)</math> i&nbsp;musi być
+
::<math>p_k \log p_k > a \cdot k \cdot (\log k)^2</math>
  
::<math>x + a = k s \qquad \qquad \Longrightarrow \qquad \qquad n (y + b) = k r</math>
+
Wynika stąd, że dla <math>k \geqslant 5</math> prawdziwy jest ciąg nierówności
  
Gdyby <math>k</math> było liczbą nieparzystą, to liczby <math>r, s</math> musiałyby być parzyste, co jest niemożliwe, bo <math>\gcd (r, s) = 1</math>. Zatem <math>k</math> jest liczbą parzystą i <math>2 s \mid (x + a)</math>, czyli możemy pokazać więcej. Musi być
+
::<math>0 < \frac{1}{p_k \log p_k} < \frac{1}{a \cdot k \cdot (\log k)^2}</math>
  
::<math>x + a = 2 l s \qquad \qquad \Longrightarrow \qquad \qquad n (y + b) = 2 l r</math>
+
Zatem na mocy kryterium porównawczego ze zbieżności szeregu <math>\sum_{k = 2}^{\infty} \frac{1}{k \cdot (\log k)^2}</math> (zobacz twierdzenie D13 p. 4 lub przykład D18 p. 5) wynika zbieżność szeregu <math>\sum_{k = 2}^{\infty} \frac{1}{p_k \log p_k}</math>
  
W przypadku gdy <math>n = 2</math> lub <math>n = 3</math>, zauważmy, że <math>n \mid l</math> lub <math>n \mid r</math>.
+
'''Punkt 2.'''<br/>
 +
Zbieżność szeregu wynika z&nbsp;kryterium porównawczego (twierdzenie D9), bo
  
Łatwo otrzymujemy
+
::<math>0 < \frac{1}{p^2 \log p} < \frac{1}{p \log p}</math>
  
::<math>x = {\small\frac{1}{2}} (2 l s + f r)</math>
+
'''Punkt 3.'''<br/>
 +
Szereg jest zbieżny, bo sumy częściowe tego szeregu tworzą ciąg rosnący i&nbsp;ograniczony
  
::<math>y = {\small\frac{1}{2 n}} (2 l r - n f s)</math>
+
::<math>\sum_{p \leqslant n} \frac{\log p}{p (p - 1)} < \sum_{k = 2}^{\infty} \frac{\log k}{k (k - 1)} = 1.2577 \ldots</math>
  
Ostatecznie
+
'''Punkt 4.'''<br/>
 +
Zbieżność szeregu wynika z&nbsp;kryterium porównawczego, bo dla każdego <math>p \geqslant 2</math> jest
  
::<math>Q = x^2 + n y^2</math>
+
::<math>0 < \frac{\log p}{p^2} < \frac{\log p}{p (p - 1)}</math><br/>
 
 
::<math>\;\;\;\: = \left[ {\small\frac{1}{2}} (2 l s + f r) \right]^2 + n \left[ {\small\frac{1}{2 n}} (2 l r - n f s) \right]^2</math>
 
 
 
::<math>\;\;\;\: = {\small\frac{1}{4 n}} [n (2 l s + f r)^2 + (2 l r - n f s)^2]</math>
 
 
 
::<math>\;\;\;\: = {\small\frac{1}{4 n}} [n (2 l s)^2 + n (f r)^2 + (2 l r)^2 + (n f s)^2]</math>
 
 
 
::<math>\;\;\;\: = {\small\frac{1}{4 n}} [(2 l)^2 + n f^2] (r^2 + n s^2)</math>
 
 
 
<div style="border: thin solid black; padding-top: 0em; margin-top: 0.5em; padding-bottom: 0em; margin-bottom: 0.5em;">
 
<math>\boldsymbol{n = 1}</math>
 
 
 
::<math>Q = {\small\frac{1}{4}} [(2 l)^2 + f^2] (r^2 + s^2) = \left[ l^2 + \left( {\small\frac{f}{2}} \right)^2 \right] (r^2 + s^2)</math>
 
 
 
<math>\boldsymbol{n = 2 , 3}</math>
 
 
 
W zależności od tego, która z&nbsp;liczb <math>l, r</math> jest podzielna przez <math>n</math>, możemy napisać
 
 
 
::<math>Q = {\small\frac{1}{4 n}} [(2 l)^2 + n f^2] (r^2 + n s^2) = \left[ {\small\frac{(2 l)^2 + n f^2}{4 n}} \right] (r^2 + n s^2) = \left[ {\small\frac{(2 l)^2 + n f^2}{4}} \right] \left( {\small\frac{r^2 + n s^2}{n}} \right)</math>
 
</div>
 
 
 
Co kończy dowód.<br/>
 
 
&#9633;
 
&#9633;
 
{{\Spoiler}}
 
{{\Spoiler}}
Linia 1835: Linia 961:
  
  
<span style="font-size: 110%; font-weight: bold;">Uwaga K54</span><br/>
+
<span style="font-size: 110%; font-weight: bold;">Twierdzenie D29</span><br/>
Zauważmy, że iloczyn liczb postaci <math>x^2 + n y^2</math> jest liczbą tej samej postaci.
+
Szereg <math>\sum_{p \geqslant 2} \frac{\log p}{p}</math> jest rozbieżny.
 
 
::<math>(a^2 + n b^2) (x^2 + n y^2) = (a x + n b y)^2 + n (a y - b x)^2</math>
 
 
 
::::::::<math>\;\;\;\:\, = (a x - n b y)^2 + n (a y + b x)^2</math>
 
 
 
 
 
 
 
<span style="font-size: 110%; font-weight: bold;">Twierdzenie K55</span><br/>
 
Niech <math>x, y, a, b \in \mathbb{Z}</math> i <math>n \in \{ 1, 2, 3 \}</math>. Jeżeli liczba parzysta <math>Q = x^2 + n y^2</math>, to <math>Q = 2^{\alpha} R</math>, gdzie <math>R = a^2 + n b^2</math> jest liczbą nieparzystą.
 
  
 
{{Spoiler|Style = font-style: italic; font-weight: bold; color: olive; text-decoration: underline;|Show=Dowód|Hide=Ukryj dowód}}
 
{{Spoiler|Style = font-style: italic; font-weight: bold; color: olive; text-decoration: underline;|Show=Dowód|Hide=Ukryj dowód}}
W szczególnym przypadku, gdy <math>R = 1</math>, mamy <math>R = 1^2 + n \cdot 0^2</math>.
+
Dla potrzeb dowodu zapiszmy szereg w&nbsp;innej postaci
 
 
Dowód sprowadza się do podania wzorów, które pozwalają obniżyć wykładnik, z&nbsp;jakim liczba <math>2</math> występuje w&nbsp;rozwinięciu na czynniki pierwsze liczby <math>Q</math>. Zauważmy, że wynik jest zawsze liczbą, której postać jest taka sama, jak postać liczby <math>Q</math>. Stosując te wzory odpowiednią ilość razy, otrzymujmy rozkład <math>Q = 2^{\alpha} R</math>, gdzie <math>R</math> jest liczbą nieparzystą postaci <math>a^2 + n b^2</math>.
 
 
 
'''1.''' <math>\boldsymbol{Q = x^2 + y^2}</math>
 
 
 
a) jeżeli liczby <math>x, y</math> są parzyste, to <math>{\small\frac{Q}{4}} = \left( {\small\frac{x}{2}} \right)^2 + \left( {\small\frac{y}{2}} \right)^2</math>
 
 
 
b) jeżeli liczby <math>x, y</math> są nieparzyste, to <math>{\small\frac{Q}{2}} = \left( {\small\frac{x + y}{2}} \right)^2 + \left( {\small\frac{x - y}{2}} \right)^2</math>
 
 
 
'''2.''' <math>\boldsymbol{Q = x^2 + 2 y^2}</math>
 
 
 
a) jeżeli liczby <math>x, y</math> są parzyste, to <math>{\small\frac{Q}{4}} = \left( {\small\frac{x}{2}} \right)^2 + 2 \left( {\small\frac{y}{2}} \right)^2</math>
 
 
 
b) jeżeli liczba <math>x</math> jest parzysta, a <math>y</math> nieparzysta, to <math>{\small\frac{Q}{2}} = y^2 + 2 \left( {\small\frac{x}{2}} \right)^2</math>
 
  
'''3.''' <math>\boldsymbol{Q = x^2 + 3 y^2}</math>
+
::<math>\sum_{p \geqslant 2} \frac{\log p}{p} = \sum_{k = 1}^{\infty} \frac{\log p_k}{p_k}</math>
  
a) jeżeli liczby <math>x, y</math> są parzyste, to <math>{\small\frac{Q}{4}} = \left( {\small\frac{x}{2}} \right)^2 + 3 \left( {\small\frac{y}{2}} \right)^2</math>
+
Zauważmy, że dla <math>k \geqslant 3</math> wyrazy szeregów <math>\sum_{k = 1}^{\infty} \frac{1}{p_k}</math> oraz <math>\sum_{k = 1}^{\infty} \frac{\log p_k}{p_k}</math> spełniają nierówności
  
b) jeżeli liczby <math>x, y</math> są nieparzyste i <math>4 \mid (x + y)</math>, to <math>{\small\frac{Q}{4}} = \left( {\small\frac{x - 3 y}{4}} \right)^2 + 3 \left( {\small\frac{x + y}{4}} \right)^2</math>
+
::<math>0 \leqslant \frac{1}{p_k} \leqslant \frac{\log p_k}{p_k}</math>
  
c) jeżeli liczby <math>x, y</math> są nieparzyste i <math>4 \mid (x - y)</math>, to <math>{\small\frac{Q}{4}} = \left( {\small\frac{x + 3 y}{4}} \right)^2 + 3 \left( {\small\frac{x - y}{4}} \right)^2</math>
+
Ponieważ szereg <math>\sum_{k = 1}^{\infty} \frac{1}{p_k}</math> jest rozbieżny, to na mocy kryterium porównawczego (twierdzenie D9) rozbieżny jest również szereg <math>\sum_{k = 1}^{\infty} \frac{\log p_k}{p_k}</math><br/>
 
 
Co należało pokazać.<br/>
 
 
&#9633;
 
&#9633;
 
{{\Spoiler}}
 
{{\Spoiler}}
Linia 1878: Linia 979:
  
  
<span style="font-size: 110%; font-weight: bold;">Twierdzenie K56</span><br/>
+
<span style="font-size: 110%; font-weight: bold;">Uwaga D30</span><br/>
Liczba pierwsza <math>p \geqslant 3</math> jest postaci
+
Moglibyśmy oszacować rozbieżność szeregu <math>\sum_{p \geqslant 2} \frac{\log p}{p}</math> podobnie, jak to uczyniliśmy w&nbsp;przypadku twierdzenia B37, ale tym razem zastosujemy inną metodę, która pozwoli nam uzyskać bardziej precyzyjny rezultat.
  
:(a)&nbsp;&nbsp;<math>4 k + 1</math>
 
  
:(b)&nbsp;&nbsp;<math>8 k + 1 \,</math> lub <math>\: 8 k + 3</math>
 
  
:(c)&nbsp;&nbsp;<math>6 k + 1</math>
+
<span style="font-size: 110%; font-weight: bold;">Twierdzenie D31</span><br/>
 +
Niech <math>n \in \mathbb{Z}_+</math>. Prawdziwe są następujące nierówności
  
wtedy i&nbsp;tylko wtedy, gdy istnieje dokładnie jedna para liczb całkowitych dodatnich <math>x, y</math>, że
+
::{| class="wikitable plainlinks"  style="font-size: 100%; text-align: center; margin-right: auto;"
 
+
|- style=height:3em
:(a)&nbsp;&nbsp;<math>p = x^2 + y^2</math>
+
| <math>\quad 1. \quad</math> || <math>n! > n^n e^{- n}</math> || <math>\text{dla} \;\; n \geqslant 1</math>
 
+
|- style=height:3em
:(b)&nbsp;&nbsp;<math>p = x^2 + 2 y^2</math>
+
| <math>\quad 2. \quad</math> || <math>n! < n^{n + 1} e^{- n}</math> || <math>\text{dla} \;\; n \geqslant 7</math>
 
+
|}
:(c)&nbsp;&nbsp;<math>p = x^2 + 3 y^2</math>
 
  
 
{{Spoiler|Style = font-style: italic; font-weight: bold; color: olive; text-decoration: underline;|Show=Dowód|Hide=Ukryj dowód}}
 
{{Spoiler|Style = font-style: italic; font-weight: bold; color: olive; text-decoration: underline;|Show=Dowód|Hide=Ukryj dowód}}
 +
'''Punkt 1. (indukcja matematyczna)'''<br/>
 +
Łatwo sprawdzić prawdziwość nierówności dla <math>n = 1</math>. Zakładając prawdziwość dla <math>n</math>, otrzymujemy dla <math>n + 1</math>
  
<math>\Large{\Longleftarrow}</math>
+
::<math>(n + 1) ! = n! \cdot (n + 1) ></math>
  
Niech <math>n = 1, 2, 3</math>. Z&nbsp;założenia liczba pierwsza <math>p \geqslant 3</math> może być przedstawiona w&nbsp;postaci <math>p = x_0^2 + n y_0^2</math>, gdzie <math>x_0, y_0</math> są liczbami takimi, że <math>1 \leqslant x_0, y_0 < p</math>. Zatem <math>p \nmid x_0</math> i <math>p \nmid y_0</math>, a&nbsp;rozpatrując równanie <math>p = x_0^2 + n y_0^2</math> modulo <math>p</math> dostajemy
+
::::<math>\;\;\; > n^n \cdot e^{- n} \cdot (n + 1) =</math>
  
::<math>x_0^2 + n y_0^2 \equiv 0 \!\! \pmod{p}</math>
+
::::<math>\;\;\; = (n + 1)^{n + 1} \cdot \frac{n^n}{(n + 1)^n} \cdot e^{- n} =</math>
  
Zauważmy, że liczba <math>x_0</math> jest rozwiązaniem kongruencji
+
::::<math>\;\;\; = (n + 1)^{n + 1} \cdot \frac{1}{\left( 1 + \frac{1}{n} \right)^n} \cdot e^{- n} ></math>
  
::<math>x^2 \equiv - n y_0^2 \!\! \pmod{p}</math>
+
::::<math>\;\;\; > (n + 1)^{n + 1} \cdot \frac{1}{e} \cdot e^{- n} =</math>
  
Wynika stąd, że liczba <math>- n y_0^2</math> jest liczbą kwadratową modulo <math>p</math>. Zatem
+
::::<math>\;\;\; = (n + 1)^{n + 1} e^{- (n + 1)}</math>
  
<div style="margin-top: 1em; margin-bottom: 1em;">
+
Ponieważ <math>\left( 1 + \frac{1}{n} \right)^n < e</math>, zatem <math>\frac{1}{\left( 1 + \frac{1}{n} \right)^n} > \frac{1}{e}</math>. Co kończy dowód punktu 1.
::<math>\left( {\small\frac{- n y_0^2}{p}} \right)_{\small{\!\! J}} = \left( {\small\frac{- n}{p}} \right)_{\small{\!\! J}} \cdot \left( {\small\frac{y_0^2}{p}} \right)_{\small{\!\! J}} = \left( {\small\frac{- n}{p}} \right)_{\small{\!\! J}} = 1</math>
 
</div>
 
  
Z twierdzenia J41 i&nbsp;zadania J45 otrzymujemy natychmiast
 
  
:(a) jeżeli <math>\left( {\small\frac{- 1}{p}} \right)_{\small{\!\! J}} = 1</math>, to liczba pierwsza <math>p</math> musi być postaci <math>4 k + 1</math>
+
'''Punkt 2. (indukcja matematyczna)'''<br/>
 +
Łatwo sprawdzić prawdziwość nierówności dla <math>n = 7</math>. Zakładając prawdziwość dla <math>n</math>, otrzymujemy dla <math>n + 1</math>
  
:(b) jeżeli <math>\left( {\small\frac{- 2}{p}} \right)_{\small{\!\! J}} = 1</math>, to liczba pierwsza <math>p</math> musi być postaci <math>8 k + 1</math> lub <math>8 k + 3</math>
+
::<math>(n + 1) ! = n! \cdot (n + 1) <</math>
  
:(c) jeżeli <math>\left( {\small\frac{- 3}{p}} \right)_{\small{\!\! J}} = 1</math>, to liczba pierwsza <math>p</math> musi być postaci <math>6 k + 1</math>
+
::::<math>\;\;\; < n^{n + 1} \cdot e^{- n} \cdot (n + 1) =</math>
  
Co należało pokazać.
+
::::<math>\;\;\; = (n + 1)^{n + 2} \cdot \frac{n^{n + 1}}{(n + 1)^{n + 1}} \cdot e^{- n} =</math>
  
 +
::::<math>\;\;\; = (n + 1)^{n + 2} \cdot \left( \frac{n}{n + 1} \right)^{n + 1} \cdot e^{- n} =</math>
  
<math>\Large{\Longrightarrow}</math>
+
::::<math>\;\;\; = (n + 1)^{n + 2} \cdot \left( 1 - \frac{1}{n + 1} \right)^{n + 1} \cdot e^{- n} <</math>
  
'''A. Istnienie rozwiązania kongruencji''' <math>\boldsymbol{x^2 + n y^2 \equiv 0 \!\! \pmod{p}}</math>
+
::::<math>\;\;\; < (n + 1)^{n + 2} \cdot \frac{1}{e} \cdot e^{- n} =</math>
  
Z założenia liczba pierwsza <math>p \geqslant 3</math> jest postaci
+
::::<math>\;\;\; = (n + 1)^{n + 2} \cdot e^{- (n + 1)}</math>
  
:(a)&nbsp;&nbsp;<math>4 k + 1</math>
+
Ostatnia nierówność wynika z&nbsp;faktu, że <math>\left( 1 - \frac{1}{n + 1} \right)^{n + 1} < \frac{1}{e}</math>. Co kończy dowód punktu 2.<br/>
 +
&#9633;
 +
{{\Spoiler}}
  
:(b)&nbsp;&nbsp;<math>8 k + 1 \,</math> lub <math>\: 8 k + 3</math>
 
  
:(c)&nbsp;&nbsp;<math>6 k + 1</math>
 
  
Wynika stąd, że dla (a) <math>n = 1</math>, (b) <math>n = 2</math>, (c) <math>n = 3</math> mamy
+
<span style="font-size: 110%; font-weight: bold;">Twierdzenie D32</span><br/>
 +
Niech <math>n \in \mathbb{Z}_+</math>. Dla wykładnika, z&nbsp;jakim liczba pierwsza <math>p</math> występuje w&nbsp;rozwinięciu liczby <math>n!</math> na czynniki pierwsze, prawdziwe są oszacowania
  
::<math>\left( {\small\frac{- n}{p}} \right)_{\small{\!\! J}} = 1</math>
+
::{| class="wikitable plainlinks"  style="font-size: 100%; text-align: right; margin-right: auto;"
 +
|- style=height:3em
 +
| <math>\quad 1. \quad</math> || <math>\frac{n}{p} - 1 < W_p (n!) < \frac{n}{p - 1}</math>
 +
|- style=height:3em
 +
| <math>\quad 2. \quad</math> || <math>\frac{n + 1}{p} - 1 \leqslant W_p (n!) \leqslant \frac{n - 1}{p - 1}</math>
 +
|}
  
(zobacz J41 i&nbsp;J45) i&nbsp;liczba <math>- n</math> jest liczbą kwadratową modulo <math>p</math>. Zatem kongruencja
+
{{Spoiler|Style = font-style: italic; font-weight: bold; color: olive; text-decoration: underline;|Show=Dowód|Hide=Ukryj dowód}}
 +
'''Punkt 1. (prawa nierówność)'''
  
::<math>x^2 \equiv - n \!\! \pmod{p}</math>
+
Zauważmy, że
  
ma rozwiązanie, czyli istnieje taka liczba <math>k</math>, że
+
::<math>W_p (n!) = \left\lfloor \frac{n}{p} \right\rfloor + \left\lfloor \frac{n}{p^2} \right\rfloor + \left\lfloor \frac{n}{p^3} \right\rfloor + \ldots <</math>
  
::<math>k^2 + n \equiv 0 \!\! \pmod{p}</math>
+
::::<math>\;\, < \frac{n}{p} + \frac{n}{p^2} + \frac{n}{p^3} + \ldots + \frac{n}{p^k} + \ldots =</math>
  
Zauważmy, że liczby <math>x_0 = k</math> i <math>y_0 = 1</math> są szczególnymi przypadkami rozwiązania kongruencji
+
::::<math>\;\, = \frac{n}{p} \cdot \frac{1}{1 - \frac{1}{p}} =</math>
  
::<math>x^2 + n y^2 \equiv 0 \!\! \pmod{p}</math>
+
::::<math>\;\, = \frac{n}{p - 1}</math>
  
W przypadku (a), korzystając z&nbsp;twierdzenia Wilsona (zobacz J18), liczbę <math>x_0</math> możemy jawnie wypisać: <math>x_0 = \left( {\small\frac{p - 1}{2}} \right) !</math>
+
'''Punkt 1. (lewa nierówność)'''
  
 +
Łatwo znajdujemy, że
  
'''B. Zmniejszenie rozwiązania początkowego'''
+
::<math>W_p (n!) = \sum_{k = 1}^{\infty} \left\lfloor \frac{n}{p^k} \right\rfloor \geqslant \left\lfloor \frac{n}{p} \right\rfloor > \frac{n}{p} - 1</math>
  
Niech liczby <math>x_0, y_0</math> takie, że <math>p \nmid x_0 \,</math> i <math>\, p \nmid y_0</math> spełniają kongruencję
+
'''Punkt 2. (prawa nierówność)'''
  
::<math>x_0^2 + n y_0^2 \equiv 0 \!\! \pmod{p}</math>
+
Z uzyskanego w&nbsp;punkcie 1. oszacowania wynika, że <math>(p - 1) W_p (n!) < n</math>. Ponieważ nierówność ta dotyczy liczb całkowitych, to możemy napisać
  
Wybierzmy liczby <math>r, s</math> tak, aby były najbliższymi liczbami całkowitymi odpowiednio dla liczb <math>{\small\frac{x_0}{p}} \,</math> i <math>\, {\small\frac{y_0}{p}}</math>. Z&nbsp;definicji mamy
+
::<math>(p - 1) W_p (n!) \leqslant n - 1</math>
  
::<math>\left| {\small\frac{x_0}{p}} - r \right| \leqslant {\small\frac{1}{2}} \qquad \qquad \text{i} \qquad \qquad \left| {\small\frac{y_0}{p}} - s \right| \leqslant {\small\frac{1}{2}}</math>
+
Skąd otrzymujemy natychmiast nierówność nieostrą <math>W_p (n!) \leqslant \frac{n - 1}{p - 1}</math>.
  
Zatem
+
'''Punkt 2. (lewa nierówność)'''
  
::<math>| x_0 - r p | \leqslant {\small\frac{p}{2}} \qquad \qquad \text{i} \qquad \qquad | y_0 - s p | \leqslant {\small\frac{p}{2}}</math>
+
Z uzyskanego w&nbsp;punkcie 1. oszacowania wynika, że <math>n - p < p \cdot W_p (n!)</math>. Ponieważ nierówność ta dotyczy liczb całkowitych, to możemy napisać
  
Ponieważ liczby po lewej stronie nierówności są liczbami całkowitymi, to nigdy nie będą równe liczbie <math>{\small\frac{p}{2}}</math>, gdzie <math>p</math> jest liczbą nieparzystą. Pozwala to wzmocnić wypisane nierówności.
+
::<math>n - p \leqslant p \cdot W_p (n!) - 1</math>
  
::<math>| x_0 - r p | < {\small\frac{p}{2}} \qquad \qquad \text{i} \qquad \qquad | y_0 - s p | < {\small\frac{p}{2}}</math>
+
Skąd otrzymujemy natychmiast nierówność nieostrą <math>W_p (n!) \geqslant \frac{n + 1}{p} - 1</math>.<br/>
 +
&#9633;
 +
{{\Spoiler}}
  
Wynika stąd, że dla dowolnego rozwiązania początkowego <math>x_0, y_0</math> możemy wybrać liczby
 
  
::<math>x = x_0 - r p \qquad \qquad \text{i} \qquad \qquad y = y_0 - s p</math>
 
  
takie, że <math>p \nmid x</math> oraz <math>p \nmid y</math> i&nbsp;dla których
+
<span style="font-size: 110%; font-weight: bold;">Twierdzenie D33</span><br/>
 +
Dla dowolnego <math>n \in \mathbb{Z}_+</math> prawdziwe jest następujące oszacowanie
  
::<math>0 < x^2 + n y^2 < \left( {\small\frac{p}{2}} \right)^2 + n \left( {\small\frac{p}{2}} \right)^2 = {\small\frac{(n + 1) p}{4}} \cdot p</math>
+
::<math>\sum_{p \leqslant n} \frac{\log p}{p - 1} - \log n > - 1</math>
  
Ponieważ modulo <math>p</math> jest <math>x \equiv x_0 \,</math> i <math>\, y \equiv y_0</math>, to liczby <math>x, y</math> spełniają kongruencję
+
{{Spoiler|Style = font-style: italic; font-weight: bold; color: olive; text-decoration: underline;|Show=Dowód|Hide=Ukryj dowód}}
 +
Z oszacowania wykładnika, z&nbsp;jakim liczba pierwsza <math>p</math> występuje w rozwinięciu liczby <math>n!</math> na czynniki pierwsze, wynika natychmiast, że dla <math>n \geqslant 2</math> mamy
  
::<math>x^2 + n y^2 \equiv 0 \!\! \pmod{p}</math>
+
::<math>n! < \prod_{p \leqslant n} p^{n / (p - 1)}</math>
  
Zatem wynikające z&nbsp;powyższej kongruencji równanie
+
Ponieważ dla <math>n \geqslant 1</math> jest <math>n! > n^n e^{- n}</math> (zobacz punkt 1. twierdzenia D31), to
  
::<math>x^2 + n y^2 = m p</math>
+
::<math>n^n e^{- n} < \prod_{p \leqslant n} p^{n / (p - 1)}</math>
  
ma rozwiązanie dla liczb
+
Logarytmując, otrzymujemy
  
::<math>| x | < {\small\frac{p}{2}} , \qquad \qquad | y | < {\small\frac{p}{2}}, \qquad \qquad 1 \leqslant m < {\small\frac{(n + 1) p}{4}}</math>
+
::<math>n \log n - n < \sum_{p \leqslant n} \frac{n \log p}{p - 1} = n \sum_{p \leqslant n} \frac{\log p}{p - 1}</math>
  
Pomysł ze zmniejszaniem liczb stanowiących rozwiązanie za chwilę wykorzystamy ponownie i&nbsp;będzie to istotny element dowodu.
+
Dzieląc strony przez <math>n</math>, dostajemy szukaną nierówność.<br/>
 +
&#9633;
 +
{{\Spoiler}}
  
  
'''C. Metoda nieskończonego schodzenia Fermata'''<ref name="InfiniteDescent1"/><ref name="Bussey1"/>
 
  
Pomysł dowodu został zaczerpnięty z&nbsp;książki Hardy'ego i&nbsp;Wrighta<ref name="HardyWright1"/>.
+
<span style="font-size: 110%; font-weight: bold;">Twierdzenie D34 (pierwsze twierdzenie Mertensa</span><ref name="Mertens1"/><ref name="Mertens2"/><span style="font-size: 110%; font-weight: bold;">, 1874)</span><br/>
 +
Dla dowolnego <math>n \in \mathbb{Z}_+</math> prawdziwe jest następujące oszacowanie
  
Jeżeli w&nbsp;rozwiązaniu <math>m = 1</math>, to <math>p = x^2 + n y^2</math> i&nbsp;twierdzenie jest udowodnione. W&nbsp;przypadku gdy <math>m > 1</math> wskażemy sposób postępowania, który pozwoli nam z&nbsp;istniejącego rozwiązania równania
+
::<math>\sum_{p \leqslant n} \frac{\log p}{p} - \log n > - 1.755367</math>
  
::<math>x^2 + n y^2 = m p</math>
+
{{Spoiler|Style = font-style: italic; font-weight: bold; color: olive; text-decoration: underline;|Show=Dowód|Hide=Ukryj dowód}}
 +
Ponieważ
  
otrzymać nowe rozwiązanie tej samej postaci
+
::<math>\frac{1}{p - 1} = \frac{1}{p} + \frac{1}{p (p - 1)}</math>
  
::<math>x_1^2 + n y_1^2 = m_1 p</math>
 
  
takie, że <math>1 \leqslant m_1 < m</math>. Powtarzając tę procedurę odpowiednią ilość razy, otrzymamy rozwiązanie <math>x_k, y_k, m_k</math>, gdzie <math>m_k = 1</math>. Istnienie takiej procedury stanowi dowód prawdziwości twierdzenia.
+
to z&nbsp;twierdzenia D33 dostajemy
  
Zauważmy, że podział na parzyste i&nbsp;nieparzyste liczby <math>m</math> jest konieczny tylko w&nbsp;przypadku gdy <math>n = 3</math>. W&nbsp;pozostałych przypadkach nie musimy wzmacniać nierówności, aby prawdziwe było oszacowanie <math>1 \leqslant m_1 < m</math>.
+
::<math>\sum_{p \leqslant n} \frac{\log p}{p} + \sum_{p \leqslant n} \frac{\log p}{p (p - 1)} - \log n > - 1</math>
  
'''Przypadek, gdy''' <math>\boldsymbol{m > 1}</math> '''jest liczbą parzystą'''
+
Czyli
  
Jeżeli <math>m > 1</math> jest liczbą parzystą, to z&nbsp;twierdzenia K55 wiemy, że liczba <math>x^2 + n y^2</math> może być zapisana w&nbsp;postaci
+
::<math>\sum_{p \leqslant n} \frac{\log p}{p} - \log n > - 1 - \sum_{p \leqslant n} \frac{\log p}{p (p - 1)}</math>
  
::<math>x^2 + n y^2 = 2^{\alpha} (x^2_1 + n y^2_1)</math>
+
:::::::<math>\;\, > - 1 - \sum_{p \geqslant 2} \frac{\log p}{p (p - 1)}</math>
  
gdzie <math>x^2_1 + n y^2_1</math> jest liczbą nieparzystą. Wystarczy położyć <math>m_1 = {\small\frac{m}{2^{\alpha}}}</math>, aby z&nbsp;istniejącego rozwiązania otrzymać nowe rozwiązanie tej samej postaci
+
:::::::<math>\;\, = - 1 - 0.755366610831 \ldots</math>
  
::<math>x_1^2 + n y_1^2 = m_1 p</math>
+
:::::::<math>\;\, > - 1.755367</math>
  
gdzie <math>m_1</math> jest liczbą nieparzystą i <math>1 \leqslant m_1 < m</math>.
+
Gdzie wykorzystaliśmy zbieżność szeregu <math>\sum_{p \geqslant 2} \frac{\log p}{p (p - 1)}</math> (twierdzenie D28 p. 3).<br/>
 +
&#9633;
 +
{{\Spoiler}}
  
'''Przypadek, gdy''' <math>\boldsymbol{m > 1}</math> '''jest liczbą nieparzystą'''
 
  
Niech liczby <math>r, s</math> będą liczbami całkowitymi najbliższymi liczbom <math>{\small\frac{x}{m}} \,</math> i <math>\, {\small\frac{y}{m}}</math>. Z&nbsp;definicji mamy
 
  
::<math>\left| {\small\frac{x}{m}} - r \right| \leqslant {\small\frac{1}{2}} \qquad \qquad \text{i} \qquad \qquad \left| {\small\frac{y}{m}} - s \right| \leqslant {\small\frac{1}{2}}</math>
+
<span style="font-size: 110%; font-weight: bold;">Twierdzenie D35 (pierwsze twierdzenie Mertensa</span><ref name="Mertens1"/><ref name="Mertens2"/><span style="font-size: 110%; font-weight: bold;">, 1874)</span><br/>
 +
Dla dowolnego <math>n \in \mathbb{Z}_+</math> prawdziwe jest następujące oszacowanie
  
Zatem
+
::<math>\sum_{p \leqslant n} \frac{\log p}{p} - \log n < 0.386295</math>
  
::<math>| x - r m | \leqslant {\small\frac{m}{2}} \qquad \qquad \text{i} \qquad \qquad | y - s m | \leqslant {\small\frac{m}{2}}</math>
+
{{Spoiler|Style = font-style: italic; font-weight: bold; color: olive; text-decoration: underline;|Show=Dowód|Hide=Ukryj dowód}}
 +
Z oszacowania wykładnika, z&nbsp;jakim liczba pierwsza <math>p</math> występuje w rozwinięciu liczby <math>n!</math> na czynniki pierwsze, wynika natychmiast, że dla <math>n \geqslant 1</math> mamy
  
Ponieważ liczby po lewej stronie nierówności są liczbami całkowitymi, to nigdy nie będą równe liczbie <math>{\small\frac{m}{2}}</math>, gdzie <math>m</math> jest liczbą nieparzystą. Pozwala to wzmocnić wypisane nierówności.
+
::<math>n! \geqslant \prod_{p \leqslant n} p^{(n + 1) / p \: - \: 1}</math>
  
::<math>| x - r m | < {\small\frac{m}{2}} \qquad \qquad \text{i} \qquad \qquad | y - s m | < {\small\frac{m}{2}}</math>
+
Ponieważ dla <math>n \geqslant 7</math> jest <math>n! < n^{n + 1} e^{- n}</math>, to
  
Połóżmy
+
::<math>\prod_{p \leqslant n} p^{(n + 1) / p \: - \: 1} < n^{n + 1} e^{- n}</math>
  
::<math>a = x - r m \qquad \qquad \text{i} \qquad \qquad b = y - s m</math>
+
Logarytmując, otrzymujemy
  
Zauważmy, że liczba <math>m</math> nie może jednocześnie dzielić liczb <math>x</math> i <math>y</math>, bo mielibyśmy <math>m^2 \mid (x^2 + n y^2)</math>, czyli <math>m \mid p</math>, co jest niemożliwe. Zatem przynajmniej jedna z&nbsp;liczb <math>a, b</math> musi być różna od <math>0</math>.
+
::<math>\sum_{p \leqslant n} \left( \frac{n + 1}{p} - 1 \right) \cdot \log p < (n + 1) \cdot \log n - n</math>
  
Rozpatrując równanie <math>x^2 + n y^2 = m p</math> modulo <math>m</math> i&nbsp;uwzględniając, że
+
::<math>(n + 1) \sum_{p \leqslant n} \frac{\log p}{p} - \sum_{p \leqslant n} \log p < (n + 1) \cdot \log n - n</math>
  
::<math>x \equiv a \!\! \pmod{m}</math>
 
  
::<math>y \equiv b \!\! \pmod{m}</math>
+
Skąd natychmiast wynika, że
  
otrzymujemy
+
::<math>\sum_{p \leqslant n} \frac{\log p}{p} - \log n < - \frac{n}{n + 1} + \frac{1}{n + 1} \cdot \log \left( \prod_{p \leqslant n} p \right)</math>
  
::<math>a^2 + n b^2 \equiv 0 \pmod{m}</math>
+
:::::::<math>\;\: = - 1 + \frac{1}{n + 1} + \frac{1}{n + 1} \cdot \log (P (n))</math>
  
Mamy też oszacowanie
+
:::::::<math>\;\: < - 1 + \frac{1}{n + 1} + \frac{n \cdot \log 4}{n + 1}</math>
  
::<math>0 < a^2 + n b^2 < \left( {\small\frac{m}{2}} \right)^2 + n \cdot \left( {\small\frac{m}{2}} \right)^2 = {\small\frac{(n + 1) m^2}{4}} = {\small\frac{(n + 1) m}{4}} \cdot m</math>
+
:::::::<math>\;\: = - 1 + \frac{1}{n + 1} + \log 4 - \frac{\log 4}{n + 1}</math>
  
Wynika stąd, że istnieje taka liczba <math>m_1</math> spełniająca warunek <math>1 \leqslant m_1 < {\small\frac{(n + 1) m}{4}}</math>, że
+
:::::::<math>\;\: = \log 4 - 1 + \frac{1 - \log 4}{n + 1}</math>
  
::<math>a^2 + n b^2 = m_1 m</math>
+
:::::::<math>\;\: = \log 4 - 1 - \frac{0.386294 \ldots}{n + 1}</math>
  
Mnożąc stronami powyższe równanie i&nbsp;równanie <math>x^2 + n y^2 = m p</math>, otrzymujemy
+
:::::::<math>\;\: < \log 4 - 1</math>
  
::<math>m_1 m^2 p = (a^2 + n b^2) (x^2 + n y^2)</math>
+
:::::::<math>\;\: = 0.386294361 \ldots</math>
  
::::<math>\;\; = (a x + n b y)^2 + n (a y - b x)^2</math>
+
Druga nierówność wynika z&nbsp;twierdzenia A9. Bezpośrednio sprawdzamy, że powyższa nierówność jest prawdziwa dla <math>n < 7</math>.<br/>
 +
&#9633;
 +
{{\Spoiler}}
  
(zobacz K54). Zauważmy teraz, że
 
  
::<math>a x + n b y = (x - r m) x + n (y - s m) y</math>
 
  
::::<math>\quad \; = x^2 - r m x + n y^2 - n s m y</math>
+
<span style="font-size: 110%; font-weight: bold;">Twierdzenie D36</span><br/>
 +
Dla dowolnego <math>n \in \mathbb{Z}_+</math> prawdziwe jest następujące oszacowanie
  
::::<math>\quad \; = m (p - r x - n s y)</math>
+
::<math>\sum_{p \leqslant n} \frac{\log p}{p - 1} - \log n < 1.141661</math>
  
::::<math>\quad \; = m x_1</math>
+
{{Spoiler|Style = font-style: italic; font-weight: bold; color: olive; text-decoration: underline;|Show=Dowód|Hide=Ukryj dowód}}
 +
Ponieważ
  
 +
::<math>\frac{1}{p} = \frac{1}{p - 1} - \frac{1}{p (p - 1)}</math>
  
::<math>a y - b x = (x - r m) y - (y - s m) x</math>
+
to z&nbsp;twierdzenia D35 dostajemy
  
::::<math>\;\;\, = x y - r m y - y x + s m x</math>
+
::<math>\sum_{p \leqslant n} \frac{\log p}{p - 1} - \sum_{p \leqslant n} \frac{\log p}{p (p - 1)} - \log n < \log 4 - 1</math>
  
::::<math>\;\;\, = m (s x - r y)</math>
+
Czyli
 
 
::::<math>\;\;\, = m y_1</math>
 
 
 
Gdzie oznaczyliśmy
 
 
 
::<math>x_1 = p - r x - n s y</math>
 
 
 
::<math>y_1 = s x - r y</math>
 
 
 
Wynika stąd, że
 
 
 
::<math>m_1 m^2 p = (m x_1)^2 + n (m y_1)^2</math>
 
 
 
Zatem
 
 
 
::<math>x^2_1 + n y^2_1 = m_1 p</math>
 
 
 
gdzie
 
 
 
::<math>1 \leqslant m_1 < {\small\frac{(n + 1) m}{4}}</math>
 
  
Czyli powtarzając odpowiednią ilość razy opisaną powyżej procedurę, otrzymamy <math>m_k = 1</math>.
+
::<math>\sum_{p \leqslant n} \frac{\log p}{p - 1} - \log n < \log 4 - 1 + \sum_{p \leqslant n} \frac{\log p}{p (p - 1)}</math>
  
 +
:::::::<math>\;\;\: < \log 4 - 1 + \sum_{p \geqslant 2} \frac{\log p}{p (p - 1)}</math>
  
'''D. Jednoznaczność rozkładu'''
+
:::::::<math>\;\;\: = \log 4 - 1 + 0.755366610831 \ldots</math>
  
Z założenia <math>p</math> jest liczbą pierwszą, zatem jednoznaczność rozkładu wynika z&nbsp;twierdzenia K53. Co kończy dowód.<br/>
+
:::::::<math>\;\;\: < 1.141661</math><br/>
 
&#9633;
 
&#9633;
 
{{\Spoiler}}
 
{{\Spoiler}}
Linia 2116: Linia 1213:
  
  
<span style="font-size: 110%; font-weight: bold;">Uwaga K57</span><br/>
+
<span style="font-size: 110%; font-weight: bold;">Uwaga D37</span><br/>
Udowodnione wyżej twierdzenie można wykorzystać do znalezienia rozkładu liczby pierwszej <math>p</math> postaci <math>4 k + 1</math> na sumę dwóch kwadratów. Dla dużych liczb pierwszych funkcja działa wolno, bo dużo czasu zajmuje obliczanie silni.
+
{| class="wikitable"
 +
|
 +
Dokładniejsze oszacowanie sumy <math>\sum_{p \leqslant n} \frac{\log p}{p}</math> jest dane wzorem
  
{{Spoiler|Style = font-style: italic; font-weight: bold; color: olive; text-decoration: underline;|Show=Pokaż kod|Hide=Ukryj kod}}
+
::<math>\sum_{p \leqslant n} \frac{\log p}{p} = \log n - E + \ldots</math>
<span style="font-size: 90%; color:black;">SumOfTwoSquares(p) =
 
{
 
'''local'''(m, r, s, x, y, x1, y1);
 
'''if'''( p%4 <> 1 || !'''isprime'''(p), '''return'''("Error") );
 
x = 1;
 
'''for'''(k = 2, (p-1)/2, x = (x*k)%p); \\ x = { [(p-1)/2]! } % p
 
x = x - '''round'''(x/p)*p;
 
y = 1;
 
m = (x^2 + y^2)/p;
 
'''while'''( m > 1,
 
        r = '''round'''(x/m);
 
        s = '''round'''(y/m);
 
        x1 = p - r*x - s*y;
 
        y1 = r*y - s*x;
 
        x = x1;
 
        y = y1;
 
        m = (x^2 + y^2)/p;
 
      );
 
'''return'''([ '''abs'''(x), '''abs'''(y), p ]);
 
}</span>
 
{{\Spoiler}}
 
  
 +
gdzie <math>E = 1.332582275733 \ldots</math>
  
 +
Dla <math>n \geqslant 319</math> mamy też<ref name="Rosser1"/>
  
<span style="font-size: 110%; font-weight: bold;">Zadanie K58</span><br/>
+
::<math>\left| \sum_{p \leqslant n} \frac{\log p}{p} - \log n + E \right| < \frac{1}{2 \log n}</math>
Niech liczby pierwsze <math>p, q</math> będą postaci <math>4 k + 1</math>, a&nbsp;liczba pierwsza <math>r</math>
 
będzie postaci <math>4 k + 3</math>. Pokazać, że
 
:*&nbsp;&nbsp;liczby <math>r, p r \,</math> i <math>\, r^2</math> nie rozkładają się na sumę dwóch kwadratów liczb całkowitych dodatnich
 
:*&nbsp;&nbsp;liczby <math>p</math>, <math>2 p</math>, <math>p^2 \,</math> i <math>\, p r^2</math> mają jeden rozkład na sumę dwóch kwadratów liczb całkowitych dodatnich
 
:*&nbsp;&nbsp;liczba <math>p q</math>, <math>p \neq q</math> ma dwa rozkłady na sumę dwóch kwadratów liczb całkowitych dodatnich
 
  
{{Spoiler|Style = font-style: italic; font-weight: bold; color: olive; text-decoration: underline;|Show=Rozwiązanie|Hide=Ukryj rozwiązanie}}
+
|}
  
'''Punkt 1.'''
 
  
Ponieważ liczby <math>r \,</math> i <math>\, p r</math> są postaci <math>4 k + 3</math>, to modulo <math>4</math> mamy
 
  
::<math>r, p r \equiv 3 \!\! \pmod{4}</math>
+
<span style="font-size: 110%; font-weight: bold;">Uwaga D38</span><br/>
 +
{| class="wikitable"
 +
|
 +
Dokładniejsze oszacowanie sumy <math>\sum_{p \leqslant n} \frac{\log p}{p - 1}</math> jest dane wzorem
  
Suma <math>x^2 + y^2</math> musi być liczbą nieparzystą, zatem liczby <math>x, y</math> muszą mieć przeciwną parzystość i&nbsp;modulo <math>4</math> mamy
+
::<math>\sum_{p \leqslant n} \frac{\log p}{p - 1} = \log n - \gamma + \ldots</math>
  
::<math>x^2 + y^2 \equiv 1 \!\! \pmod{4}</math>
+
gdzie <math>\gamma = 0.5772156649 \ldots</math> jest stałą Eulera.
  
Przypuśćmy, że
+
Dla <math>n \geqslant 318</math> prawdziwe jest oszacowanie<ref name="twierdzenie"/>
  
::<math>r^2 = x^2 + y^2</math>
+
::<math>\left| \sum_{p \leqslant n} \frac{\log p}{p - 1} - \log n + \gamma \right| < \frac{1}{2 \log n}</math>
  
gdzie <math>x, y \in \mathbb{Z}_+</math>. Liczby <math>x, y</math> muszą mieć przeciwną parzystość, zatem <math>x \neq y</math>. Z&nbsp;twierdzenia J24 wynika, że liczba <math>x^2 + y^2</math> musi mieć dzielnik pierwszy postaci <math>4 k + 1</math>, co w&nbsp;sposób oczywisty jest niemożliwe.
+
|}
  
'''Punkt 2.'''
 
  
W przypadku liczby pierwszej <math>p</math> odpowiedzi udziela twierdzenie K56. Niech <math>p = x^2 + y^2</math>, mamy
 
  
::<math>2 p = (x + y)^2 + (x - y)^2</math>
+
<span style="font-size: 110%; font-weight: bold;">Uwaga D39</span><br/>
 +
Dla <math>n \leqslant 10^{10}</math> wartości wyrażeń
  
::<math>p^2 = (x^2 - y^2)^2 + (2 x y)^2</math>
+
::<math>\sum_{p \leqslant n} \frac{\log p}{p} - \log n + E</math>
  
::<math>p r^2 = (r x)^2 + (r y)^2</math>
+
::<math>\sum_{p \leqslant n} \frac{\log p}{p - 1} - \log n + \gamma</math>
  
'''Punkt 3.'''
+
są liczbami dodatnimi.
  
Niech <math>p = x^2 + y^2</math> i <math>q = a^2 + b^2</math>. Ze wzorów podanych w&nbsp;uwadze K54 mamy
 
  
::<math>p q = (a^2 + b^2) (x^2 + y^2) = (a x + b y)^2 + (a y - b x)^2</math>
 
  
:::::::::<math>\:\, = (a x - b y)^2 + (a y + b x)^2</math>
+
<span style="font-size: 110%; font-weight: bold;">Twierdzenie D40</span><br/>
 +
Prawdziwy jest następujący związek
  
Co należało pokazać.<br/>
+
::<math>\sum_{p \geqslant 2} \frac{\log p}{p (p - 1)} = \sum_{n = 2}^{\infty} \left( \sum_{p \geqslant 2} \frac{\log p}{p^n} \right) = E - \gamma</math>
&#9633;
 
{{\Spoiler}}
 
  
 +
gdzie
  
 +
* <math>\quad \gamma = 0.577215664901532 \ldots</math> jest stałą Eulera<ref name="A001620"/>
 +
* <math>\quad E = 1.332582275733220 \ldots</math><ref name="A083343"/>
 +
* <math>\quad E - \gamma = 0.755366610831688 \ldots</math><ref name="A138312"/>
  
 +
{{Spoiler|Style = font-style: italic; font-weight: bold; color: olive; text-decoration: underline;|Show=Dowód|Hide=Ukryj dowód}}
 +
Ponieważ
  
 +
::<math>\frac{1}{p (p - 1)} = \frac{1}{p - 1} - \frac{1}{p}</math>
  
== Twierdzenia o&nbsp;istnieniu liczb pierwszych kwadratowych i&nbsp;niekwadratowych modulo ==
+
zatem
  
<span style="font-size: 110%; font-weight: bold;">Zadanie K59</span><br/>
+
::<math>\sum_{p \leqslant n} \frac{\log p}{p (p - 1)} = \sum_{p \leqslant n} \frac{\log p}{p - 1} - \sum_{p \leqslant n} \frac{\log p}{p} = (\log n - \gamma + \ldots) - (\log n - E + \ldots)</math>
Niech <math>s = \pm 1</math>. Zbadać podzielność liczby <math>p - s a^2</math>
 
  
:* przez <math>4</math>, gdy <math>p = 4 k + r</math>, gdzie <math>r = 1, 3</math>
+
Przechodząc z <math>n</math> do nieskończoności, otrzymujemy
:* przez <math>8</math>, gdy <math>p = 8 k + r</math>, gdzie <math>r = 1, 3, 5, 7</math>
 
  
{{Spoiler|Style = font-style: italic; font-weight: bold; color: olive; text-decoration: underline;|Show=Rozwiązanie|Hide=Ukryj rozwiązanie}}
+
::<math>\sum_{p \geqslant 2} \frac{\log p}{p (p - 1)} = E - \gamma</math>
Problem sprowadza się do uzyskania odpowiedzi, kiedy kongruencja
 
  
::<math>p - s a^2 \equiv 0 \pmod{2^n}</math>
 
  
gdzie <math>n = 2, 3</math>, ma rozwiązanie. Podstawiając, dostajemy
+
Zauważmy teraz, że
  
::<math>2^n k + r \equiv s a^2 \pmod{2^n}</math>
+
::<math>\frac{1}{p - 1} = \frac{1}{p} \cdot \frac{1}{1 - \frac{1}{p}} =</math>
  
::<math>s a^2 \equiv r \pmod{2^n}</math>
+
::::<math>\;\! = \frac{1}{p} \cdot \left( 1 + \frac{1}{p} + \frac{1}{p^2} + \frac{1}{p^3} + \ldots + \frac{1}{p^k} + \ldots \right) =</math>
  
::<math>a^2 \equiv s r \pmod{2^n}</math>
+
::::<math>\;\! = \frac{1}{p} + \frac{1}{p^2} + \frac{1}{p^3} + \ldots + \frac{1}{p^k} + \ldots</math>
  
Z twierdzenia J54 wiemy, że aby powyższa kongruencja miała rozwiązanie, musi być <math>2^n \mid (s r - 1)</math>, co jest możliwe tylko, gdy
+
Zatem
 
 
::<math>s =
 
\begin{cases}
 
\;\;\: 1 & \text{gdy } r = 1 \\
 
      - 1 & \text{gdy } r = 3 \\
 
\end{cases}</math>
 
 
 
dla <math>2^n = 4</math> i&nbsp;gdy
 
 
 
::<math>s =
 
\begin{cases}
 
\;\;\: 1 & \text{gdy } r = 1 \\
 
      - 1 & \text{gdy } r = 7 \\
 
\end{cases}</math>
 
  
dla <math>2^n = 8</math>. Dla <math>2^n = 8</math> i <math>r = 3, 5</math> rozpatrywana kongruencja nie ma rozwiązania.<br/>
+
::<math>\sum_{p \geqslant 2} \frac{\log p}{p (p - 1)} = \sum_{p \geqslant 2} \frac{\log p}{p} \cdot \left( \frac{1}{p} + \frac{1}{p^2} + \frac{1}{p^3} + \ldots + \frac{1}{p^k} + \ldots \right) = \sum_{n = 2}^{\infty} \left( \sum_{p \geqslant 2} \frac{\log p}{p^n} \right)</math><br/>
 
&#9633;
 
&#9633;
 
{{\Spoiler}}
 
{{\Spoiler}}
Linia 2238: Linia 1299:
  
  
<span style="font-size: 110%; font-weight: bold;">Uwaga K60</span><br/>
+
<span style="font-size: 110%; font-weight: bold;">Twierdzenie D41</span><br/>
Poniżej udowodnimy trzy twierdzenia dotyczące istnienia liczb pierwszych, które są liczbami kwadratowymi modulo <math>p</math>. Pomysł ujęcia problemu zaczerpnęliśmy z&nbsp;pracy Alexandru Gicy<ref name="Gica1"/>. Zadanie K59 należy traktować jako uzupełnienie do dowodu twierdzenia K61. Z&nbsp;zadania łatwo widzimy, że powiązanie liczby <math>s</math> z&nbsp;postacią liczby pierwszej <math>p</math> nie jest przypadkowe.
+
Dla <math>n \geqslant 318</math> prawdziwe jest oszacowanie
 
 
Zauważmy, że twierdzenia ograniczają się do liczb pierwszych <math>p</math>, ponieważ dla liczb złożonych nieparzystych <math>m > 0</math> wynik <math>\left( {\small\frac{q}{m}} \right)_{\small{\!\! J}} = 1</math> nie oznacza, że liczba pierwsza <math>q</math> jest liczbą kwadratową modulo <math>m</math>.
 
  
W tabeli przedstawiamy najmniejsze liczby pierwsze <math>q</math> postaci <math>4 k + 1</math> kwadratowe modulo <math>p</math>.
+
::<math>\left| \sum_{p \leqslant n} \frac{\log p}{p - 1} - \log n + \gamma \right| < \frac{1}{2 \log n}</math>
 
 
::{| class="wikitable plainlinks"  style="font-size: 90%; text-align: center; margin-right: auto;"
 
|-
 
! <math>\boldsymbol{p}</math>
 
| <math>3</math> || <math>5</math> || <math>7</math> || <math>11</math> || <math>13</math> || <math>17</math> || <math>19</math> || <math>23</math> || <math>29</math> || <math>31</math> || <math>37</math> || <math>41</math> || <math>43</math> || <math>47</math> || <math>53</math> || <math>59</math> || <math>61</math> || <math>67</math> || <math>71</math> || <math>73</math> || <math>79</math> || <math>83</math> || <math>89</math> || <math>97</math>
 
|-
 
! <math>\boldsymbol{q}</math>
 
| style="background-color: red" | <math>13</math> || style="background-color: red" | <math>29</math> || style="background-color: red" | <math>29</math> || <math>5</math> || style="background-color: red" | <math>17</math> || <math>13</math> || <math>5</math> || <math>13</math> || <math>5</math> || <math>5</math> || style="background-color: red" | <math>41</math> || <math>5</math> || <math>13</math> || <math>17</math> || <math>13</math> || <math>5</math> || <math>5</math> || <math>17</math> || <math>5</math> || <math>37</math> || <math>5</math> || <math>17</math> || <math>5</math> || <math>53</math>
 
|}
 
 
 
 
 
W kolejnej tabeli przedstawiamy najmniejsze liczby pierwsze <math>q</math> postaci <math>4 k + 3</math> kwadratowe modulo <math>p</math>.
 
 
 
::{| class="wikitable plainlinks"  style="font-size: 90%; text-align: center; margin-right: auto;"
 
|-
 
! <math>\boldsymbol{p}</math>
 
| <math>3</math> || <math>5</math> || <math>7</math> || <math>11</math> || <math>13</math> || <math>17</math> || <math>19</math> || <math>23</math> || <math>29</math> || <math>31</math> || <math>37</math> || <math>41</math> || <math>43</math> || <math>47</math> || <math>53</math> || <math>59</math> || <math>61</math> || <math>67</math> || <math>71</math> || <math>73</math> || <math>79</math> || <math>83</math> || <math>89</math> || <math>97</math>
 
|-
 
! <math>\boldsymbol{q}</math>
 
| style="background-color: red" | <math>7</math> || style="background-color: red" | <math>11</math> || style="background-color: red" | <math>11</math> || <math>3</math> || <math>3</math> || style="background-color: red" | <math>19</math> || <math>7</math> || <math>3</math> || <math>7</math> || <math>7</math> || <math>3</math> || <math>23</math> || <math>11</math> || <math>3</math> || <math>7</math> || <math>3</math> || <math>3</math> || <math>19</math> || <math>3</math> || <math>3</math> || <math>11</math> || <math>3</math> || <math>11</math> || <math>3</math>
 
|}
 
 
 
 
 
 
 
<span style="font-size: 110%; font-weight: bold;">Twierdzenie K61</span><br/>
 
Jeżeli <math>p \geqslant 11</math> jest liczbą pierwszą i <math>p \neq 17</math>, to istnieje liczba pierwsza <math>q < p</math> postaci <math>4 k + 3</math> kwadratowa modulo <math>p</math>.
 
  
 
{{Spoiler|Style = font-style: italic; font-weight: bold; color: olive; text-decoration: underline;|Show=Dowód|Hide=Ukryj dowód}}
 
{{Spoiler|Style = font-style: italic; font-weight: bold; color: olive; text-decoration: underline;|Show=Dowód|Hide=Ukryj dowód}}
Niech
+
Należy zauważyć, że tak dokładnego oszacowania nie można udowodnić metodami elementarnymi, dlatego punktem wyjścia jest oszacowanie podane w&nbsp;pracy Pierre'a Dusarta<ref name="Dusart1"/>
::<math>s =
 
\begin{cases}
 
\;\;\: 1 & \text{gdy } \, p \, \text{ jest postaci } \, 4 k + 1 \\
 
      - 1 & \text{gdy } \, p \, \text{ jest postaci } \, 4 k + 3 \\
 
\end{cases}</math>
 
  
Dla ustalonych liczb <math>n</math> i <math>s</math> rozważmy liczbę <math>u(a) = {\small\frac{p - s a^2}{2^n}}</math> taką, że <math>3 \leqslant u (a) < p</math>. Jeżeli liczba ta jest postaci <math>4 k + 3</math>, to ma dzielnik pierwszy <math>q < p</math> postaci <math>4 k + 3</math> (zobacz C21). Zatem możemy napisać <math>u (a) = t q</math>, co oznacza, że
+
::<math>- \left( \frac{0.2}{\log n} + \frac{0.2}{\log^2 n} \right) \; \underset{n \geqslant 2}{<} \; \sum_{p \leqslant n} \frac{\log p}{p} - \log n + E \; \underset{n \geqslant 2974}{<} \; \frac{0.2}{\log n} + \frac{0.2}{\log^2 n}</math>
  
::<math>p - s a^2 = 2^n u (a) = 2^n t q</math>
+
Ponieważ dla <math>x > e^2 \approx 7.389</math> jest <math>1 + \frac{1}{\log x} < 1.5</math>, to dla <math>n \geqslant 8</math> mamy
  
Czyli
+
::<math>\frac{0.2}{\log n} + \frac{0.2}{\log^2 n} = \frac{0.2}{\log n} \left( 1 + \frac{1}{\log n} \right) < \frac{0.3}{\log n}</math>
  
::<math>p \equiv s a^2 \pmod{q}</math>
 
  
i otrzymujemy
+
Zatem wyjściowy układ nierówności możemy zapisać w&nbsp;postaci
  
::<math>\left( {\small\frac{q}{p}} \right)_{\small{\!\! J}} = s \cdot \left( {\small\frac{p}{q}} \right)_{\small{\!\! J}} = s \cdot \left( {\small\frac{s a^2}{q}} \right)_{\small{\!\! J}} = s \cdot \left( {\small\frac{s}{q}} \right)_{\small{\!\! J}} \cdot \left( {\small\frac{a^2}{q}} \right)_{\small{\!\! J}} =s \cdot \left( {\small\frac{s}{q}} \right)_{\small{\!\! J}} = 1</math>
+
::<math>- \frac{0.3}{\log n} \; \underset{n \geqslant 8}{<} \; \sum_{p \leqslant n} \frac{\log p}{p} - \log n + E \; \underset{n \geqslant 2974}{<} \; \frac{0.3}{\log n}</math>
  
Zatem liczba <math>q < p</math> jest liczbą kwadratową modulo <math>p</math>.
 
  
Pomysł dowodu polega na wskazaniu kilku liczb <math>u(a_1), \ldots, u(a_r)</math> takich, że
+
Z tożsamości
  
::<math>3 \leqslant u(a_1) < \ldots < u(a_r) < p</math>
+
::<math>\frac{1}{p} = \frac{1}{p - 1} - \frac{1}{p (p - 1)}</math>
  
z których jedna musi być postaci <math>4 k + 3</math>.
 
  
'''Przypadek pierwszy:''' <math>\boldsymbol{p \equiv 3 \!\! \pmod{8}}</math>
+
wynika natychmiast, że
  
Mamy <math>s = - 1</math> i&nbsp;przyjmujemy <math>n = 2</math>. Rozważmy liczby
+
::<math>- \frac{0.3}{\log n} \; \underset{n \geqslant 8}{<} \; \sum_{p \leqslant n}  \frac{\log p}{p - 1} - \sum_{p \leqslant n} \frac{\log p}{p (p - 1)} - \log n + E \; \underset{n \geqslant 2974}{<} \; \frac{0.3}{\log n}</math>
  
::<math>3 \leqslant {\small\frac{p + 1}{4}} < {\small\frac{p + 9}{4}} < p</math>
 
  
Oszacowania są jednocześnie spełnione dla <math>p \geqslant 11</math>. Z&nbsp;założenia <math>p = 8 k + 3</math>, zatem rozpatrywane liczby to <math>\{ 2 k + 1, 2 k + 3 \}</math>. Ponieważ są to dwie kolejne liczby nieparzyste, to jedna z&nbsp;nich jest postaci <math>4 k + 3</math>.
+
'''Prawa nierówność'''
  
'''Przypadek drugi:''' <math>\boldsymbol{p \equiv 5 \!\! \pmod{8}}</math>
+
Rozważmy prawą nierówność prawdziwą dla <math>n \geqslant 2974</math>
  
Mamy <math>s = + 1</math> i&nbsp;przyjmujemy <math>n = 2</math>. Rozważmy liczby
+
::<math>\sum_{p \leqslant n} \frac{\log p}{p - 1} - \sum_{p \leqslant n} \frac{\log p}{p (p - 1)} - \log n + E < \frac{0.3}{\log n}</math>
  
::<math>3 \leqslant {\small\frac{p - 9}{4}} < {\small\frac{p - 1}{4}} < p</math>
 
  
Oszacowania są jednocześnie spełnione dla <math>p \geqslant 21</math>. Z&nbsp;założenia <math>p = 8 k + 5</math>, zatem rozpatrywane liczby to <math>\{ 2 k - 1, 2 k + 1 \}</math>. Ponieważ są to dwie kolejne liczby nieparzyste, to jedna z&nbsp;nich jest postaci <math>4 k + 3</math>.
+
Z twierdzenia D40 wiemy, że
  
'''Przypadek trzeci:''' <math>\boldsymbol{p \equiv 7 \!\! \pmod{8}}</math>
+
::<math>\sum_{p \geqslant 2} \frac{\log p}{p (p - 1)} - E = - \gamma</math>
  
Mamy <math>s = - 1</math> i&nbsp;przyjmujemy <math>n = 3</math>. Rozważmy liczby
+
Zatem
  
::<math>3 \leqslant {\small\frac{p + 1}{8}} < {\small\frac{p + 9}{8}} < {\small\frac{p + 25}{8}} < {\small\frac{p + 49}{8}} < p</math>
+
::<math>\sum_{p \leqslant n} \frac{\log p}{p - 1} - \log n < \sum_{p \leqslant n} \frac{\log p}{p (p - 1)} - E + \frac{0.3}{\log n}</math>
  
Oszacowania są jednocześnie spełnione dla <math>p \geqslant 23</math>. Z&nbsp;założenia <math>p = 8 k + 7</math>, zatem rozpatrywane liczby to <math>\{ k + 1, k + 2, k + 4, k + 7 \}</math>. Jeżeli <math>k \equiv r \!\! \pmod{4}</math>, to modulo <math>4</math> mamy zbiór <math>\{ r + 1, r + 2, r, r + 3 \}</math>. Zatem jedna z&nbsp;liczb w&nbsp;tym zbiorze jest postaci <math>4 k + 3</math>.
+
:::::::<math>\;\;\: < \sum_{p \geqslant 2} \frac{\log p}{p (p - 1)} - E + \frac{0.3}{\log n}</math>
  
'''Przypadek czwarty:''' <math>\boldsymbol{p \equiv 1 \!\! \pmod{8}}</math>
+
:::::::<math>\;\;\: = - \gamma + \frac{0.3}{\log n}</math>
  
Mamy <math>s = + 1</math> i&nbsp;przyjmujemy <math>n = 3</math>. Rozważmy liczby
+
:::::::<math>\;\;\: < - \gamma + \frac{0.5}{\log n}</math>
  
::<math>3 \leqslant {\small\frac{p - 49}{8}} < {\small\frac{p - 25}{8}} < {\small\frac{p - 9}{8}} < {\small\frac{p - 1}{8}} < p</math>
 
  
Oszacowania są jednocześnie spełnione dla <math>p \geqslant 73</math>. Z&nbsp;założenia <math>p = 8 k + 1</math>, zatem rozpatrywane liczby to <math>\{ k - 6, k - 3, k - 1, k \}</math>. Jeżeli <math>k \equiv r \!\! \pmod{4}</math>, to modulo <math>4</math> mamy zbiór <math>\{ r + 2, r + 1, r + 3, r \}</math>. Zatem jedna z&nbsp;liczb w&nbsp;tym zbiorze jest postaci <math>4 k + 3</math>.
+
Bezpośrednio obliczając, sprawdzamy, że nierówność
  
Pozostaje sprawdzić twierdzenie dla liczb pierwszych <math>p < 73</math>. Co kończy dowód.<br/>
+
::<math>\sum_{p \leqslant n} \frac{\log p}{p - 1} - \log n < - \gamma + \frac{0.5}{\log n}</math>
&#9633;
 
{{\Spoiler}}
 
  
 +
jest prawdziwa dla wszystkich liczb <math>318 \leqslant n \leqslant 3000</math>
  
  
<span style="font-size: 110%; font-weight: bold;">Twierdzenie K62</span><br/>
+
'''Lewa nierówność'''
Jeżeli <math>p \geqslant 11</math> jest liczbą pierwszą postaci <math>8 k + 1</math> lub <math>8 k + 3</math>, to istnieje liczba pierwsza <math>q < p</math> postaci <math>4 k + 1</math> kwadratowa modulo <math>p</math>.
 
  
{{Spoiler|Style = font-style: italic; font-weight: bold; color: olive; text-decoration: underline;|Show=Dowód|Hide=Ukryj dowód}}
+
Rozważmy teraz lewą nierówność prawdziwą dla <math>n \geqslant 8</math>
W przypadku, gdy liczba pierwsza <math>p</math> jest postaci <math>8 k + 1</math> lub <math>8 k + 3</math>, to istnieją takie liczby całkowite dodatnie <math>x, y</math>, że <math>p = x^2 + 2 y^2</math> (zobacz K56). Ponieważ z&nbsp;założenia <math>p \geqslant 11</math>, to musi być <math>x \neq y</math>. Z&nbsp;twierdzenia J24 wynika, że liczba <math>x^2 + y^2</math> ma dzielnik pierwszy <math>q</math> postaci <math>4 k + 1</math>. Łatwo widzimy, że <math>q \leqslant x^2 + y^2 < x^2 + 2 y^2 = p</math>.
 
  
Modulo <math>q</math> możemy napisać
+
::<math>\sum_{p \leqslant n} \frac{\log p}{p - 1} - \sum_{p \leqslant n} \frac{\log p}{p (p - 1)} - \log n + E > - \frac{0.3}{\log n}</math>
  
::<math>x^2 + y^2 \equiv 0 \!\! \pmod{q}</math>
+
Mamy
  
Liczba pierwsza <math>q < p</math> nie może dzielić <math>y</math>, bo mielibyśmy <math>q \mid x</math>, czyli <math>q \mid p</math>, co jest niemożliwe. Rozpatrując równość <math>p = x^2 + 2 y^2</math> modulo <math>q</math>, dostajemy
+
::<math>\sum_{p \leqslant n} \frac{\log p}{p - 1} - \log n > \sum_{p \leqslant n} \frac{\log p}{p (p - 1)} - E - \frac{0.3}{\log n}</math>
 
 
::<math>p \equiv y^2 \!\! \pmod{q}</math>
 
 
 
Wynika stąd natychmiast (zobacz J41 p.9)
 
 
 
::<math>\left( {\small\frac{q}{p}} \right)_{\small{\!\! J}} = \left( {\small\frac{p}{q}} \right)_{\small{\!\! J}} = \left( {\small\frac{y^2}{q}} \right)_{\small{\!\! J}} = 1</math>
 
 
 
Co kończy dowód.<br/>
 
&#9633;
 
{{\Spoiler}}
 
  
 +
:::::::<math>\;\;\, = \sum_{p \geqslant 2} \frac{\log p}{p (p - 1)} - \sum_{p > n} \frac{\log p}{p (p - 1)} - E - \frac{0.3}{\log n}</math>
  
 +
:::::::<math>\;\;\, = - \gamma - \frac{0.3}{\log n} - \sum_{p > n} \frac{\log p}{p (p - 1)}</math>
  
<span style="font-size: 110%; font-weight: bold;">Twierdzenie K63</span><br/>
+
:::::::<math>\;\;\, > - \gamma - \frac{0.3}{\log n} - \sum_{k = n + 1}^{\infty} \frac{\log k}{k (k - 1)}</math>
Jeżeli <math>p \geqslant 19</math> jest liczbą pierwszą postaci <math>12 k + 7</math>, to istnieje liczba pierwsza <math>q < p</math> postaci <math>4 k + 1</math> kwadratowa modulo <math>p</math>.
 
  
{{Spoiler|Style = font-style: italic; font-weight: bold; color: olive; text-decoration: underline;|Show=Dowód|Hide=Ukryj dowód}}
+
:::::::<math>\;\;\, > - \gamma - \frac{0.3}{\log n} - \sum_{k = n + 1}^{\infty} \frac{\log k}{(k - 1)^2}</math>
Z założenia <math>p \equiv 1 \!\! \pmod{6}</math>, zatem istnieją takie liczby <math>x, y \in \mathbb{Z}_+</math>, że <math>p = x^2 + 3 y^2</math> (zobacz K56).
 
Liczby <math>x, y</math> muszą mieć przeciwną parzystość i&nbsp;być względnie pierwsze. Gdyby liczba <math>x</math> była nieparzysta, to modulo <math>4</math> mielibyśmy
 
  
::<math>1 + 3 \cdot 0 \equiv 3 \!\! \pmod{4}</math>
 
  
Co jest niemożliwe. Zatem <math>x = 2 k</math>, a&nbsp;liczba <math>y</math> musi być nieparzysta. Otrzymujemy
+
Korzystając kolejno z&nbsp;twierdzeń D15 i&nbsp;C18, dostajemy
  
::<math>p = 4 k^2 + 3 y^2 = 4 (k^2 + y^2) - y^2</math>
+
::<math>\sum_{p \leqslant n} \frac{\log p}{p - 1} - \log n > - \gamma - \frac{0.3}{\log n} - \int_{n}^{\infty} \frac{\log x}{(x - 1)^2} d x</math>
  
Ponieważ <math>p</math> jest liczbą pierwszą, to jedynie w&nbsp;przypadku gdy <math>k = y = 1</math> możliwa jest sytuacja, że <math>k = y</math>. Mielibyśmy wtedy <math>p = 7</math>, ale z&nbsp;założenia musi być <math>p \geqslant 19</math>. Wynika stąd, że <math>k \neq y</math>, zatem liczba <math>k^2 + y^2</math> ma dzielnik pierwszy <math>q</math> postaci <math>4 k + 1</math> (zobacz J24). Oczywiście <math>q \leqslant k^2 + y^2 < 4 k^2 + 3 y^2 = p</math>.
+
:::::::<math>\;\;\, = - \gamma - \frac{0.3}{\log n} - \frac{\log n}{n - 1} + \log \left( 1 - \frac{1}{n} \right)</math>
  
Modulo <math>q</math> możemy napisać
+
:::::::<math>\;\;\, > - \gamma - \frac{0.3}{\log n} - \frac{\log n}{n - 1} - \frac{1}{n - 1}</math>
  
::<math>k^2 + y^2 \equiv 0 \!\! \pmod{q}</math>
+
:::::::<math>\;\;\, = - \gamma - \frac{0.5}{\log n} + \left( \frac{0.2}{\log n} - \frac{\log n + 1}{n - 1} \right)</math>
  
Liczba pierwsza <math>q</math> nie może dzielić <math>y</math>, bo mielibyśmy <math>q \mid k</math>, czyli <math>q \mid p</math>, co jest niemożliwe. Rozpatrując równość <math>p = 4 (k^2 + y^2) - y^2</math> modulo <math>q</math>, dostajemy
+
:::::::<math>\;\;\, > - \gamma - \frac{0.5}{\log n}</math>
  
::<math>p \equiv - y^2 \!\! \pmod{q}</math>
 
  
Wynika stąd natychmiast (zobacz J41 p.9 i&nbsp;p.6)
+
Do znalezienia całki oznaczonej Czytelnik może wykorzystać stronę [https://www.wolframalpha.com/input?i=int+log%28x%29%2F%28x-1%29%5E2+from+n+to+inf WolframAlpha]. Ostatnia nierówność jest prawdziwa dla <math>n \geqslant 153</math>. Bezpośrednio obliczając, sprawdzamy, że nierówność
  
::<math>\left( {\small\frac{q}{p}} \right)_{\small{\!\! J}}
+
::<math>\sum_{p \leqslant n} \frac{\log p}{p - 1} - \log n > - \gamma - \frac{0.5}{\log n}</math>
= \left( {\small\frac{p}{q}} \right)_{\small{\!\! J}}
 
= \left( {\small\frac{- y^2}{q}} \right)_{\small{\!\! J}}
 
= \left( {\small\frac{- 1}{q}} \right)_{\small{\!\! J}} \cdot \left( {\small\frac{y^2}{q}} \right)_{\small{\!\! J}}
 
= \left( {\small\frac{- 1}{q}} \right)_{\small{\!\! J}} = 1</math>
 
  
Co kończy dowód.<br/>
+
jest prawdziwa dla wszystkich <math>2 \leqslant n \leqslant 200</math>.<br/>
 
&#9633;
 
&#9633;
 
{{\Spoiler}}
 
{{\Spoiler}}
Linia 2398: Linia 1400:
  
  
Twierdzenia K62 i&nbsp;K63 można uogólnić na wszystkie liczby pierwsze.<ref name="Gica1"/><br/>
+
<span style="font-size: 110%; font-weight: bold;">Zadanie D42</span><br/>
<span style="font-size: 110%; font-weight: bold;">Twierdzenie K64*</span><br/>
+
Niech <math>r = 1 - \log (2) \approx 0.30685281944</math>. Pokazać, że z&nbsp;nierówności prawdziwej dla <math>x \geqslant 32</math>
Jeżeli <math>p \geqslant 11</math> jest liczbą pierwszą i <math>p \neq 13, 37</math>, to istnieje liczba pierwsza <math>q < p</math> postaci <math>4 k + 1</math> kwadratowa modulo <math>p</math>.
 
  
 +
::<math>\sum_{p \leqslant x} \frac{\log p}{p - 1} < \log x - r</math>
  
 +
wynika twierdzenie Czebyszewa.
  
<span style="font-size: 110%; font-weight: bold;">Uwaga K65</span><br/>
+
{{Spoiler|Style = font-style: italic; font-weight: bold; color: olive; text-decoration: underline;|Show=Rozwiązanie|Hide=Ukryj rozwiązanie}}
W tabeli przedstawiamy najmniejsze liczby pierwsze <math>q</math> postaci <math>4 k + 1</math> niekwadratowe modulo <math>m</math>.
+
Z twierdzenia D41 wiemy, że dla <math>x \geqslant 318</math> jest
  
:{| class="wikitable plainlinks"  style="font-size: 80%; text-align: center; margin-right: auto;"
+
::<math>\sum_{p \leqslant x} \frac{\log p}{p - 1} - \log x < - \gamma + \frac{1}{2\log x} \leqslant - \gamma + \frac{1}{2 \log (318)} = - 0.490441 \ldots < - 0.306852 \ldots = - r</math>
|-
 
! <math>\boldsymbol{m}</math>
 
| <math>2</math> || <math>3</math> || <math>4</math> || <math>5</math> || <math>6</math> || <math>7</math> || <math>8</math> || <math>9</math> || <math>10</math> || <math>11</math> || <math>12</math> || <math>13</math> || <math>14</math> || <math>15</math> || <math>16</math> || <math>17</math> || <math>18</math> || <math>19</math> || <math>20</math> || <math>21</math> || <math>22</math> || <math>23</math> || <math>24</math> || <math>25</math> || <math>26</math> || <math>27</math> || <math>28</math> || <math>29</math> || <math>30</math> || <math>31</math> || <math>32</math> || <math>33</math> || <math>34</math> || <math>35</math> || <math>36</math> || <math>37</math> || <math>38</math> || <math>39</math> || <math>40</math>
 
|-
 
! <math>\boldsymbol{q}</math>
 
| style="background-color: red" | <math>-</math> || style="background-color: red" | <math>5</math> || style="background-color: red" | <math>-</math> || style="background-color: red" | <math>13</math> || <math>5</math> || <math>5</math> || <math>5</math> || <math>5</math> || style="background-color: red" | <math>13</math> || style="background-color: red" | <math>13</math> || <math>5</math> || <math>5</math> || <math>5</math> || <math>13</math> || <math>5</math> || <math>5</math> || <math>5</math> || <math>13</math> || <math>13</math> || <math>5</math> || <math>13</math> || <math>5</math> || <math>5</math> || <math>13</math> || <math>5</math> || <math>5</math> || <math>5</math> || <math>17</math> || <math>13</math> || <math>13</math> || <math>5</math> || <math>5</math> || <math>5</math> || <math>13</math> || <math>5</math> || <math>5</math> || <math>13</math> || <math>5</math> || <math>13</math>
 
|}
 
  
 +
Zatem postulowane oszacowanie jest prawdziwe dla <math>n \geqslant 318</math>. Sprawdzając bezpośrednio dla <math>2 \leqslant x \leqslant 317</math>, łatwo potwierdzamy prawdziwość nierówności
  
W kolejnej tabeli przedstawiamy najmniejsze liczby pierwsze <math>q</math> postaci <math>4 k + 3</math> niekwadratowe modulo <math>m</math>.
+
::<math>\sum_{p \leqslant x} \frac{\log p}{p - 1} < \log x - r</math>
  
:{| class="wikitable plainlinks"  style="font-size: 80%; text-align: center; margin-right: auto;"
+
dla <math>x \geqslant 32</math>.
|-
 
! <math>\boldsymbol{m}</math>
 
| <math>2</math> || <math>3</math> || <math>4</math> || <math>5</math> || <math>6</math> || <math>7</math> || <math>8</math> || <math>9</math> || <math>10</math> || <math>11</math> || <math>12</math> || <math>13</math> || <math>14</math> || <math>15</math> || <math>16</math> || <math>17</math> || <math>18</math> || <math>19</math> || <math>20</math> || <math>21</math> || <math>22</math> || <math>23</math> || <math>24</math> || <math>25</math> || <math>26</math> || <math>27</math> || <math>28</math> || <math>29</math> || <math>30</math> || <math>31</math> || <math>32</math> || <math>33</math> || <math>34</math> || <math>35</math> || <math>36</math> || <math>37</math> || <math>38</math> || <math>39</math> || <math>40</math>
 
|-
 
! <math>\boldsymbol{q}</math>
 
| style="background-color: red" | <math>-</math> || style="background-color: red" | <math>11</math> || <math>3</math> || <math>3</math> || style="background-color: red" | <math>11</math> || <math>3</math> || <math>3</math> || style="background-color: red" | <math>11</math> || <math>3</math> || <math>7</math> || <math>7</math> || <math>7</math> || <math>3</math> || <math>7</math> || <math>3</math> || <math>3</math> || <math>11</math> || <math>3</math> || <math>3</math> || <math>11</math> || <math>7</math> || <math>7</math> || <math>7</math> || <math>3</math> || <math>7</math> || <math>11</math> || <math>3</math> || <math>3</math> || <math>7</math> || <math>3</math> || <math>3</math> || <math>7</math> || <math>3</math> || <math>3</math> || <math>7</math> || <math>19</math> || <math>3</math> || <math>7</math> || <math>3</math>
 
|}
 
  
  
 +
Niech <math>a \in \mathbb{Z}</math> i <math>a \geqslant 32</math>. Korzystając z&nbsp;twierdzenia D32, łatwo znajdujemy oszacowanie
  
<span style="font-size: 110%; font-weight: bold;">Twierdzenie K66</span><br/>
+
::<math>a! = p^{\alpha_1}_1 \cdot \ldots \cdot p^{\alpha_n}_n</math>
Jeżeli <math>m \geqslant 7</math> jest liczbą całkowitą postaci <math>4 k + 3</math>, to istnieje liczba pierwsza <math>q < m</math> postaci <math>4 k + 3</math> niekwadratowa modulo <math>m</math>.
 
  
{{Spoiler|Style = font-style: italic; font-weight: bold; color: olive; text-decoration: underline;|Show=Dowód|Hide=Ukryj dowód}}
+
::<math>\quad \leqslant p^{(a - 1) / (p_1 - 1)}_1 \cdot \ldots \cdot p^{(a - 1) / (p_n - 1)}_n</math>
Ponieważ liczba <math>m - 4 \geqslant 3</math> jest postaci <math>4 k + 3</math>, to ma dzielnik pierwszy <math>q < m</math> postaci <math>4 k + 3</math> (zobacz C21). Czyli <math>m - 4 = k q</math> i&nbsp;z&nbsp;twierdzenia J41 p.9 dostajemy
 
  
::<math>\left( {\small\frac{q}{m}} \right)_{\small{\!\! J}} =
+
::<math>\quad = (p^{1 / (p_1 - 1)}_1 \cdot \ldots \cdot p^{1 / (p_n - 1)}_n)^{a - 1}</math>
- \left( {\small\frac{m}{q}} \right)_{\small{\!\! J}} =
 
- \left( {\small\frac{k q + 4}{q}} \right)_{\small{\!\! J}} =
 
- \left( {\small\frac{4}{q}} \right)_{\small{\!\! J}} = - 1</math>
 
  
Zatem <math>q</math> jest liczbą niekwadratową modulo <math>m</math>. Co należało pokazać.<br/>
+
gdzie <math>p_n \leqslant a < p_{n + 1}</math>. Oznaczając wyrażenie w&nbsp;nawiasie przez <math>U</math>, mamy
&#9633;
 
{{\Spoiler}}
 
  
 +
::<math>\log U = \frac{\log p_1}{p_1 - 1} + \ldots + \frac{\log p_n}{p_n - 1} = \sum_{p \leqslant a} \frac{\log p}{p - 1} < \log a - r</math>
  
 +
gdzie skorzystaliśmy z&nbsp;oszacowania wskazanego w&nbsp;treści zadania. Zatem <math>U < a \cdot e^{- r}</math>.
  
Można też pokazać, że<ref name="Pollack2"/><br/>
 
<span style="font-size: 110%; font-weight: bold;">Twierdzenie K67*</span><br/>
 
'''A.''' Jeżeli <math>p \geqslant 13</math> jest liczbą pierwszą, to istnieje liczba pierwsza <math>q < p</math> postaci <math>4 k + 1</math> niekwadratowa modulo <math>p</math>.
 
  
'''B.''' Jeżeli <math>p \geqslant 5</math> jest liczbą pierwszą, to istnieje liczba pierwsza <math>q < p</math> postaci <math>4 k + 3</math> niekwadratowa modulo <math>p</math>.
+
Przypuśćmy, że mnożymy liczbę <math>a!</math> przez kolejne liczby naturalne <math>a + 1, a + 2, \ldots, b - 1, b</math>. Możemy postawić pytanie: kiedy w&nbsp;rozkładzie na czynniki pierwsze liczby <math>b!</math> musi pojawić się nowy czynnik pierwszy? Jeżeli takiego nowego czynnika pierwszego nie ma, to
  
 +
::<math>a! \cdot (a + 1) \cdot \ldots \cdot b = b!</math>
  
 +
:::::::<math>\;\;\; = p^{\beta_1}_1 \cdot \ldots \cdot p^{\beta_n}_n</math>
  
Zauważmy, że twierdzenie K67 można łatwo uogólnić na liczby całkowite dodatnie.<br/>
+
:::::::<math>\;\;\; \leqslant p^{(b - 1) / (p_1 - 1)}_1 \cdot \ldots \cdot p^{(b - 1) / (p_n - 1)}_n</math>
<span style="font-size: 110%; font-weight: bold;">Twierdzenie K68</span><br/>
 
'''A.''' Jeżeli <math>m \geqslant 6</math> jest liczbą całkowitą i <math>m \neq 10 , 11</math>, to istnieje liczba pierwsza <math>q < m</math> postaci <math>4 k + 1</math> niekwadratowa modulo <math>m</math>.
 
  
'''B.''' Jeżeli <math>m \geqslant 4</math> jest liczbą całkowitą i <math>m \neq 6 , 9</math>, to istnieje liczba pierwsza <math>q < m</math> postaci <math>4 k + 3</math> niekwadratowa modulo <math>m</math>.
+
:::::::<math>\;\;\; = (p^{1 / (p_1 - 1)}_1 \cdot \ldots \cdot p^{1 / (p_n - 1)}_n)^{b - 1}</math>
  
{{Spoiler|Style = font-style: italic; font-weight: bold; color: olive; text-decoration: underline;|Show=Dowód|Hide=Ukryj dowód}}
+
:::::::<math>\;\;\; = U^{b - 1}</math>
  
'''Punkt B'''
+
:::::::<math>\;\;\; < (a \cdot e^{- r})^{b - 1}</math>
  
Rozważmy liczby <math>m</math> postaci <math>m = 2^a 3^b</math>.
 
  
Jeżeli <math>3 \mid m</math>, to <math>11</math> jest liczbą niekwadratową modulo <math>m</math>, bo <math>\left( {\small\frac{11}{3}} \right)_{\small{\!\! J}} = - 1</math> (zobacz J55 i&nbsp;K41).
+
Jednocześnie z&nbsp;twierdzenia D31 wiemy, że prawdziwa jest nierówność <math>b! > b^b e^{- b}</math>, zatem
  
Jeżeli <math>3 \nmid m</math>, ale <math>8 \mid m</math>, to <math>8 \nmid (11 - 1)</math>, zatem liczba <math>11</math> jest liczbą niekwadratową modulo <math>m</math> (zobacz J55).
+
::<math>b^b e^{- b} < b! < \frac{(a \cdot e^{- r})^b}{a \cdot e^{-r}}</math>
  
Jeżeli <math>3 \nmid m</math> i <math>8 \nmid m</math>, ale <math>4 \mid m</math>, to <math>4 \nmid (11 - 1)</math>, zatem liczba <math>11</math> jest liczbą niekwadratową modulo <math>m</math> (zobacz J55).
+
::<math>b e^{- 1} < \frac{a \cdot e^{- r}}{(a \cdot e^{- r})^{1 / b}}</math>
  
Jeżeli <math>m = 2</math>, to łatwo zauważamy, że nie istnieją liczby niekwadratowe modulo <math>2</math>.
+
::<math>b < \frac{a \cdot e^{1 - r}}{(a \cdot e^{- r})^{1 / b}}</math>
  
  
Zbierając:
+
Ponieważ <math>e^{1 - r} = e^{\log (2)} = 2</math>, to
  
:* jeśli liczba <math>m \geqslant 12</math> nie ma dzielnika pierwszego <math>p \geqslant 5</math>, czyli jest postaci <math>m = 2^a 3^b</math>, to liczba pierwsza <math>q = 11</math> jest mniejsza od <math>m</math>, jest postaci <math>4 k + 3</math> i&nbsp;jest liczbą niekwadratową modulo <math>m</math>.
+
::<math>b < \frac{2 a}{(a \cdot e^{- r})^{1 / b}} < 2 a</math>
:* jeśli liczba <math>m \geqslant 12</math> ma dzielnik pierwszy <math>p \geqslant 5</math>, to istnieje liczba pierwsza <math>q < p \leqslant m</math> taka, że <math>q</math> jest postaci <math>4 k + 3</math> i&nbsp;jest liczbą niekwadratową modulo <math>m</math> (zobacz K67 i&nbsp;K41).
 
  
  
Pozostaje wypisać dla liczb <math>3 \leqslant m \leqslant 11</math> najmniejsze liczby niekwadratowe, które są liczbami pierwszymi postaci <math>4 k + 3</math>.
+
Z oszacowania <math>b < 2 a</math> wynika, że <math>(a \cdot e^{- r})^{1 / b} > (a \cdot e^{-r})^{1 / 2 a}</math>. Możemy teraz zapisać uzyskane wyżej oszacowanie w&nbsp;postaci, w&nbsp;której prawa strona nierówności nie zależy od <math>b</math>
  
<span style="font-size: 90%; color:black;">'''for'''(m = 3, 15, '''forprimestep'''(q = 3, 100, 4, '''if'''( isQR(q,m) == -1, '''print'''(m, "  ", q); '''break'''() )))</span>
+
::<math>b < \frac{2 a}{(a \cdot e^{- r})^{1 / b}} < \frac{2 a}{(a \cdot e^{- r})^{1 / 2 a}}</math>
  
::{| class="wikitable plainlinks"  style="font-size: 90%; text-align: center; margin-right: auto;"
 
|-
 
! <math>\boldsymbol{m}</math>
 
| <math>3</math> || <math>4</math> || <math>5</math> || <math>6</math> || <math>7</math> || <math>8</math> || <math>9</math> || <math>10</math> || <math>11</math> || <math>12</math> || <math>13</math> || <math>14</math> || <math>15</math>
 
|-
 
! <math>\boldsymbol{q}</math>
 
| style="background-color: red" | <math>11</math> || <math>3</math> || <math>3</math> || style="background-color: red" | <math>11</math> || <math>3</math> || <math>3</math> || style="background-color: red" | <math>11</math> || <math>3</math> || <math>7</math> || <math>7</math> || <math>7</math> || <math>3</math> || <math>7</math>
 
|}
 
  
Widzimy, że twierdzenie jest prawdziwe dla <math>m \geqslant 4</math>, o ile <math>m \neq 6 , 9</math>.
+
Ponieważ <math>e^{- r} = 0.735758 \ldots</math>, to <math>(a \cdot e^{- r})^{1 / 2 a} > (a / 2)^{1 / 2 a}</math>, co pozwala uprościć uzyskane oszacowanie
  
'''Punkt A'''
+
::<math>b < \frac{2 a}{(a \cdot e^{- r})^{1 / 2 a}} < \frac{2 a}{(a / 2)^{1 / 2 a}}</math>
  
Rozważmy liczby <math>m</math> postaci <math>m = 2^a 3^b 5^c 7^d 11^e</math>.
 
  
Jeżeli jedna z&nbsp;liczb <math>3, 5, 7, 11</math> dzieli <math>m</math>, to <math>17</math> jest liczbą niekwadratową modulo <math>m</math>, bo
+
Pokażemy, że dla <math>a > 303.05</math>
<math>\left( {\small\frac{17}{3}} \right)_{\small{\!\! J}}
 
= \left( {\small\frac{17}{5}} \right)_{\small{\!\! J}}
 
= \left( {\small\frac{17}{7}} \right)_{\small{\!\! J}}
 
= \left( {\small\frac{17}{11}} \right)_{\small{\!\! J}}
 
= - 1</math>.
 
  
Jeżeli żadna z&nbsp;liczb <math>3, 5, 7, 11</math> nie dzieli <math>m</math>, ale <math>8 \mid m</math>, to <math>8 \nmid (5 - 1)</math>, zatem liczba <math>5</math> jest liczbą niekwadratową modulo <math>m</math>.
+
::<math>\frac{2 a}{(a / 2)^{1 / 2 a}} < 2 a - 5</math>
  
Jeżeli żadna z&nbsp;liczb <math>3, 5, 7, 11</math> nie dzieli <math>m</math> i <math>8 \nmid m</math>, ale <math>4 \mid m</math>, to nie istnieją liczby pierwsze postaci <math>4 k + 1</math> niekwadratowe modulo <math>m</math>, bo <math>4 \mid [(4 k + 1) - 1]</math>
+
Istotnie
  
Jeżeli <math>m = 2</math>, to łatwo zauważamy, że nie istnieją liczby niekwadratowe modulo <math>2</math>.
+
::<math>\frac{1}{(a / 2)^{1 / 2 a}} < 1 - \frac{5}{2 a}</math>
  
Zbierając:
+
::<math>\frac{a}{2} \cdot \left( 1 - \frac{5}{2 a} \right)^{2 a} > 1</math>
  
:* jeśli liczba <math>m \geqslant 18</math> nie ma dzielnika pierwszego <math>p \geqslant 13</math>, czyli jest postaci <math>m = 2^a 3^b 5^c 7^d 11^e</math>, to liczba pierwsza <math>q = 5</math> lub <math>q = 17</math> jest mniejsza od <math>m</math>, jest postaci <math>4 k + 1</math> i&nbsp;jest liczbą niekwadratową modulo <math>m</math>.
+
::<math>\frac{a}{2} \cdot \left[ \left( 1 - \frac{5}{2 a} \right)^{\tfrac{2 a}{5}} \right]^5 > 1</math>
:* jeśli liczba <math>m \geqslant 18</math> ma dzielnik pierwszy <math>p \geqslant 13</math>, to istnieje liczba pierwsza <math>q < p \leqslant m</math> taka, że <math>q</math> jest postaci <math>4 k + 1</math> i&nbsp;jest liczbą niekwadratową modulo <math>m</math> (zobacz K67 i&nbsp;K41).
 
  
Pozostaje wypisać dla liczb <math>3 \leqslant m \leqslant 17</math> najmniejsze liczby niekwadratowe, które są liczbami pierwszymi postaci <math>4 k + 1</math>.
+
Wyrażenie w&nbsp;nawiasie kwadratowym jest funkcją rosnącą i&nbsp;ograniczoną (zobacz twierdzenie C17) i&nbsp;dla <math>a \geqslant 32</math> przyjmuje wartości z&nbsp;przedziału <math>[0.353 \ldots, e^{- 1})</math>. Zatem dla odpowiednio dużego <math>a</math> powyższa nierówność z&nbsp;pewnością jest prawdziwa. Łatwo sprawdzamy, że dla <math>a = 304</math> jest
  
<span style="font-size: 90%; color:black;">'''for'''(m = 3, 20, '''forprimestep'''(q = 1, 100, 4, '''if'''( isQR(q,m) == -1, '''print'''(m, "  ", q); '''break'''() )))</span>
+
::<math>\frac{a}{2} \cdot \left( 1 - \frac{5}{2 a} \right)^{2 a} = 1.003213 \ldots</math>
  
::{| class="wikitable plainlinks"  style="font-size: 90%; text-align: center; margin-right: auto;"
+
Wynika stąd, że wszystkie kolejne liczby naturalne <math>a + 1, a + 2, \ldots, b - 1, b</math> mogą być liczbami złożonymi co najwyżej do chwili, gdy <math>b < 2 a -
|-
+
5</math>, czyli <math>b \leqslant 2 a - 6</math>. Zatem w&nbsp;przedziale <math>(a, 2 a)</math> musi znajdować się przynajmniej jedna liczba pierwsza. Dla <math>a \leqslant 303</math> prawdziwość twierdzenia sprawdzamy bezpośrednio.<br/>
! <math>\boldsymbol{m}</math>  
 
| <math>3</math> || <math>4</math> || <math>5</math> || <math>6</math> || <math>7</math> || <math>8</math> || <math>9</math> || <math>10</math> || <math>11</math> || <math>12</math> || <math>13</math> || <math>14</math> || <math>15</math> || <math>16</math> || <math>17</math> || <math>18</math> || <math>19</math> || <math>20</math>
 
|-
 
! <math>\boldsymbol{q}</math>
 
| style="background-color: red" | <math>5</math> || style="background-color: red" | <math>-</math> || style="background-color: red" | <math>13</math> || <math>5</math> || <math>5</math> || <math>5</math> || <math>5</math> || style="background-color: red" | <math>13</math> || style="background-color: red" | <math>13</math> || <math>5</math> || <math>5</math> || <math>5</math> || <math>13</math> || <math>5</math> || <math>5</math> || <math>5</math> || <math>13</math> || <math>13</math>
 
|}
 
 
 
Widzimy, że twierdzenie jest prawdziwe dla <math>m \geqslant 6</math>, o ile <math>m \neq 10 , 11</math>.<br/>
 
 
&#9633;
 
&#9633;
 
{{\Spoiler}}
 
{{\Spoiler}}
Linia 2538: Linia 1496:
  
  
<span style="font-size: 110%; font-weight: bold;">Twierdzenie K69</span><br/>
+
<span style="font-size: 110%; font-weight: bold;">Definicja D43</span><br/>
Jeżeli <math>p \geqslant 5</math> jest liczbą pierwszą, to istnieje liczba pierwsza nieparzysta <math>q < p</math> taka, że <math>\left( {\small\frac{p}{q}} \right)_{\small{\!\! J}} = - 1 .</math>
+
Powiemy, że liczby pierwsze <math>p, q</math> są liczbami bliźniaczymi (tworzą parę liczb bliźniaczych), jeżeli <math>\left | p - q \right | = 2</math>
  
{{Spoiler|Style = font-style: italic; font-weight: bold; color: olive; text-decoration: underline;|Show=Dowód|Hide=Ukryj dowód}}
 
Łatwo sprawdzamy, że
 
  
::<math>\left( {\small\frac{5}{3}} \right)_{\small{\!\! J}} = \left( {\small\frac{7}{5}} \right)_{\small{\!\! J}} = \left( {\small\frac{11}{3}} \right)_{\small{\!\! J}} = - 1</math>
 
  
(zobacz J41&nbsp;p.7). Zatem dowód wystarczy przeprowadzić dla <math>p \geqslant 13</math>.
+
<span style="font-size: 110%; font-weight: bold;">Twierdzenie D44* (Viggo Brun, 1919)</span><br/>
 +
Suma odwrotności par liczb pierwszych <math>p</math> i <math>p + 2</math>, takich że liczba <math>p + 2</math> jest również pierwsza, jest skończona
  
'''A. Liczba pierwsza''' <math>\, \boldsymbol{p} \,</math> '''jest postaci''' <math>\, \boldsymbol{4 k + 1}</math>
+
::<math>\underset{p + 2 \in \mathbb{P}}{\sum_{p \geqslant 2}} \left( \frac{1}{p} + \frac{1}{p + 2} \right) = \left( \frac{1}{3} + \frac{1}{5}
 +
\right) + \left( \frac{1}{5} + \frac{1}{7} \right) + \left( \frac{1}{11} + \frac{1}{13} \right) + \left( \frac{1}{17} + \frac{1}{19} \right) + \ldots = B_2</math>
  
Niech liczba <math>q</math> będzie najmniejszą '''nieparzystą''' liczbą niekwadratową modulo <math>p</math>. Z&nbsp;twierdzenia K25 wiemy, że dla <math>p \geqslant 5</math> liczba <math>q</math> jest liczbą pierwszą i&nbsp;jest mniejsza od <math>p</math>. Ponieważ <math>p \equiv 1 \!\! \pmod{4}</math>, to z&nbsp;twierdzenia J41&nbsp;p.9 otrzymujemy natychmiast
+
gdzie <math>B_2 = 1.90216058 \ldots</math> jest stałą Bruna<ref name="Wiki1"/><ref name="A065421"/>.
  
<div style="margin-top: 1em; margin-bottom: 1em;">
 
::<math>\left( {\small\frac{p}{q}} \right)_{\small{\!\! J}} = \left( {\small\frac{q}{p}} \right)_{\small{\!\! J}} = - 1</math>
 
</div>
 
  
'''B. Liczba pierwsza''' <math>\, \boldsymbol{p} \,</math> '''jest postaci''' <math>\, \boldsymbol{4 k + 3}</math>
 
  
Z twierdzenia K61 wynika, że dla każdej liczby pierwszej <math>p \geqslant 11</math> postaci <math>4 k + 3</math> istnieje liczba pierwsza <math>q < p</math> taka, że <math>q</math> jest postaci <math>4 k + 3</math> i&nbsp;jest liczbą kwadratową modulo <math>p</math>. Ponieważ <math>p \equiv q \equiv 3 \!\! \pmod{4}</math>, to z&nbsp;twierdzenia J41 p.9 otrzymujemy natychmiast
+
<span style="font-size: 110%; font-weight: bold;">Zadanie D45</span><br/>
 +
Pokazać, że istnieje nieskończenie wiele liczb pierwszych nie tworzących par liczb bliźniaczych.
  
<div style="margin-top: 1em; margin-bottom: 1em;">
+
{{Spoiler|Style = font-style: italic; font-weight: bold; color: olive; text-decoration: underline;|Show=Rozwiązanie|Hide=Ukryj rozwiązanie}}
::<math>\left( {\small\frac{p}{q}} \right)_{\small{\!\! J}} = - \left( {\small\frac{q}{p}} \right)_{\small{\!\! J}} = - 1</math>
+
Niech <math>p</math> i <math>q = p + 4</math> będą liczbami pierwszymi i <math>n \geqslant 1</math>. Ponieważ liczby <math>p q</math> i <math>p + 2</math> są względnie pierwsze, to z&nbsp;twierdzenia Dirichleta wiemy, że wśród liczb <math>a_n = p q n + (p + 2)</math> jest nieskończenie wiele liczb pierwszych, a&nbsp;jednocześnie żadna z&nbsp;liczb <math>a_n</math> nie tworzy pary liczb bliźniaczych, bo
</div>
 
  
Co kończy dowód.<br/>
+
::<math>a_n - 2 = p q n + p = p (q n + 1)</math>
&#9633;
 
{{\Spoiler}}
 
  
 +
::<math>a_n + 2 = p q n + (p + 4) = q (p n + 1)</math>
  
 +
są liczbami złożonymi. Najprostsze przykłady to <math>a_n = 21 n + 5</math> i <math>b_n = 77 n + 9</math>
  
<span style="font-size: 110%; font-weight: bold;">Zadanie K70</span><br/>
+
Najłatwiej wszystkie przypadki takich ciągów wyszukać w&nbsp;programie PARI/GP. Polecenie
Udowodnić twierdzenie K69 w&nbsp;przypadku, gdy liczba pierwsza <math>p \geqslant 19</math> jest postaci <math>4 k + 3</math>, nie korzystając z&nbsp;twierdzenia K61.
 
  
{{Spoiler|Style = font-style: italic; font-weight: bold; color: olive; text-decoration: underline;|Show=Rozwiązanie|Hide=Ukryj rozwiązanie}}
+
for(a=1,50, for(b=3,floor(a/2), g=gcd(a,b); g1=gcd(a,b-2); g2=gcd(a,b+2); if( g==1 && g1>1 && g2>1, print("a= ", a, "  b= ",b) )))
Z założenia <math>p = 4 k + 3</math>. Liczba <math>k</math> może być postaci <math>k = 3 j</math>, <math>k = 3 j + 1</math> i <math>k = 3 j + 2</math>. Odpowiada to liczbom pierwszym postaci <math>p = 12 j + 3</math>, <math>p = 12 j + 7</math> i <math>p = 12 j + 11</math>.
 
  
Ponieważ nie ma liczb pierwszych <math>p \geqslant 19</math> i&nbsp;będących postaci <math>p = 12 j + 3</math>, to pozostaje rozważyć przypadki <math>p = 12 j + 7</math> i <math>p = 12 j + 11</math>.
+
wyszukuje wszystkie liczby dodatnie <math>a, b</math>, gdzie <math>b \leqslant \left\lfloor \frac{a}{2} \right\rfloor</math>, które tworzą ciągi <math>a k + b</math> o&nbsp;poszukiwanych właściwościach. Oczywiście ciągi <math>a k + (a - b)</math> również są odpowiednie. Przykładowo dla <math>a \leqslant 50</math> mamy
  
'''A. Liczba pierwsza''' <math>\, \boldsymbol{p} \,</math> '''jest postaci''' <math>\, \boldsymbol{12 j + 11}</math>
+
::<math>15 k + 7, \quad 21 k + 5, \quad 30 k + 7, \quad 33 k + 13, \quad 35 k + 12, \quad 39 k + 11, \quad 42 k + 5, \quad 45 k + 7, \quad 45 k + 8, \quad 45 k + 22</math><br/>
 +
&#9633;
 +
{{\Spoiler}}
  
Wiemy, że w&nbsp;tym przypadku <math>\left( {\small\frac{3}{p}} \right)_{\small{\!\! J}} = + 1</math> (zobacz J46). Mamy
 
  
<div style="margin-top: 1em; margin-bottom: 1em;">
 
::<math>\left( {\small\frac{p}{3}} \right)_{\small{\!\! J}} = - \left( {\small\frac{3}{p}} \right)_{\small{\!\! J}} = - 1</math>
 
</div>
 
  
Czyli wystarczy przyjąć <math>q = 3</math>.
 
  
'''B. Liczba pierwsza''' <math>\, \boldsymbol{p} \,</math> '''jest postaci''' <math>\, \boldsymbol{12 j + 7}</math>
 
  
Wiemy, że w&nbsp;tym przypadku <math>\left( {\small\frac{- 1}{p}} \right)_{\small{\!\! J}} = \left( {\small\frac{3}{p}} \right)_{\small{\!\! J}} = - 1</math> (zobacz J41&nbsp;p.6 oraz J46). Otrzymujemy
 
  
<div style="margin-top: 1em; margin-bottom: 1em;">
 
::<math>\left( {\small\frac{p}{p - 12}} \right)_{\small{\!\! J}} = - \left( {\small\frac{p - 12}{p}} \right)_{\small{\!\! J}} = - \left( {\small\frac{- 12}{p}} \right)_{\small{\!\! J}} = \left[ - \left( {\small\frac{- 1}{p}} \right)_{\small{\!\! J}} \right] \cdot \left( {\small\frac{2^2}{p}} \right)_{\small{\!\! J}} \cdot \left( {\small\frac{3}{p}} \right)_{\small{\!\! J}} = \left( {\small\frac{3}{p}} \right)_{\small{\!\! J}} = -1</math>
 
</div>
 
  
Ponieważ liczba <math>p - 12 \geqslant 7</math> jest nieparzysta, to musi istnieć nieparzysty dzielnik pierwszy <math>q < p</math> liczby <math>p - 12</math> taki, że <math>\left( {\small\frac{p}{q}} \right)_{\small{\!\! J}} = - 1</math>. W&nbsp;przeciwnym razie z&nbsp;twierdzenia J41&nbsp;p.4 mielibyśmy <math>\left( {\small\frac{p}{p - 12}} \right)_{\small{\!\! J}} = 1</math>. Co kończy dowód.<br/>
 
&#9633;
 
{{\Spoiler}}
 
  
  
Linia 2609: Linia 1549:
  
  
 +
== Przypisy ==
 +
<references>
  
 +
<ref name="DirichletEta">Wikipedia, ''Funkcja η'', ([https://pl.wikipedia.org/wiki/Funkcja_%CE%B7 Wiki-pl]), ([https://en.wikipedia.org/wiki/Dirichlet_eta_function Wiki-en])</ref>
  
 +
<ref name="RiemannZeta">Wikipedia, ''Funkcja dzeta Riemanna'', ([https://pl.wikipedia.org/wiki/Funkcja_dzeta_Riemanna Wiki-pl]), ([https://en.wikipedia.org/wiki/Riemann_zeta_function Wiki-en])</ref>
  
 +
<ref name="calkowalnosc1">Twierdzenie: funkcja ciągła w przedziale domkniętym jest całkowalna w tym przedziale.</ref>
  
== Przypisy ==
+
<ref name="calkowalnosc2">W szczególności: funkcja ograniczona i mająca skończoną liczbę punktów nieciągłości w przedziale domkniętym jest w tym przedziale całkowalna.</ref>
  
<references>
+
<ref name="Mertens1">Wikipedia, ''Twierdzenia Mertensa'', ([https://pl.wikipedia.org/wiki/Twierdzenia_Mertensa Wiki-pl]), ([https://en.wikipedia.org/wiki/Mertens%27_theorems Wiki-en])</ref>
  
<ref name="Dukic1">Dušan Đukić, ''Quadratic Congruences'', International Mathematical Olympiad training materials, ([https://imomath.com/index.cgi?page=quadraticCongruencesSumsLegendreSymbols IMOmath.com])</ref>
+
<ref name="Mertens2">Wikipedia, ''Franciszek Mertens'', ([https://pl.wikipedia.org/wiki/Franciszek_Mertens Wiki-pl])</ref>
  
<ref name="Hasse1">Helmut Hasse, ''Zur Theorie der abstrakten elliptischen Funktionenkörper. I. Die Struktur der Gruppe der Divisisorenklassen endlicher Ordnung. II. Automorphismen und Meromorphismen. Das Additionstheorem. III. Die Struktur des Meromorphismenrings. Die Riemannsche Vermutung'', Journal für die reine und angewandte Mathematik 175 (1936) 55–62, 69–88, 193–207.</ref>
+
<ref name="Rosser1">J. B. Rosser and L. Schoenfeld, ''Approximate formulas for some functions of prime numbers'', Illinois J. Math. 6 (1962), 64-94, ([https://projecteuclid.org/journals/illinois-journal-of-mathematics/volume-6/issue-1/Approximate-formulas-for-some-functions-of-prime-numbers/10.1215/ijm/1255631807.full LINK])</ref>
  
<ref name="Hasse2">Wikipedia, ''Hasse's theorem on elliptic curves'', ([https://en.wikipedia.org/wiki/Hasse%27s_theorem_on_elliptic_curves Wiki-en]), ([https://ru.wikipedia.org/wiki/%D0%A2%D0%B5%D0%BE%D1%80%D0%B5%D0%BC%D0%B0_%D0%A5%D0%B0%D1%81%D1%81%D0%B5 Wiki-ru])</ref>
+
<ref name="twierdzenie">Zobacz twierdzenie D41.</ref>
  
<ref name="Manin1">Yu. I. Manin, ''On cubic congruences to a prime modulus'', Izv. Akad. Nauk SSSR Ser. Mat., 1956, Volume 20, Issue 5, 673–678</ref>
+
<ref name="A001620">The On-Line Encyclopedia of Integer Sequences, ''A001620 - Decimal expansion of Euler's constant'', ([https://oeis.org/A001620 A001620])</ref>
  
<ref name="Norton1">Karl K. Norton, ''Numbers with Small Prime Factors, and the Least ''k''th Power Non-Residue'', Memoirs of the American Mathematical Society, No. 106 (1971)</ref>
+
<ref name="A083343">The On-Line Encyclopedia of Integer Sequences, ''A083343 - Decimal expansion of constant  B3 (or B_3) related to the Mertens constant'', ([https://oeis.org/A083343 A083343])</ref>
  
<ref name="Trevino1">Enrique Treviño, ''The least k-th power non-residue'', Journal of Number Theory, Volume 149 (2015)</ref>
+
<ref name="A138312">The On-Line Encyclopedia of Integer Sequences, ''A138312 - Decimal expansion of Mertens's constant minus Euler's constant'', ([https://oeis.org/A138312 A138312])</ref>
  
<ref name="Trevino2">Kevin J. McGown and Enrique Treviño, ''The least quadratic non-residue'', Mexican Mathematicians in the World (2021)</ref>
+
<ref name="Dusart1">P. Dusart, ''Estimates of Some Functions Over Primes without R.H.'', ([https://arxiv.org/abs/1002.0442 LINK])</ref>
  
<ref name="Erdos1">Paul Erdős, ''Számelméleti megjegyzések I'', Afar. Lapok, v. 12 (1961)</ref>
+
<ref name="Wiki1">Wikipedia, ''Stałe Bruna'', ([https://pl.wikipedia.org/wiki/Sta%C5%82e_Bruna Wiki-pl]), ([https://en.wikipedia.org/wiki/Brun%27s_theorem Wiki-en])</ref>
  
<ref name="Pollack1">Paul Pollack, ''The average least quadratic nonresidue modulo <math>m</math> and other variations on a&nbsp;theme of Erdős'', Journal of Number Theory, Vol. 132 (2012), No. 6, pp. 1185-1202.</ref>
+
<ref name="A065421">The On-Line Encyclopedia of Integer Sequences, ''A065421 - Decimal expansion of Viggo Brun's constant B'', ([https://oeis.org/A065421 A065421])</ref>
  
<ref name="InfiniteDescent1">Wikipedia, ''Proof by infinite descent'', ([https://en.wikipedia.org/wiki/Proof_by_infinite_descent Wiki-en])</ref>
+
</references>
 
 
<ref name="Bussey1">W. H. Bussey, ''Fermat's Method of Infinite Descent'', The American Mathematical Monthly, Vol. 25, No. 8 (1918)</ref>
 
  
<ref name="HardyWright1">G. H. Hardy and Edward M. Wright, ''An Introduction to the Theory of Numbers'', New York: Oxford University Press, 5th Edition, zobacz dowód Twierdzenia 366 w&nbsp;sekcji 20.4 na stronie 301.</ref>
 
  
<ref name="Gica1">Alexandru Gica, ''Quadratic Residues of Certain Types'', Rocky Mountain J. Math. 36 (2006), no. 6, 1867-1871.</ref>
 
  
<ref name="Pollack2">Paul Pollack, ''The least prime quadratic nonresidue in a&nbsp;prescribed residue class mod 4'', Journal of Number Theory 187 (2018), 403-414</ref>
 
  
</references>
 
  
  

Wersja z 14:13, 23 lut 2024

07.04.2022



Szeregi nieskończone

Definicja D1
Sumę wszystkich wyrazów ciągu nieskończonego [math]\displaystyle{ (a_n) }[/math]

[math]\displaystyle{ a_1 + a_2 + a_3 + \ldots + a_n + \ldots = \sum_{k = 1}^{\infty} a_k }[/math]

nazywamy szeregiem nieskończonym o wyrazach [math]\displaystyle{ a_n }[/math].


Definicja D2
Ciąg [math]\displaystyle{ S_n = \sum_{k = 1}^{n} a_k }[/math] nazywamy ciągiem sum częściowych szeregu [math]\displaystyle{ \sum_{k = 1}^{\infty} a_k }[/math].


Definicja D3
Szereg [math]\displaystyle{ \sum_{k = 1}^{\infty} a_k }[/math] będziemy nazywali zbieżnym, jeżeli ciąg sum częściowych [math]\displaystyle{ \left ( S_n \right ) }[/math] jest zbieżny.


Twierdzenie D4 (warunek konieczny zbieżności szeregu)
Jeżeli szereg [math]\displaystyle{ \sum_{k = 1}^{\infty} a_k }[/math] jest zbieżny, to [math]\displaystyle{ \lim_{n \to \infty} a_n = 0 }[/math].

Dowód

Niech [math]\displaystyle{ S_n = \sum_{k = 1}^{n} a_k }[/math] będzie ciągiem sum częściowych, wtedy [math]\displaystyle{ a_{n + 1} = S_{n + 1} - S_n }[/math]. Z założenia ciąg [math]\displaystyle{ (S_n) }[/math] jest zbieżny, zatem

[math]\displaystyle{ \lim_{n \to \infty} a_{n + 1} = \lim_{n \to \infty} \left ( S_{n+1} - S_{n} \right ) = \lim_{n \to \infty} S_{n + 1} - \lim_{n \to \infty} S_n = 0 }[/math]


Okazuje się, że bardzo łatwo podać przykład szeregów, dla których warunek [math]\displaystyle{ \lim_{n \to \infty} a_n = 0 }[/math] jest warunkiem wystarczającym. Opisany w poniższym twierdzeniu rodzaj szeregów nazywamy szeregami naprzemiennymi.
Twierdzenie D5 (kryterium Leibniza)
Niech ciąg [math]\displaystyle{ (a_n) }[/math] będzie ciągiem malejącym o wyrazach nieujemnych. Jeżeli

[math]\displaystyle{ \underset{n \rightarrow \infty}{\lim} a_n = 0 }[/math]

to szereg [math]\displaystyle{ \underset{k = 1}{\overset{\infty}{\sum}} (- 1)^{k + 1} \cdot a_k }[/math] jest zbieżny.

Dowód

Grupując wyrazy szeregu po dwa, otrzymujemy sumę częściową postaci

[math]\displaystyle{ S_{2 m} = (a_1 - a_2) + (a_3 - a_4) + \ldots + (a_{2 m - 1} - a_{2 m}) }[/math]

Ponieważ ciąg [math]\displaystyle{ (a_n) }[/math] jest ciągiem malejącym, to każde wyrażenie w nawiasie jest liczbą nieujemną. Z drugiej strony

[math]\displaystyle{ S_{2 m} = a_1 - (a_2 - a_3) - (a_4 - a_5) - \ldots - (a_{2 m - 2} - a_{2 m - 1}) {- a_{2 m}} \lt a_1 }[/math]

Zatem dla każdego [math]\displaystyle{ m }[/math] ciąg sum częściowych [math]\displaystyle{ S_{2 m} }[/math] jest rosnący i ograniczony od góry, skąd na mocy twierdzenia C10 jest zbieżny, czyli

[math]\displaystyle{ \lim_{m \to \infty} S_{2 m} = g }[/math]

Pozostaje zbadać sumy częściowe [math]\displaystyle{ S_{2 m + 1} }[/math]. Rezultat jest natychmiastowy

[math]\displaystyle{ \lim_{m \to \infty} S_{2 m + 1} = \lim_{m \to \infty} (S_{2 m} + a_{2 m + 1}) = \lim_{m \to \infty} S_{2 m} + \lim_{m \to \infty} a_{2 m + 1} = g + 0 = g }[/math]

Co kończy dowód.


Twierdzenie D6
Dla [math]\displaystyle{ s \gt 1 }[/math] prawdziwy jest następujący związek

[math]\displaystyle{ \sum_{k = 1}^{\infty} \frac{(- 1)^{k + 1}}{k^s} = (1 - 2^{1 - s}) \sum_{k = 1}^{\infty} \frac{1}{k^s} }[/math]
Dowód

Zauważmy, że założenie [math]\displaystyle{ s \gt 1 }[/math] zapewnia zbieżność szeregu po prawej stronie. Zapiszmy szereg [math]\displaystyle{ \sum_{k = 1}^{\infty} \frac{1}{k^s} }[/math] w postaci sumy dla [math]\displaystyle{ k }[/math] parzystych i nieparzystych

[math]\displaystyle{ \sum_{k = 1}^{\infty} \frac{1}{k^s} = 1 + \frac{1}{2^s} + \frac{1}{3^s} + \frac{1}{4^s} + \frac{1}{5^s} + \ldots = }[/math]
[math]\displaystyle{ \:\, = \sum_{k = 1}^{\infty} \frac{1}{(2 k - 1)^s} + \sum_{k = 1}^{\infty} \frac{1}{(2 k)^s} = }[/math]
[math]\displaystyle{ \:\, = \sum_{k = 1}^{\infty} \frac{1}{(2 k - 1)^s} + \frac{1}{2^s} \sum_{k = 1}^{\infty} \frac{1}{k^s} }[/math]

Otrzymujemy wzór

[math]\displaystyle{ \sum_{k = 1}^{\infty} \frac{1}{(2 k - 1)^s} = (1 - 2^{- s}) \sum_{k = 1}^{\infty} \frac{1}{k^s} }[/math]


Podobnie rozpiszmy szereg naprzemienny

[math]\displaystyle{ \sum_{k = 1}^{\infty} \frac{(- 1)^{k + 1}}{k^s} = 1 - \frac{1}{2^s} + \frac{1}{3^s} - \frac{1}{4^s} + \frac{1}{5^s} - \ldots = }[/math]
[math]\displaystyle{ = \sum_{k = 1}^{\infty} \frac{1}{(2 k - 1)^s} - \sum_{k = 1}^{\infty} \frac{1}{(2 k)^s} = }[/math]
[math]\displaystyle{ = (1 - 2^{- s}) \sum_{k = 1}^{\infty} \frac{1}{k^s} - \frac{1}{2^s} \sum_{k = 1}^{\infty} \frac{1}{k^s} = }[/math]
[math]\displaystyle{ = (1 - 2^{1 - s}) \sum_{k = 1}^{\infty} \frac{1}{k^s} }[/math]

gdzie skorzystaliśmy ze znalezionego wyżej wzoru dla sumy szeregu [math]\displaystyle{ \sum_{k = 1}^{\infty} \frac{1}{(2 k - 1)^s} }[/math]


Przykład D7
Szeregi niekończone często definiują ważne funkcje. Dobrym przykładem może być funkcja eta Dirichleta[1], którą definiuje szereg naprzemienny

[math]\displaystyle{ \eta (s) = \sum_{k = 1}^{\infty} \frac{(- 1)^{k + 1}}{k^s} }[/math]

lub funkcja dzeta Riemanna[2], którą definiuje inny szereg

[math]\displaystyle{ \zeta (s) = \sum_{k = 1}^{\infty} \frac{1}{k^s} }[/math]

Na podstawie twierdzenia D6 funkcje te są związane wzorem

[math]\displaystyle{ \eta (s) = (1 - 2^{1 - s}) \zeta (s) }[/math]

Dla [math]\displaystyle{ s \in \mathbb{R}_+ }[/math] funkcja eta Dirichleta jest zbieżna. Możemy ją wykorzystać do znajdowania sumy szeregu naprzemiennego [math]\displaystyle{ \sum_{k = 1}^{\infty} \frac{(- 1)^{k + 1}}{k^s} }[/math].


Twierdzenie D8
Niech [math]\displaystyle{ N \in \mathbb{Z}_+ }[/math]. Szeregi [math]\displaystyle{ \sum_{k = 1}^{\infty} a_k }[/math] oraz [math]\displaystyle{ \sum_{k = N}^{\infty} a_k }[/math] są jednocześnie zbieżne lub jednocześnie rozbieżne. W przypadku zbieżności zachodzi związek

[math]\displaystyle{ \sum_{k = 1}^{\infty} a_k = \left ( a_1 + a_2 + \ldots + a_{N - 1} \right ) + \sum_{k = N}^{\infty} a_k }[/math]
Dowód

Niech [math]\displaystyle{ S(n) =\sum_{k = 1}^{n} a_k }[/math] (gdzie [math]\displaystyle{ n \geqslant 1 }[/math]) oznacza sumę częściową pierwszego szeregu, a [math]\displaystyle{ T(n) = \sum_{k = N}^{\infty} a_k }[/math] (gdzie [math]\displaystyle{ n \geqslant N }[/math]) oznacza sumę częściową drugiego szeregu. Dla [math]\displaystyle{ n \geqslant N }[/math] mamy

[math]\displaystyle{ S(n) = (a_1 + a_2 + \ldots + a_{N - 1}) + T (n) }[/math]

Widzimy, że dla [math]\displaystyle{ n }[/math] dążącego do nieskończoności zbieżność (rozbieżność) jednego ciągu implikuje zbieżność (rozbieżność) drugiego.


Twierdzenie D9 (kryterium porównawcze)
Jeżeli istnieje taka liczba całkowita [math]\displaystyle{ N_0 }[/math], że dla każdego [math]\displaystyle{ k \gt N_0 }[/math] jest spełniony warunek

[math]\displaystyle{ 0 \leqslant a_k \leqslant b_k }[/math]

to

  1.    zbieżność szeregu [math]\displaystyle{ \sum_{k = 1}^{\infty} b_k }[/math] pociąga za sobą zbieżność szeregu [math]\displaystyle{ \sum_{k = 1}^{\infty} a_k }[/math]
  2.    rozbieżność szeregu [math]\displaystyle{ \sum_{k = 1}^{\infty} a_k }[/math] pociąga za sobą rozbieżność szeregu [math]\displaystyle{ \sum_{k = 1}^{\infty} b_k }[/math]
Dowód

Dowód przeprowadzimy dla szeregów [math]\displaystyle{ \sum_{k = N_0}^{\infty} a_k }[/math] oraz [math]\displaystyle{ \sum_{k = N_0}^{\infty} b_k }[/math], które są (odpowiednio) jednocześnie zbieżne lub jednocześnie rozbieżne z szeregami [math]\displaystyle{ \sum_{k = 1}^{\infty} a_k }[/math] oraz [math]\displaystyle{ \sum_{k = 1}^{\infty} b_k }[/math].

Punkt 1.
Z założenia szereg [math]\displaystyle{ \sum_{k = N_0}^{\infty} b_k }[/math] jest zbieżny. Niech [math]\displaystyle{ \sum_{k = N_0}^{\infty} b_k = b }[/math], zatem z założonych w twierdzeniu nierówności dostajemy

[math]\displaystyle{ 0 \leqslant \sum_{k = N_0}^{n} a_k \leqslant \sum_{k = N_0}^{n} b_k \leqslant b }[/math]

Zauważmy, że ciąg sum częściowych [math]\displaystyle{ A_n = \sum_{k = N_0}^{n} a_k }[/math] jest ciągiem rosnącym (bo [math]\displaystyle{ a_k \geqslant 0 }[/math]) i ograniczonym od góry. Wynika stąd, że ciąg [math]\displaystyle{ \left ( A_n \right ) }[/math] jest zbieżny, zatem szereg [math]\displaystyle{ \sum_{k = N_0}^{\infty} a_k }[/math] jest zbieżny.

Punkt 2.
Z założenia szereg [math]\displaystyle{ \sum_{k = N_0}^{\infty} a_k }[/math] jest rozbieżny, a z założonych w twierdzeniu nierówności dostajemy

[math]\displaystyle{ 0 \leqslant \sum_{k = N_0}^{n} a_k \leqslant \sum_{k = N_0}^{n} b_k }[/math]

Rosnący ciąg sum częściowych [math]\displaystyle{ A_n = \sum_{k = N_0}^{n} a_k }[/math] nie może być ograniczony od góry, bo przeczyłoby to założeniu, że szereg [math]\displaystyle{ \sum_{k = N_0}^{\infty} a_k }[/math] jest rozbieżny. Wynika stąd i z wypisanych wyżej nierówności, że również ciąg sum częściowych [math]\displaystyle{ B_n = \sum_{k = N_0}^{n} b_k }[/math] nie może być ograniczony od góry, zatem szereg [math]\displaystyle{ \sum_{k = N_0}^{\infty} b_k }[/math] jest rozbieżny.


Twierdzenie D10
Jeżeli szereg [math]\displaystyle{ \sum_{k = 1}^{\infty} \left | a_k \right | }[/math] jest zbieżny, to szereg [math]\displaystyle{ \sum_{k = 1}^{\infty} a_k }[/math] jest również zbieżny.

Dowód

Niech [math]\displaystyle{ b_k = a_k + | a_k | }[/math]. Z definicji prawdziwe jest następujące kryterium porównawcze

[math]\displaystyle{ 0 \leqslant b_k \leqslant 2 | a_k | }[/math]

Zatem z punktu 1. twierdzenia D9 wynika, że szereg [math]\displaystyle{ \sum_{k = 1}^{\infty} b_k }[/math] jest zbieżny. Z definicji wyrazów ciągu [math]\displaystyle{ \left ( b_k \right ) }[/math] mamy [math]\displaystyle{ a_k = b_k - | a_k | }[/math] i możemy napisać

[math]\displaystyle{ \sum_{k = 1}^{\infty} a_k = \sum_{k = 1}^{\infty} b_k - \sum_{k = 1}^{\infty} | a_k | }[/math]

Ponieważ szeregi po prawej stronie są zbieżne, to zbieżny jest też szereg [math]\displaystyle{ \sum_{k = 1}^{\infty} a_k }[/math]. Zauważmy, że jedynie w przypadku, gdyby obydwa szeregi po prawej stronie były rozbieżne, nie moglibyśmy wnioskować o zbieżności / rozbieżności szeregu [math]\displaystyle{ \sum_{k = 1}^{\infty} a_k }[/math], bo suma szeregów rozbieżnych może być zbieżna.


Twierdzenie D11
Niech [math]\displaystyle{ n \in \mathbb{Z}_+ }[/math]. Jeżeli wyrazy ciągu [math]\displaystyle{ (a_n) }[/math] można zapisać w jednej z postaci

  1. [math]\displaystyle{ \quad a_k = f_k - f_{k + 1} }[/math]
  2. [math]\displaystyle{ \quad a_k = f_{k - 1} - f_k }[/math]

to odpowiadający temu ciągowi szereg nazywamy szeregiem teleskopowym. Suma częściowa szeregu teleskopowego jest odpowiednio równa

  1. [math]\displaystyle{ \quad \sum_{k = m}^{n} a_k = f_m - f_{n + 1} }[/math]
  2. [math]\displaystyle{ \quad \sum_{k = m}^{n} a_k = f_{m - 1} - f_n }[/math]
Dowód
[math]\displaystyle{ \sum_{k = m}^{n} a_k = \sum_{k = m}^{n} (f_k - f_{k + 1}) = }[/math]
[math]\displaystyle{ = (f_m - f_{m + 1}) + (f_{m + 1} - f_{m + 2}) + (f_{m + 2} - f_{m + 3}) + \ldots + (f_{n - 1} - f_n) + (f_n - f_{n + 1}) = }[/math]
[math]\displaystyle{ = f_m - f_{m + 1} + f_{m + 1} - f_{m + 2} + f_{m + 2} - f_{m + 3} + \ldots + f_{n - 1} - f_n + f_n - f_{n + 1} = }[/math]
[math]\displaystyle{ = f_m + (- f_{m + 1} + f_{m + 1}) + (- f_{m + 2} + f_{m + 2}) + (- f_{m + 3} + \ldots + f_{n - 1}) + (- f_n + f_n) - f_{n + 1} = }[/math]
[math]\displaystyle{ = f_m - f_{n + 1} }[/math]


[math]\displaystyle{ \sum_{k = m}^{n} a_k = \sum_{k = m}^{n} (f_{k - 1} - f_k) = }[/math]
[math]\displaystyle{ = (f_{m - 1} - f_m) + (f_m - f_{m + 1}) + (f_{m + 1} - f_{m + 2}) + \ldots + (f_{n - 2} - f_{n - 1}) + (f_{n - 1} - f_n) = }[/math]
[math]\displaystyle{ = f_{m - 1} - f_m + f_m - f_{m + 1} + f_{m + 1} - f_{m + 2} + \ldots + f_{n - 2} - f_{n - 1} + f_{n - 1} - f_n = }[/math]
[math]\displaystyle{ = f_{m - 1} + (- f_m + f_m) + (- f_{m + 1} + f_{m + 1}) + (- f_{m + 2} + \ldots + f_{n - 2}) + (- f_{n - 1} + f_{n - 1}) - f_n = }[/math]
[math]\displaystyle{ = f_{m - 1} - f_n }[/math]


Twierdzenie D12
Następujące szeregi są zbieżne

Dowód

Punkt 1.
Dla dowodu wykorzystamy fakt, że rozpatrywany szereg jest szeregiem teleskopowym

[math]\displaystyle{ \frac{1}{k (k + 1)} = \frac{1}{k} - \frac{1}{k + 1} }[/math]

Zatem

[math]\displaystyle{ \sum^n_{k = 1} \frac{1}{k (k + 1)} = \sum^n_{k = 1} \left( \frac{1}{k} - \frac{1}{k + 1} \right) = 1 - \frac{1}{n + 1} }[/math]

Przechodząc z [math]\displaystyle{ n }[/math] do nieskończoności, dostajemy

[math]\displaystyle{ \sum^{\infty}_{k = 1} \frac{1}{k (k + 1)} = 1 }[/math]

Punkt 2.
Szereg jest identyczny z szeregiem z punktu 1., co łatwo zauważyć zmieniając zmienną sumowania [math]\displaystyle{ k = s + 1 }[/math] i odpowiednio granice sumowania.

Punkt 3.
Należy skorzystać z tożsamości

[math]\displaystyle{ \frac{1}{k^2 - 1} = \frac{1}{2} \left[ \left( \frac{1}{k} - \frac{1}{k + 1} \right) + \left( \frac{1}{k - 1} - \frac{1}{k} \right) \right] }[/math]

Punkt 4.
Ponieważ dla [math]\displaystyle{ k \geqslant 2 }[/math] prawdziwa jest nierówność

[math]\displaystyle{ 0 \lt \frac{1}{k^2} \lt \frac{1}{k^2 - 1} }[/math]

to na mocy kryterium porównawczego (twierdzenie D9) ze zbieżności szeregu [math]\displaystyle{ \sum^{\infty}_{k = 2} \frac{1}{k^2 - 1} }[/math] wynika zbieżność szeregu [math]\displaystyle{ \sum_{k = 1}^{\infty} \frac{1}{k^2} }[/math]


Twierdzenie D13
Następujące szeregi są zbieżne

Dowód

Punkt 1.

Wystarczy zauważyć, że

[math]\displaystyle{ \frac{1}{\sqrt{k}} - \frac{1}{\sqrt{k + 1}} = \frac{\sqrt{k + 1} - \sqrt{k}}{\sqrt{k} \cdot \sqrt{k + 1}} = }[/math]
[math]\displaystyle{ \quad\: = \frac{1}{\sqrt{k} \cdot \sqrt{k + 1} \cdot \left( \sqrt{k + 1} + \sqrt{k} \right)} \gt }[/math]
[math]\displaystyle{ \quad\: \gt \frac{1}{\sqrt{k} \cdot \sqrt{k + 1} \cdot 2 \sqrt{k + 1}} }[/math]
[math]\displaystyle{ \quad\: = \frac{1}{2 (k + 1) \sqrt{k}} }[/math]

Zatem

[math]\displaystyle{ \sum_{k = 1}^n \frac{1}{(k + 1) \sqrt{k}} = 2 \sum_{k = 1}^n \frac{1}{2 (k + 1) \sqrt{k}} \lt }[/math]
[math]\displaystyle{ \;\;\;\: \lt 2 \sum_{k = 1}^n \left( \frac{1}{\sqrt{k}} - \frac{1}{\sqrt{k + 1}} \right) = }[/math]
[math]\displaystyle{ \;\;\;\: = 2 \left( 1 - \frac{1}{\sqrt{n + 1}} \right) \lt }[/math]
[math]\displaystyle{ \;\;\;\: \lt 2 }[/math]

Ponieważ ciąg sum częściowych szeregu jest rosnący i ograniczony, to szereg jest zbieżny.

Punkt 2.
Korzystając z twierdzenia A37, możemy napisać oszacowanie

[math]\displaystyle{ 0 \lt \frac{\log k}{k (k + 1)} \lt \frac{2 \sqrt{k}}{k (k + 1)} \lt \frac{2}{(k + 1) \sqrt{k}} }[/math]

Zatem na mocy kryterium porównawczego ze zbieżności szeregu [math]\displaystyle{ \sum_{k = 1}^{\infty} \frac{1}{(k + 1) \sqrt{k}} }[/math] wynika zbieżność szeregu [math]\displaystyle{ \sum^{\infty}_{k = 2} \frac{\log k}{k (k + 1)} }[/math]

Punkt 3.
Zauważmy, że

[math]\displaystyle{ \frac{\log (k - 1)}{k - 1} - \frac{\log (k)}{k} = \frac{k \log (k - 1) - (k - 1) \log (k)}{k (k - 1)} = }[/math]
[math]\displaystyle{ \;\;\, = \frac{k \log \left( k \left( 1 - \frac{1}{k} \right) \right) - (k - 1) \log (k)}{k (k - 1)} = }[/math]
[math]\displaystyle{ \;\;\, = \frac{k \log (k) + k \log \left( 1 - \frac{1}{k} \right) - k \log (k) + \log (k)}{k (k - 1)} \gt }[/math]
[math]\displaystyle{ \;\;\, \gt \frac{\log (k) - k \cdot \frac{1}{k - 1}}{k (k - 1)} = }[/math]
[math]\displaystyle{ \;\;\, = \frac{\log (k)}{k (k - 1)} - \frac{1}{(k - 1)^2} }[/math]

Czyli prawdziwe jest oszacowanie

[math]\displaystyle{ \frac{\log (k)}{k (k - 1)} \lt \left[ \frac{\log (k - 1)}{k - 1} - \frac{\log (k)}{k} \right] + \frac{1}{(k - 1)^2} }[/math]

Zatem możemy napisać

[math]\displaystyle{ \sum_{k = 2}^{n} \frac{\log (k)}{k (k - 1)} \lt \sum_{k = 2}^{n} \left[ \frac{\log (k - 1)}{k - 1} - \frac{\log (k)}{k} \right] + \sum_{k = 2}^{n} \frac{1}{(k - 1)^2} }[/math]
[math]\displaystyle{ \: \lt - \frac{\log (n)}{n} + \sum_{j = 1}^{n - 1} \frac{1}{j^2} }[/math]
[math]\displaystyle{ \: \lt \sum_{j = 1}^{\infty} \frac{1}{j^2} = }[/math]
[math]\displaystyle{ \: = \frac{\pi^2}{6} }[/math]

Ponieważ ciąg sum częściowych szeregu jest rosnący i ograniczony, to szereg jest zbieżny.

Punkt 4.
Zauważmy, że

[math]\displaystyle{ \frac{1}{\log (k)} - \frac{1}{\log (k + 1)} = \frac{\log (k + 1) - \log (k)}{\log (k) \log (k + 1)} = }[/math]
[math]\displaystyle{ \;\;\, = \frac{\log \left( 1 + \frac{1}{k} \right)}{\log (k) \log (k + 1)} \lt }[/math]
[math]\displaystyle{ \;\;\, \lt \frac{1}{k \cdot \log (k) \log (k + 1)} \lt }[/math]
[math]\displaystyle{ \;\;\, \lt \frac{1}{k \cdot \log^2 \! k} }[/math]

Z drugiej strony mamy

[math]\displaystyle{ \frac{1}{\log (k - 1)} - \frac{1}{\log (k)} = \frac{\log (k) - \log (k - 1)}{\log (k - 1) \log (k)} = }[/math]
[math]\displaystyle{ \;\;\, = \frac{\log \left( 1 + \frac{1}{k - 1} \right)}{\log (k - 1) \log (k)} \gt }[/math]
[math]\displaystyle{ \;\;\, \gt \frac{1}{k \cdot \log (k - 1) \log (k)} \gt }[/math]
[math]\displaystyle{ \;\;\, \gt \frac{1}{k \cdot \log^2 \! k} }[/math]

Wynika stąd następujący ciąg nierówności

[math]\displaystyle{ \frac{1}{\log (k)} - \frac{1}{\log (k + 1)} \lt \frac{1}{k \cdot \log^2 \! k} \lt \frac{1}{\log (k - 1)} - \frac{1}{\log (k)} }[/math]


Rezultat ten wykorzystamy w pełni w przykładzie D14, a do pokazania zbieżności szeregu wystarczy nam prawa nierówność. Mamy

[math]\displaystyle{ \sum_{k = 3}^{n} \frac{1}{k \cdot \log^2 \! k} \lt \sum_{k = 3}^{n} \left[ \frac{1}{\log (k - 1)} - \frac{1}{\log (k)} \right] = }[/math]
[math]\displaystyle{ \; = \frac{1}{\log 2} - \frac{1}{\log (n)} \lt }[/math]
[math]\displaystyle{ \; \lt \frac{1}{\log 2} }[/math]

Ponieważ ciąg sum częściowych szeregu jest rosnący i ograniczony, to szereg jest zbieżny.


Przykład D14
Na przykładzie szeregu [math]\displaystyle{ \sum_{k = 3}^{\infty} \frac{1}{k \cdot \log^2 k} }[/math] pokażemy, jak należy obliczać przybliżoną wartość sumy szeregu.

Ponieważ nie jesteśmy w stanie zsumować nieskończenie wielu wyrazów, zatem najlepiej będzie podzielić szereg na dwie części

[math]\displaystyle{ \sum_{k = 3}^{\infty} \frac{1}{k \cdot \log^2 k} = \sum_{k = 3}^{m} \frac{1}{k \cdot \log^2 k} + \sum_{k = m + 1}^{\infty} \frac{1}{k \cdot \log^2 k} }[/math]


Wartość pierwszej części możemy policzyć bezpośrednio, a dla drugiej części powinniśmy znaleźć jak najlepsze oszacowanie.

Dowodząc twierdzenie D13, w punkcie 4. pokazaliśmy, że prawdziwy jest ciąg nierówności

[math]\displaystyle{ \frac{1}{\log (k)} - \frac{1}{\log (k + 1)} \lt \frac{1}{k \cdot \log^2 k} \lt \frac{1}{\log (k - 1)} - \frac{1}{\log (k)} }[/math]


Wykorzystamy powyższy wzór do znalezienia potrzebnego nam oszacowania. Sumując strony nierówności, dostajemy

[math]\displaystyle{ \sum_{k = m + 1}^{n} \left( \frac{1}{\log (k)} - \frac{1}{\log (k + 1)} \right) \lt \sum_{k = m + 1}^{n} \frac{1}{k \cdot \log^2 k} \lt \sum_{k = m + 1}^{n} \left( \frac{1}{\log (k - 1)} - \frac{1}{\log (k)} \right) }[/math]


Ponieważ szeregi po lewej i po prawej stronie są szeregami teleskopowymi, to łatwo znajdujemy, że

[math]\displaystyle{ \frac{1}{\log (m + 1)} - \frac{1}{\log (n + 1)} \lt \sum_{k = m + 1}^{n} \frac{1}{k \cdot \log^2 k} \lt \frac{1}{\log m} - \frac{1}{\log n} }[/math]


Przechodząc z [math]\displaystyle{ n }[/math] do nieskończoności, otrzymujemy oszacowanie

[math]\displaystyle{ \frac{1}{\log (m + 1)} \lt \sum_{k = m + 1}^{\infty} \frac{1}{k \cdot \log^2 k} \lt \frac{1}{\log m} }[/math]


Teraz pozostaje dodać sumę wyrazów szeregu od [math]\displaystyle{ k = 3 }[/math] do [math]\displaystyle{ k = m }[/math]

[math]\displaystyle{ \frac{1}{\log (m + 1)} + \sum_{k = 3}^{m} \frac{1}{k \cdot \log^2 k} \lt \sum_{k = 3}^{\infty} \frac{1}{k \cdot \log^2 k} \lt \frac{1}{\log m} + \sum_{k = 3}^{m} \frac{1}{k \cdot \log^2 k} }[/math]


Poniżej przedstawiamy wartości oszacowania sumy szeregu znalezione przy pomocy programu PARI/GP dla kolejnych wartości [math]\displaystyle{ m }[/math]. Wystarczy proste polecenie

for(n=1, 8, s = sum( k = 3, 10^n, 1/k/(log(k))^2 ); print("n= ", n, "   a= ", s+1/log(10^n+1), "   b= ", s+1/log(10^n) ))

Dysponując oszacowaniem reszty szeregu, znaleźliśmy wartość sumy szeregu z dokładnością 10 miejsc po przecinku.

Natomiast samo zsumowanie [math]\displaystyle{ 10^8 }[/math] wyrazów szeregu daje wynik

[math]\displaystyle{ \sum_{k = 3}^{10^8} \frac{1}{k \cdot \log^2 k} = 1.014 771 500 510 916 \ldots }[/math]

Zatem mimo zsumowania stu milionów(!) wyrazów szeregu otrzymaliśmy rezultat z dokładnością jednego(!) miejsca po przecinku. Co więcej, nie wiemy, jaka jest dokładność uzyskanego rezultatu. Znając oszacowanie od dołu i od góry, dokładność jednego miejsca po przecinku uzyskaliśmy po zsumowaniu dziesięciu(!) wyrazów szeregu.

Rozpatrywana wyżej sytuacja pokazuje, że w przypadku znajdowania przybliżonej wartości sumy szeregu ważniejsze od sumowania ogromnej ilości wyrazów jest posiadanie oszacowania nieskończonej reszty szeregu. Ponieważ wyznaczenie tego oszacowania na ogół nie jest proste, pokażemy jak ten problem rozwiązać przy pomocy całki oznaczonej.



Szeregi nieskończone i całka oznaczona

Twierdzenie D15
Jeżeli funkcja [math]\displaystyle{ f(x) }[/math] jest ciągła, dodatnia i malejąca w przedziale [math]\displaystyle{ [m, n + 1] }[/math], to prawdziwy jest następujący ciąg nierówności

[math]\displaystyle{ 0 \leqslant \int_{m}^{n + 1} f(x) d x \leqslant \sum_{k = m}^{n} f(k) \leqslant f (m) + \int_{m}^{n} f(x) d x }[/math]
Dowód

Ponieważ funkcja [math]\displaystyle{ f(x) }[/math] jest z założenia ciągła, dodatnia i malejąca, to zamieszczony niżej rysunek dobrze prezentuje problem.

D Szereg-i-calka-1.png

Przedstawiona na rysunku krzywa odpowiada funkcji [math]\displaystyle{ f(x) }[/math]. Dla współrzędnej [math]\displaystyle{ x = k }[/math] zaznaczyliśmy wartość funkcji [math]\displaystyle{ f(k) }[/math], a po lewej i prawej stronie tych punktów zaznaczyliśmy pasy o jednostkowej szerokości. Łatwo zauważamy, że

  • po lewej stronie pole pod krzywą (zaznaczone kolorem zielonym) jest większe od pola prostokąta o wysokości [math]\displaystyle{ f(k) }[/math] i jednostkowej szerokości
  • po prawej stronie pole pod krzywą (zaznaczone kolorem niebieskim) jest mniejsze od pola prostokąta o wysokości [math]\displaystyle{ f(k) }[/math] i jednostkowej szerokości

Korzystając z własności całki oznaczonej, otrzymujemy ciąg nierówności

[math]\displaystyle{ \int_{k}^{k + 1} f(x) d x \leqslant f(k) \leqslant \int_{k - 1}^{k} f(x) d x }[/math]

W powyższym wzorze występują nierówności nieostre, bo rysunek przedstawia funkcję silnie malejącą, ale zgodnie z uczynionym założeniem funkcja [math]\displaystyle{ f(x) }[/math] może być funkcją słabo malejącą.

Sumując lewą nierówność od [math]\displaystyle{ k = m }[/math] do [math]\displaystyle{ k = n }[/math], a prawą od [math]\displaystyle{ k = m + 1 }[/math] do [math]\displaystyle{ k = n }[/math], dostajemy

[math]\displaystyle{ \int_{m}^{n + 1} f (x) d x \leqslant \sum_{k = m}^{n} f (k) }[/math]
[math]\displaystyle{ \sum_{k = m + 1}^{n} f (k) \leqslant \int_{m}^{n} f (x) d x }[/math]

Dodając [math]\displaystyle{ f(m) }[/math] do obydwu stron drugiej z powyższych nierówności i łącząc je ze sobą, otrzymujemy kolejny i docelowy ciąg nierówności

[math]\displaystyle{ 0 \leqslant \int_{m}^{n + 1} f (x) d x \leqslant \sum_{k = m}^{n} f (k) \leqslant f (m) + \int_{m}^{n} f (x) d x }[/math]


Przykład D16
Rozważmy szereg [math]\displaystyle{ \sum_{k = 1}^{\infty} \frac{1}{k} }[/math].

Funkcja [math]\displaystyle{ f(x) = \frac{1}{x} }[/math] jest ciągła, dodatnia i silnie malejąca w przedziale [math]\displaystyle{ (0, + \infty) }[/math], zatem dla dowolnego [math]\displaystyle{ n \in \mathbb{Z}_+ }[/math] prawdziwe jest oszacowanie

[math]\displaystyle{ \int_{1}^{n + 1} \frac{d x}{x} \lt \sum_{k = 1}^{n} \frac{1}{k} \lt 1 + \int_{1}^{n} \frac{d x}{x} }[/math]

Przy obliczaniu całek oznaczonych Czytelnik może skorzystać ze strony WolframAlpha.

[math]\displaystyle{ \log (n + 1) \lt \sum_{k = 1}^{n} \frac{1}{k} \lt 1 + \log n }[/math]

Ponieważ

[math]\displaystyle{ \log (n + 1) = \log \left( n \left( 1 + \frac{1}{n} \right) \right) = \log n + \log \left( 1 + \frac{1}{n} \right) \gt \log n + \frac{1}{n + 1} }[/math]

to dostajemy

[math]\displaystyle{ \frac{1}{n + 1} \lt \sum_{k = 1}^{n} \frac{1}{k} - \log n \lt 1 }[/math]

Zauważmy: nie tylko wiemy, że szereg [math]\displaystyle{ \sum_{k = 1}^{\infty} \frac{1}{k} }[/math] jest rozbieżny, ale jeszcze potrafimy określić, jaka funkcja tę rozbieżność opisuje! Mamy zatem podstawy, by przypuszczać, że całki umożliwią opracowanie metody, która pozwoli rozstrzygać o zbieżności szeregów.



Twierdzenie D17 (kryterium całkowe zbieżności szeregów)
Załóżmy, że funkcja [math]\displaystyle{ f(x) }[/math] jest ciągła, dodatnia i malejąca w przedziale [math]\displaystyle{ [m, + \infty) }[/math]. Szereg [math]\displaystyle{ \sum_{k = m}^{\infty} f(k) }[/math] jest zbieżny lub rozbieżny w zależności od tego, czy funkcja pierwotna [math]\displaystyle{ F(x) = \int f (x) d x }[/math] ma dla [math]\displaystyle{ x \rightarrow \infty }[/math] granicę skończoną, czy nie.

Dowód

Nim przejdziemy do dowodu, wyjaśnimy uczynione założenia. Założenie, że funkcja [math]\displaystyle{ f(x) }[/math] jest malejąca, będzie wykorzystane w czasie dowodu twierdzenia, ale rozważanie przypadku, gdy [math]\displaystyle{ f(x) }[/math] jest rosnąca, nie ma sensu, bo wtedy nie mógłby być spełniony warunek konieczny zbieżności szeregu [math]\displaystyle{ \sum_{k = m}^{\infty} f(k) }[/math] (zobacz twierdzenie D4).

Moglibyśmy założyć bardziej ogólnie, że funkcja jest nieujemna, ale wtedy twierdzenie obejmowałoby przypadki funkcji takich, że dla pewnego [math]\displaystyle{ x_0 }[/math] byłoby [math]\displaystyle{ f(x_0) = 0 }[/math]. Ponieważ z założenia funkcja [math]\displaystyle{ f(x) }[/math] jest malejąca, zatem mielibyśmy [math]\displaystyle{ f(x) = 0 }[/math] dla [math]\displaystyle{ x \geqslant x_0 }[/math]. Odpowiadający tej funkcji szereg [math]\displaystyle{ \sum_{k = m}^{\infty} f (k) }[/math] miałby dla [math]\displaystyle{ k \geqslant x_0 }[/math] tylko wyrazy zerowe i byłby w sposób oczywisty zbieżny.

Założenie ciągłości funkcji [math]\displaystyle{ f(x) }[/math] ma zapewnić całkowalność funkcji [math]\displaystyle{ f(x) }[/math][3]. Założenie to można osłabić[4], tutaj ograniczymy się tylko do podania przykładów. Niech [math]\displaystyle{ a, b \in \mathbb{R} }[/math], mamy

[math]\displaystyle{ \int_a^b \text{sgn}(x) d x = | b | - | a | }[/math] [math]\displaystyle{ \qquad \qquad \int_0^a \lfloor x \rfloor d x = \frac{1}{2} \lfloor a \rfloor (2 a - \lfloor a \rfloor - 1) }[/math] [math]\displaystyle{ \qquad \qquad \int_{-a}^a \lfloor x \rfloor d x = - a }[/math]


Po tych uwagach dotyczących założeń możemy przejść do właściwego dowodu. Korzystając ze wzoru udowodnionego w twierdzeniu D15 i przechodząc z [math]\displaystyle{ n }[/math] do nieskończoności, dostajemy

[math]\displaystyle{ 0 \leqslant \int_{m}^{\infty} f(x) d x \leqslant \sum_{k = m}^{\infty} f(k) \leqslant f (m) + \int_{m}^{\infty} f(x) d x }[/math]


Z drugiej nierówności wynika, że jeżeli całka [math]\displaystyle{ \int_{m}^{\infty} f(x) d x }[/math] jest rozbieżna, to rosnący ciąg kolejnych całek oznaczonych [math]\displaystyle{ C_j = \int_{m}^{j} f (x) d x }[/math] nie może być ograniczony od góry (w przeciwnym wypadku całka [math]\displaystyle{ \int_{m}^{\infty} f (x) d x }[/math] byłby zbieżna), zatem również rosnący ciąg sum częściowych [math]\displaystyle{ F_j = \sum_{k = m}^{j} f(k) }[/math] nie może być ograniczony od góry, co oznacza, że szereg [math]\displaystyle{ \sum_{k = m}^{\infty} f(k) }[/math] jest rozbieżny.

Z trzeciej nierówności wynika, że jeżeli całka [math]\displaystyle{ \int_{m}^{\infty} f(x) d x }[/math] jest zbieżna, to ciąg sum częściowych [math]\displaystyle{ F_j = \sum_{k = m}^{j} f (k) }[/math] jest ciągiem rosnącym i ograniczonym od góry. Wynika stąd, że ciąg [math]\displaystyle{ F_j }[/math] jest zbieżny, zatem szereg [math]\displaystyle{ \sum_{k = m}^{\infty} f(k) }[/math] jest zbieżny.

Ponieważ zbieżność (rozbieżność) całki [math]\displaystyle{ \int_{m}^{\infty} f(x) d x }[/math] nie zależy od wyboru dolnej granicy całkowania, to wystarczy badać granicę [math]\displaystyle{ \lim_{x \to \infty} F (x) }[/math], gdzie [math]\displaystyle{ F(x) = \int f (x) d x }[/math] jest dowolną funkcją pierwotną.


Przykład D18
Przykłady zebraliśmy w tabeli. Przy obliczaniu całek nieoznaczonych Czytelnik może skorzystać ze strony WolframAlpha.

Stosując kryterium całkowe można łatwo pokazać, że szeregi

[math]\displaystyle{ \sum_{k = 1}^{\infty} \frac{1}{k^s} }[/math]
[math]\displaystyle{ \sum_{k = 2}^{\infty} \frac{1}{k \log^s \! k} }[/math]

są zbieżne dla [math]\displaystyle{ s \gt 1 }[/math] i rozbieżne dla [math]\displaystyle{ s \leqslant 1 }[/math].



Twierdzenie D19
Jeżeli funkcja [math]\displaystyle{ f(x) }[/math] jest ciągła, dodatnia i malejąca w przedziale [math]\displaystyle{ [m, \infty) }[/math] oraz

[math]\displaystyle{ R(m) = \int_{m}^{\infty} f(x) d x }[/math]
[math]\displaystyle{ S(m) = \sum_{k = a}^{m} f(k) }[/math]

gdzie [math]\displaystyle{ a \lt m }[/math], to prawdziwe jest następujące oszacowanie sumy szeregu nieskończonego [math]\displaystyle{ \sum_{k = a}^{\infty} f (k) }[/math]

[math]\displaystyle{ S(m) + R(m) - f(m) \leqslant \sum_{k = a}^{\infty} f(k) \leqslant S(m) + R(m) }[/math]
Dowód

Korzystając ze wzoru udowodnionego w twierdzeniu D15 i przechodząc z [math]\displaystyle{ n }[/math] do nieskończoności, dostajemy

[math]\displaystyle{ \int_{m}^{\infty} f(x) d x \leqslant \sum_{k = m}^{\infty} f(k) \leqslant f(m) + \int_{m}^{\infty} f(x) d x }[/math]

Czyli

[math]\displaystyle{ R(m) \leqslant \sum_{k = m}^{\infty} f(k) \leqslant f(m) + R (m) }[/math]

Odejmując od każdej ze stron nierówności liczbę [math]\displaystyle{ f(m) }[/math] i dodając do każdej ze stron nierówności sumę skończoną [math]\displaystyle{ S(m) = \sum_{k = a}^{m} f(k) }[/math], otrzymujemy

[math]\displaystyle{ S(m) + R (m) - f(m) \leqslant \sum_{k = a}^{\infty} f(k) \leqslant S(m) + R (m) }[/math]

Co należało pokazać.


Przykład D20
Twierdzenie D19 umożliwia określenie, z jaką dokładnością została wyznaczona suma szeregu. Wyznaczmy sumę szeregu [math]\displaystyle{ \sum_{k = 1}^{\infty} \frac{1}{(k + 1) \sqrt{k}} }[/math]. Mamy

[math]\displaystyle{ S(m) = \sum_{k = 1}^{m} \frac{1}{(k + 1) \sqrt{k}} }[/math]
[math]\displaystyle{ \int \frac{d x}{(x + 1) \sqrt{x}} = 2 \text{arctg} \left( \sqrt{x} \right) }[/math]
[math]\displaystyle{ R(m) = \int_{m}^{\infty} \frac{d x}{(x + 1) \sqrt{x}} = \pi - 2 \text{arctg} \left( \sqrt{m} \right) }[/math]

Zatem

[math]\displaystyle{ S(m) + R (m) - f (m) \leqslant \sum_{k = 1}^{\infty} \frac{1}{(k + 1) \sqrt{k}} \leqslant S (m) + R (m) }[/math]

Dla kolejnych wartości [math]\displaystyle{ m }[/math] otrzymujemy


W programie PARI/GP wystarczy napisać:

f(k) = 1.0/(k+1)/sqrt(k)
S(m) = sum( k = 1, m, f(k) )
R(m) = Pi - 2*atan( sqrt(m) )
for(j=1, 9, m=10^j; suma=S(m); reszta=R(m); print( "j= ", j, "   a= ", suma + reszta - f(m), "   b= ", suma + reszta ))



Prostym wnioskiem z twierdzenia D15 jest następujące
Twierdzenie D21
Niech [math]\displaystyle{ f(x) }[/math] będzie funkcją ciągłą, dodatnią i malejącą w przedziale [math]\displaystyle{ [m, + \infty) }[/math]. Jeżeli przy wyliczaniu sumy szeregu nieskończonego [math]\displaystyle{ \sum_{k = a}^{\infty} f (k) }[/math] (gdzie [math]\displaystyle{ a \lt m }[/math]) zastąpimy sumę [math]\displaystyle{ \sum_{k = m}^{\infty} f (k) }[/math] całką [math]\displaystyle{ \int_{m}^{\infty} f (x) d x }[/math], to błąd wyznaczenia sumy szeregu nie przekroczy [math]\displaystyle{ f(m) }[/math].

Dowód

Korzystając ze wzoru z twierdzenia D15 i przechodząc z [math]\displaystyle{ n }[/math] do nieskończoności, otrzymujemy

[math]\displaystyle{ \int_{m}^{\infty} f(x) d x \leqslant \sum_{k = m}^{\infty} f(k) \leqslant f(m) + \int_{m}^{\infty} f(x) d x }[/math]

Dodając do każdej ze stron nierówności wyrażenie [math]\displaystyle{ - f(m) + \sum_{k = a}^{m} f(k) }[/math], dostajemy

[math]\displaystyle{ - f(m) + \sum_{k = a}^{m} f(k) + \int_{m}^{\infty} f(x) d x \leqslant \sum_{k = a}^{\infty} f(k) \leqslant \sum_{k = a}^{m} f(k) + \int_{m}^{\infty} f(x) d x }[/math]

Skąd wynika natychmiast

[math]\displaystyle{ - f(m) \leqslant \sum_{k = a}^{\infty} f(k) - \left( \sum_{k = a}^{m} f(k) + \int_{m}^{\infty} f(x) d x \right) \leqslant 0 \lt f(m) }[/math]

Czyli

[math]\displaystyle{ \left| \sum_{k = a}^{\infty} f(k) - \left( \sum_{k = a}^{m} f(k) + \int_{m}^{\infty} f(x) d x \right) \right| \leqslant f(m) }[/math]

Co kończy dowód.


Twierdzenie D22
Niech [math]\displaystyle{ f(x) }[/math] będzie funkcją ciągłą, dodatnią i malejącą w przedziale [math]\displaystyle{ [m, + \infty) }[/math]. Jeżeli szereg [math]\displaystyle{ \sum_{k = m}^{\infty} f (k) }[/math] jest zbieżny, to dla każdego [math]\displaystyle{ n \geqslant m }[/math] prawdziwe jest następujące oszacowanie sumy częściowej szeregu [math]\displaystyle{ S(n) }[/math]

[math]\displaystyle{ S(n) = \sum_{k = m}^{n} f (k) \leqslant C - B \int_{n}^{\infty} f (x) d x }[/math]

gdzie [math]\displaystyle{ B }[/math] oraz [math]\displaystyle{ C }[/math] są dowolnymi stałymi spełniającymi nierówności

[math]\displaystyle{ B \geqslant 1 }[/math]
[math]\displaystyle{ C \geqslant f (m) + B \int_{m}^{\infty} f (x) d x }[/math]
Dowód

Z twierdzenia D15 mamy

[math]\displaystyle{ S(n) = \sum_{k = m}^{n} f (k) \leqslant f (m) + \int_{m}^{n} f (x) d x \leqslant }[/math]
[math]\displaystyle{ \;\! \leqslant f (m) + B \int_{m}^{n} f (x) d x = }[/math]
[math]\displaystyle{ \;\! = f (m) + B \int_{m}^{n} f (x) d x - B \int_{m}^{\infty} f (x) d x + B \int_{m}^{\infty} f (x) d x = }[/math]
[math]\displaystyle{ \;\! = f (m) + B \int_{m}^{n} f (x) d x - B \int^n_m f (x) d x - B \int_{n}^{\infty} f (x) d x + B \int_{m}^{\infty} f (x) d x = }[/math]
[math]\displaystyle{ \;\! = f (m) - B \int_{n}^{\infty} f (x) d x + B \int_{m}^{\infty} f (x) d x = }[/math]
[math]\displaystyle{ \;\! = \left[ f (m) + B \int_{m}^{\infty} f (x) d x \right] - B \int_{n}^{\infty} f (x) d x \leqslant }[/math]
[math]\displaystyle{ \;\! \leqslant C - B \int_{n}^{\infty} f (x) d x }[/math]


Uwaga D23
Niech [math]\displaystyle{ f(x) }[/math] będzie funkcją ciągłą, dodatnią i malejącą w przedziale [math]\displaystyle{ [m, \infty) }[/math]. Rozważmy szereg [math]\displaystyle{ \sum_{k = m}^{\infty} f (k) }[/math]. Zauważmy, że:

  • korzystając z całkowego kryterium zbieżności, możemy łatwo zbadać, czy szereg [math]\displaystyle{ \sum_{k = m}^{\infty} f (k) }[/math] jest zbieżny
  • jeżeli szereg jest zbieżny, to ponownie wykorzystując całki (twierdzenie D22), możemy znaleźć oszacowanie sumy częściowej szeregu [math]\displaystyle{ S(n) = \sum_{k = m}^{n} f(k) }[/math]

Jednak dysponując już oszacowaniem sumy częściowej szeregu [math]\displaystyle{ S(n) = \sum_{k = m}^{n} f(k) }[/math], możemy udowodnić jego poprawność przy pomocy indukcji matematycznej, a stąd łatwo pokazać zbieżność szeregu [math]\displaystyle{ \sum_{k = m}^{\infty} f(k) }[/math]. Zauważmy, że wybór większego [math]\displaystyle{ B }[/math] ułatwia dowód indukcyjny. Stałą [math]\displaystyle{ C }[/math] najlepiej zaokrąglić w górę do wygodnej dla nas wartości.


Czasami potrzebujemy takiego uproszczenia problemu, aby udowodnić zbieżność szeregów bez odwoływania się do całek. Zauważmy, że Czytelnik nawet nie musi znać całek – wystarczy, że policzy je przy pomocy programów, które potrafią to robić (np. WolframAlpha). Kiedy już znajdziemy oszacowanie sumy częściowej szeregu, nie musimy wyjaśniać, w jaki sposób je znaleźliśmy – wystarczy udowodnić, że jest ono poprawne, a do tego wystarczy indukcja matematyczna.

Zamieszczonej niżej zadania pokazują, jak wykorzystać w tym celu twierdzenie D22.


Zadanie D24
Korzystając z twierdzenia D22, znaleźć oszacowania sumy częściowej szeregów

[math]\displaystyle{ \sum_{k = 1}^{\infty} \frac{1}{k^2} \qquad }[/math] oraz [math]\displaystyle{ \qquad \sum_{k = 2}^{\infty} \frac{1}{k (\log k)^2} }[/math]
Rozwiązanie

Rozważmy szereg [math]\displaystyle{ \sum_{k = 1}^{\infty} \frac{1}{k^2} }[/math]. Funkcja [math]\displaystyle{ f(x) = \frac{1}{x^2} }[/math] jest funkcją ciągłą, dodatnią i malejącą w przedziale [math]\displaystyle{ (0, + \infty) }[/math]. Dla [math]\displaystyle{ n \gt 0 }[/math] jest

[math]\displaystyle{ \int_{n}^{\infty} \frac{1}{x^2} d x = \frac{1}{n} \qquad }[/math] (zobacz: WolframAlpha)
[math]\displaystyle{ C \geqslant 1 + \int_{1}^{\infty} \frac{1}{x^2} d x = 2 }[/math]

Zatem

[math]\displaystyle{ \sum_{k = 1}^{n} \frac{1}{k^2} \leqslant 2 - \frac{1}{n} }[/math]


Rozważmy szereg [math]\displaystyle{ \sum_{k = 2}^{\infty} \frac{1}{k (\log k)^2} }[/math]. Funkcja [math]\displaystyle{ f(x) = \frac{1}{x (\log x)^2} }[/math] jest funkcją ciągłą, dodatnią i malejącą w przedziale [math]\displaystyle{ (1, + \infty) }[/math]. Dla [math]\displaystyle{ n \gt 1 }[/math] jest

[math]\displaystyle{ \int_{n}^{\infty} \frac{1}{x (\log x)^2} d x = \frac{1}{\log n} \qquad }[/math] (zobacz: WolframAlpha)
[math]\displaystyle{ C \geqslant \frac{1}{2 \cdot (\log 2)^2} + \int_{2}^{\infty} \frac{1}{x (\log x)^2} d x = \frac{1}{2 \cdot (\log 2)^2} + \frac{1}{\log 2} = 2.483379 \ldots }[/math]

Przyjmijmy [math]\displaystyle{ C = 2.5 }[/math], zatem

[math]\displaystyle{ \sum_{k = 2}^{n} \frac{1}{k (\log k)^2} \lt 2.5 - \frac{1}{\log n} }[/math]


Zadanie D25
Stosując indukcję matematyczną, udowodnić prawdziwość oszacowania [math]\displaystyle{ \sum_{k = 1}^{n} \frac{1}{k^2} \leqslant 2 - \frac{1}{n} }[/math] i udowodnić, że szereg [math]\displaystyle{ \sum_{k = 1}^{\infty} \frac{1}{k^2} }[/math] jest zbieżny.

Rozwiązanie

Indukcja matematyczna. Łatwo zauważamy, że oszacowanie jest prawdziwe dla [math]\displaystyle{ n = 1 }[/math]. Zakładając, że oszacowanie jest prawdziwe dla [math]\displaystyle{ n }[/math], otrzymujemy dla [math]\displaystyle{ n + 1 }[/math]

[math]\displaystyle{ \sum_{k = 1}^{n + 1} \frac{1}{k^2} = \sum_{k = 1}^{n} \frac{1}{k^2} + \frac{1}{(n + 1)^2} \leqslant }[/math]
[math]\displaystyle{ \;\: \leqslant 2 - \frac{1}{n} + \frac{1}{(n + 1)^2} \leqslant }[/math]
[math]\displaystyle{ \;\: \leqslant 2 - \frac{1}{n + 1} + \left( \frac{1}{n + 1} - \frac{1}{n} + \frac{1}{(n + 1)^2} \right) = }[/math]
[math]\displaystyle{ \;\: = 2 - \frac{1}{n + 1} - \frac{1}{n (n + 1)^2} \lt }[/math]
[math]\displaystyle{ \;\: \lt 2 - \frac{1}{n + 1} }[/math]

Co kończy dowód indukcyjny. Zatem dla [math]\displaystyle{ n \geqslant 1 }[/math] mamy

[math]\displaystyle{ S(n) = \sum_{k = 1}^{n} \frac{1}{k^2} \leqslant 2 - \frac{1}{n} \lt 2 }[/math]

Czyli ciąg sum częściowych [math]\displaystyle{ S(n) = \sum_{k = 1}^{n} \frac{1}{k^2} }[/math] szeregu [math]\displaystyle{ \sum_{k = 1}^{\infty} \frac{1}{k^2} }[/math] jest rosnący i ograniczony od góry, a zatem zbieżny. Co oznacza, że szereg jest zbieżny.


Zadanie D26
Stosując indukcję matematyczną, udowodnić prawdziwość oszacowania [math]\displaystyle{ \sum_{k = 2}^{n} \frac{1}{k (\log k)^2} \lt 2.5 - \frac{1}{\log n} }[/math] i udowodnić, że szereg [math]\displaystyle{ \sum_{k = 2}^{\infty} \frac{1}{k (\log k)^2} }[/math] jest zbieżny.

Rozwiązanie

Indukcja matematyczna. Łatwo sprawdzamy, że oszacowanie jest prawdziwe dla [math]\displaystyle{ n = 2 }[/math]

[math]\displaystyle{ \sum_{k = 2}^{2} \frac{1}{k (\log k)^2} \approx 1.040684 \lt 2.5 - \frac{1}{\log 2} \approx 1.05730 }[/math]

Zakładając, że oszacowanie jest prawdziwe dla [math]\displaystyle{ n }[/math], otrzymujemy dla [math]\displaystyle{ n + 1 }[/math]

[math]\displaystyle{ \sum_{k = m}^{n + 1} \frac{1}{k (\log k)^2} = \sum_{k = m}^{n} \frac{1}{k (\log k)^2} + \frac{1}{(n + 1) \cdot (\log (n + 1))^2} \lt }[/math]
[math]\displaystyle{ \;\: \lt 2.5 - \frac{1}{\log n} + \frac{1}{(n + 1) \cdot (\log (n + 1))^2} = }[/math]
[math]\displaystyle{ \;\: = 2.5 - \frac{1}{\log (n + 1)} + \left( \frac{1}{\log (n + 1)} - \frac{1}{\log n} + \frac{1}{(n + 1) \cdot (\log (n + 1))^2} \right) = }[/math]
[math]\displaystyle{ \;\: = 2.5 - \frac{1}{\log (n + 1)} + \frac{1}{\log (n + 1)} \left( 1 - \frac{\log (n + 1)}{\log n} + \frac{1}{(n + 1) \cdot \log (n + 1)} \right) = }[/math]
[math]\displaystyle{ \;\: = 2.5 - \frac{1}{\log (n + 1)} + \frac{1}{\log (n + 1)} \left( 1 - \frac{\log \left( n \left( 1 + \frac{1}{n} \right) \right)}{\log n} + \frac{1}{(n + 1) \cdot \log (n + 1)} \right) = }[/math]
[math]\displaystyle{ \;\: = 2.5 - \frac{1}{\log (n + 1)} + \frac{1}{\log (n + 1)} \left( 1 - 1 - \frac{\log \left( 1 + \frac{1}{n} \right)}{\log n} + \frac{1}{(n + 1) \cdot \log (n + 1)} \right) \lt }[/math]
[math]\displaystyle{ \;\: \lt 2.5 - \frac{1}{\log (n + 1)} + \frac{1}{\log (n + 1)} \left( - \frac{1}{(n + 1) \log n} + \frac{1}{(n + 1) \cdot \log (n + 1)} \right) \lt }[/math]
[math]\displaystyle{ \;\: \lt 2.5 - \frac{1}{\log (n + 1)} }[/math]

Co kończy dowód indukcyjny. Zatem dla [math]\displaystyle{ n \geqslant 2 }[/math] mamy

[math]\displaystyle{ S(n) = \sum_{k = 2}^{n} \frac{1}{k (\log k)^2} \lt 2.5 - \frac{1}{\log n} \lt 2.5 }[/math]

Czyli ciąg sum częściowych [math]\displaystyle{ S(n) }[/math] szeregu [math]\displaystyle{ \sum_{k = 2}^{\infty} \frac{1}{k (\log k)^2} }[/math] jest rosnący i ograniczony od góry, a zatem zbieżny. Co oznacza, że szereg jest zbieżny.



Szeregi nieskończone i liczby pierwsze

Twierdzenie D27
Następujące szeregi są zbieżne

Dowód

Punkt 1.
Szereg jest szeregiem naprzemiennym i jego zbieżność wynika z twierdzenia D5.

Punkt 2.
Szereg jest zbieżny, bo sumy częściowe tego szeregu tworzą ciąg rosnący i ograniczony

[math]\displaystyle{ \sum_{p \leqslant n} \frac{1}{p^2} \lt \sum_{k = 2}^{\infty} \frac{1}{k^2} \lt \frac{\pi^2}{6} }[/math]

Punkt 3.
Szereg jest zbieżny, bo sumy częściowe tego szeregu tworzą ciąg rosnący i ograniczony

[math]\displaystyle{ \sum_{p \leqslant n} \frac{1}{(p - 1)^2} \lt \sum_{j = 2}^{\infty} \frac{1}{(j - 1)^2} = \sum_{k = 1}^{\infty} \frac{1}{k^2} = \frac{\pi^2}{6} }[/math]

Punkt 4.
Zbieżność wzoru wynika z kryterium porównawczego, bo dla każdego [math]\displaystyle{ p \geqslant 2 }[/math] jest

[math]\displaystyle{ 0 \lt \frac{1}{p (p - 1)} \lt \frac{1}{(p - 1)^2} }[/math]


Twierdzenie D28
Następujące szeregi są zbieżne

Dowód

Punkt 1.
Zbieżność tego szeregu udowodniliśmy w twierdzeniu B39, ale obecnie potrafimy uzyskać rezultat znacznie łatwiej. Zauważmy, że rozpatrywaną sumę możemy zapisać w postaci

[math]\displaystyle{ \sum_{p \geqslant 2} \frac{1}{p \log p} = \sum_{k = 1}^{\infty} \frac{1}{p_k \log p_k} = \frac{1}{2 \log 2} + \sum_{k = 2}^{\infty} \frac{1}{p_k \log p_k} }[/math]

Wyrażenie w mianowniku ułamka możemy łatwo oszacować. Z twierdzenia A1 mamy ([math]\displaystyle{ a = 0.72 }[/math])

[math]\displaystyle{ p_k \log p_k \gt a \cdot k \log k \cdot \log (a \cdot k \log k) = }[/math]
[math]\displaystyle{ \;\;\:\, = a \cdot k \log k \cdot (\log a + \log k + \log \log k) = }[/math]
[math]\displaystyle{ \;\;\:\, = a \cdot k \cdot (\log k)^2 \cdot \left( 1 + \frac{\log a + \log \log k}{\log k} \right) }[/math]

Ponieważ dla [math]\displaystyle{ k \gt \exp \left( \tfrac{1}{a} \right) = 4.01039 \ldots }[/math] jest

[math]\displaystyle{ \log a + \log \log k \gt 0 }[/math]

to dla [math]\displaystyle{ k \geqslant 5 }[/math] prawdziwe jest oszacowanie

[math]\displaystyle{ p_k \log p_k \gt a \cdot k \cdot (\log k)^2 }[/math]

Wynika stąd, że dla [math]\displaystyle{ k \geqslant 5 }[/math] prawdziwy jest ciąg nierówności

[math]\displaystyle{ 0 \lt \frac{1}{p_k \log p_k} \lt \frac{1}{a \cdot k \cdot (\log k)^2} }[/math]

Zatem na mocy kryterium porównawczego ze zbieżności szeregu [math]\displaystyle{ \sum_{k = 2}^{\infty} \frac{1}{k \cdot (\log k)^2} }[/math] (zobacz twierdzenie D13 p. 4 lub przykład D18 p. 5) wynika zbieżność szeregu [math]\displaystyle{ \sum_{k = 2}^{\infty} \frac{1}{p_k \log p_k} }[/math]

Punkt 2.
Zbieżność szeregu wynika z kryterium porównawczego (twierdzenie D9), bo

[math]\displaystyle{ 0 \lt \frac{1}{p^2 \log p} \lt \frac{1}{p \log p} }[/math]

Punkt 3.
Szereg jest zbieżny, bo sumy częściowe tego szeregu tworzą ciąg rosnący i ograniczony

[math]\displaystyle{ \sum_{p \leqslant n} \frac{\log p}{p (p - 1)} \lt \sum_{k = 2}^{\infty} \frac{\log k}{k (k - 1)} = 1.2577 \ldots }[/math]

Punkt 4.
Zbieżność szeregu wynika z kryterium porównawczego, bo dla każdego [math]\displaystyle{ p \geqslant 2 }[/math] jest

[math]\displaystyle{ 0 \lt \frac{\log p}{p^2} \lt \frac{\log p}{p (p - 1)} }[/math]


Twierdzenie D29
Szereg [math]\displaystyle{ \sum_{p \geqslant 2} \frac{\log p}{p} }[/math] jest rozbieżny.

Dowód

Dla potrzeb dowodu zapiszmy szereg w innej postaci

[math]\displaystyle{ \sum_{p \geqslant 2} \frac{\log p}{p} = \sum_{k = 1}^{\infty} \frac{\log p_k}{p_k} }[/math]

Zauważmy, że dla [math]\displaystyle{ k \geqslant 3 }[/math] wyrazy szeregów [math]\displaystyle{ \sum_{k = 1}^{\infty} \frac{1}{p_k} }[/math] oraz [math]\displaystyle{ \sum_{k = 1}^{\infty} \frac{\log p_k}{p_k} }[/math] spełniają nierówności

[math]\displaystyle{ 0 \leqslant \frac{1}{p_k} \leqslant \frac{\log p_k}{p_k} }[/math]

Ponieważ szereg [math]\displaystyle{ \sum_{k = 1}^{\infty} \frac{1}{p_k} }[/math] jest rozbieżny, to na mocy kryterium porównawczego (twierdzenie D9) rozbieżny jest również szereg [math]\displaystyle{ \sum_{k = 1}^{\infty} \frac{\log p_k}{p_k} }[/math]


Uwaga D30
Moglibyśmy oszacować rozbieżność szeregu [math]\displaystyle{ \sum_{p \geqslant 2} \frac{\log p}{p} }[/math] podobnie, jak to uczyniliśmy w przypadku twierdzenia B37, ale tym razem zastosujemy inną metodę, która pozwoli nam uzyskać bardziej precyzyjny rezultat.


Twierdzenie D31
Niech [math]\displaystyle{ n \in \mathbb{Z}_+ }[/math]. Prawdziwe są następujące nierówności

Dowód

Punkt 1. (indukcja matematyczna)
Łatwo sprawdzić prawdziwość nierówności dla [math]\displaystyle{ n = 1 }[/math]. Zakładając prawdziwość dla [math]\displaystyle{ n }[/math], otrzymujemy dla [math]\displaystyle{ n + 1 }[/math]

[math]\displaystyle{ (n + 1) ! = n! \cdot (n + 1) \gt }[/math]
[math]\displaystyle{ \;\;\; \gt n^n \cdot e^{- n} \cdot (n + 1) = }[/math]
[math]\displaystyle{ \;\;\; = (n + 1)^{n + 1} \cdot \frac{n^n}{(n + 1)^n} \cdot e^{- n} = }[/math]
[math]\displaystyle{ \;\;\; = (n + 1)^{n + 1} \cdot \frac{1}{\left( 1 + \frac{1}{n} \right)^n} \cdot e^{- n} \gt }[/math]
[math]\displaystyle{ \;\;\; \gt (n + 1)^{n + 1} \cdot \frac{1}{e} \cdot e^{- n} = }[/math]
[math]\displaystyle{ \;\;\; = (n + 1)^{n + 1} e^{- (n + 1)} }[/math]

Ponieważ [math]\displaystyle{ \left( 1 + \frac{1}{n} \right)^n \lt e }[/math], zatem [math]\displaystyle{ \frac{1}{\left( 1 + \frac{1}{n} \right)^n} \gt \frac{1}{e} }[/math]. Co kończy dowód punktu 1.


Punkt 2. (indukcja matematyczna)
Łatwo sprawdzić prawdziwość nierówności dla [math]\displaystyle{ n = 7 }[/math]. Zakładając prawdziwość dla [math]\displaystyle{ n }[/math], otrzymujemy dla [math]\displaystyle{ n + 1 }[/math]

[math]\displaystyle{ (n + 1) ! = n! \cdot (n + 1) \lt }[/math]
[math]\displaystyle{ \;\;\; \lt n^{n + 1} \cdot e^{- n} \cdot (n + 1) = }[/math]
[math]\displaystyle{ \;\;\; = (n + 1)^{n + 2} \cdot \frac{n^{n + 1}}{(n + 1)^{n + 1}} \cdot e^{- n} = }[/math]
[math]\displaystyle{ \;\;\; = (n + 1)^{n + 2} \cdot \left( \frac{n}{n + 1} \right)^{n + 1} \cdot e^{- n} = }[/math]
[math]\displaystyle{ \;\;\; = (n + 1)^{n + 2} \cdot \left( 1 - \frac{1}{n + 1} \right)^{n + 1} \cdot e^{- n} \lt }[/math]
[math]\displaystyle{ \;\;\; \lt (n + 1)^{n + 2} \cdot \frac{1}{e} \cdot e^{- n} = }[/math]
[math]\displaystyle{ \;\;\; = (n + 1)^{n + 2} \cdot e^{- (n + 1)} }[/math]

Ostatnia nierówność wynika z faktu, że [math]\displaystyle{ \left( 1 - \frac{1}{n + 1} \right)^{n + 1} \lt \frac{1}{e} }[/math]. Co kończy dowód punktu 2.


Twierdzenie D32
Niech [math]\displaystyle{ n \in \mathbb{Z}_+ }[/math]. Dla wykładnika, z jakim liczba pierwsza [math]\displaystyle{ p }[/math] występuje w rozwinięciu liczby [math]\displaystyle{ n! }[/math] na czynniki pierwsze, prawdziwe są oszacowania

Dowód

Punkt 1. (prawa nierówność)

Zauważmy, że

[math]\displaystyle{ W_p (n!) = \left\lfloor \frac{n}{p} \right\rfloor + \left\lfloor \frac{n}{p^2} \right\rfloor + \left\lfloor \frac{n}{p^3} \right\rfloor + \ldots \lt }[/math]
[math]\displaystyle{ \;\, \lt \frac{n}{p} + \frac{n}{p^2} + \frac{n}{p^3} + \ldots + \frac{n}{p^k} + \ldots = }[/math]
[math]\displaystyle{ \;\, = \frac{n}{p} \cdot \frac{1}{1 - \frac{1}{p}} = }[/math]
[math]\displaystyle{ \;\, = \frac{n}{p - 1} }[/math]

Punkt 1. (lewa nierówność)

Łatwo znajdujemy, że

[math]\displaystyle{ W_p (n!) = \sum_{k = 1}^{\infty} \left\lfloor \frac{n}{p^k} \right\rfloor \geqslant \left\lfloor \frac{n}{p} \right\rfloor \gt \frac{n}{p} - 1 }[/math]

Punkt 2. (prawa nierówność)

Z uzyskanego w punkcie 1. oszacowania wynika, że [math]\displaystyle{ (p - 1) W_p (n!) \lt n }[/math]. Ponieważ nierówność ta dotyczy liczb całkowitych, to możemy napisać

[math]\displaystyle{ (p - 1) W_p (n!) \leqslant n - 1 }[/math]

Skąd otrzymujemy natychmiast nierówność nieostrą [math]\displaystyle{ W_p (n!) \leqslant \frac{n - 1}{p - 1} }[/math].

Punkt 2. (lewa nierówność)

Z uzyskanego w punkcie 1. oszacowania wynika, że [math]\displaystyle{ n - p \lt p \cdot W_p (n!) }[/math]. Ponieważ nierówność ta dotyczy liczb całkowitych, to możemy napisać

[math]\displaystyle{ n - p \leqslant p \cdot W_p (n!) - 1 }[/math]

Skąd otrzymujemy natychmiast nierówność nieostrą [math]\displaystyle{ W_p (n!) \geqslant \frac{n + 1}{p} - 1 }[/math].


Twierdzenie D33
Dla dowolnego [math]\displaystyle{ n \in \mathbb{Z}_+ }[/math] prawdziwe jest następujące oszacowanie

[math]\displaystyle{ \sum_{p \leqslant n} \frac{\log p}{p - 1} - \log n \gt - 1 }[/math]
Dowód

Z oszacowania wykładnika, z jakim liczba pierwsza [math]\displaystyle{ p }[/math] występuje w rozwinięciu liczby [math]\displaystyle{ n! }[/math] na czynniki pierwsze, wynika natychmiast, że dla [math]\displaystyle{ n \geqslant 2 }[/math] mamy

[math]\displaystyle{ n! \lt \prod_{p \leqslant n} p^{n / (p - 1)} }[/math]

Ponieważ dla [math]\displaystyle{ n \geqslant 1 }[/math] jest [math]\displaystyle{ n! \gt n^n e^{- n} }[/math] (zobacz punkt 1. twierdzenia D31), to

[math]\displaystyle{ n^n e^{- n} \lt \prod_{p \leqslant n} p^{n / (p - 1)} }[/math]

Logarytmując, otrzymujemy

[math]\displaystyle{ n \log n - n \lt \sum_{p \leqslant n} \frac{n \log p}{p - 1} = n \sum_{p \leqslant n} \frac{\log p}{p - 1} }[/math]

Dzieląc strony przez [math]\displaystyle{ n }[/math], dostajemy szukaną nierówność.


Twierdzenie D34 (pierwsze twierdzenie Mertensa[5][6], 1874)
Dla dowolnego [math]\displaystyle{ n \in \mathbb{Z}_+ }[/math] prawdziwe jest następujące oszacowanie

[math]\displaystyle{ \sum_{p \leqslant n} \frac{\log p}{p} - \log n \gt - 1.755367 }[/math]
Dowód

Ponieważ

[math]\displaystyle{ \frac{1}{p - 1} = \frac{1}{p} + \frac{1}{p (p - 1)} }[/math]


to z twierdzenia D33 dostajemy

[math]\displaystyle{ \sum_{p \leqslant n} \frac{\log p}{p} + \sum_{p \leqslant n} \frac{\log p}{p (p - 1)} - \log n \gt - 1 }[/math]

Czyli

[math]\displaystyle{ \sum_{p \leqslant n} \frac{\log p}{p} - \log n \gt - 1 - \sum_{p \leqslant n} \frac{\log p}{p (p - 1)} }[/math]
[math]\displaystyle{ \;\, \gt - 1 - \sum_{p \geqslant 2} \frac{\log p}{p (p - 1)} }[/math]
[math]\displaystyle{ \;\, = - 1 - 0.755366610831 \ldots }[/math]
[math]\displaystyle{ \;\, \gt - 1.755367 }[/math]

Gdzie wykorzystaliśmy zbieżność szeregu [math]\displaystyle{ \sum_{p \geqslant 2} \frac{\log p}{p (p - 1)} }[/math] (twierdzenie D28 p. 3).


Twierdzenie D35 (pierwsze twierdzenie Mertensa[5][6], 1874)
Dla dowolnego [math]\displaystyle{ n \in \mathbb{Z}_+ }[/math] prawdziwe jest następujące oszacowanie

[math]\displaystyle{ \sum_{p \leqslant n} \frac{\log p}{p} - \log n \lt 0.386295 }[/math]
Dowód

Z oszacowania wykładnika, z jakim liczba pierwsza [math]\displaystyle{ p }[/math] występuje w rozwinięciu liczby [math]\displaystyle{ n! }[/math] na czynniki pierwsze, wynika natychmiast, że dla [math]\displaystyle{ n \geqslant 1 }[/math] mamy

[math]\displaystyle{ n! \geqslant \prod_{p \leqslant n} p^{(n + 1) / p \: - \: 1} }[/math]

Ponieważ dla [math]\displaystyle{ n \geqslant 7 }[/math] jest [math]\displaystyle{ n! \lt n^{n + 1} e^{- n} }[/math], to

[math]\displaystyle{ \prod_{p \leqslant n} p^{(n + 1) / p \: - \: 1} \lt n^{n + 1} e^{- n} }[/math]

Logarytmując, otrzymujemy

[math]\displaystyle{ \sum_{p \leqslant n} \left( \frac{n + 1}{p} - 1 \right) \cdot \log p \lt (n + 1) \cdot \log n - n }[/math]
[math]\displaystyle{ (n + 1) \sum_{p \leqslant n} \frac{\log p}{p} - \sum_{p \leqslant n} \log p \lt (n + 1) \cdot \log n - n }[/math]


Skąd natychmiast wynika, że

[math]\displaystyle{ \sum_{p \leqslant n} \frac{\log p}{p} - \log n \lt - \frac{n}{n + 1} + \frac{1}{n + 1} \cdot \log \left( \prod_{p \leqslant n} p \right) }[/math]
[math]\displaystyle{ \;\: = - 1 + \frac{1}{n + 1} + \frac{1}{n + 1} \cdot \log (P (n)) }[/math]
[math]\displaystyle{ \;\: \lt - 1 + \frac{1}{n + 1} + \frac{n \cdot \log 4}{n + 1} }[/math]
[math]\displaystyle{ \;\: = - 1 + \frac{1}{n + 1} + \log 4 - \frac{\log 4}{n + 1} }[/math]
[math]\displaystyle{ \;\: = \log 4 - 1 + \frac{1 - \log 4}{n + 1} }[/math]
[math]\displaystyle{ \;\: = \log 4 - 1 - \frac{0.386294 \ldots}{n + 1} }[/math]
[math]\displaystyle{ \;\: \lt \log 4 - 1 }[/math]
[math]\displaystyle{ \;\: = 0.386294361 \ldots }[/math]

Druga nierówność wynika z twierdzenia A9. Bezpośrednio sprawdzamy, że powyższa nierówność jest prawdziwa dla [math]\displaystyle{ n \lt 7 }[/math].


Twierdzenie D36
Dla dowolnego [math]\displaystyle{ n \in \mathbb{Z}_+ }[/math] prawdziwe jest następujące oszacowanie

[math]\displaystyle{ \sum_{p \leqslant n} \frac{\log p}{p - 1} - \log n \lt 1.141661 }[/math]
Dowód

Ponieważ

[math]\displaystyle{ \frac{1}{p} = \frac{1}{p - 1} - \frac{1}{p (p - 1)} }[/math]

to z twierdzenia D35 dostajemy

[math]\displaystyle{ \sum_{p \leqslant n} \frac{\log p}{p - 1} - \sum_{p \leqslant n} \frac{\log p}{p (p - 1)} - \log n \lt \log 4 - 1 }[/math]

Czyli

[math]\displaystyle{ \sum_{p \leqslant n} \frac{\log p}{p - 1} - \log n \lt \log 4 - 1 + \sum_{p \leqslant n} \frac{\log p}{p (p - 1)} }[/math]
[math]\displaystyle{ \;\;\: \lt \log 4 - 1 + \sum_{p \geqslant 2} \frac{\log p}{p (p - 1)} }[/math]
[math]\displaystyle{ \;\;\: = \log 4 - 1 + 0.755366610831 \ldots }[/math]
[math]\displaystyle{ \;\;\: \lt 1.141661 }[/math]


Uwaga D37

Dokładniejsze oszacowanie sumy [math]\displaystyle{ \sum_{p \leqslant n} \frac{\log p}{p} }[/math] jest dane wzorem

[math]\displaystyle{ \sum_{p \leqslant n} \frac{\log p}{p} = \log n - E + \ldots }[/math]

gdzie [math]\displaystyle{ E = 1.332582275733 \ldots }[/math]

Dla [math]\displaystyle{ n \geqslant 319 }[/math] mamy też[7]

[math]\displaystyle{ \left| \sum_{p \leqslant n} \frac{\log p}{p} - \log n + E \right| \lt \frac{1}{2 \log n} }[/math]


Uwaga D38

Dokładniejsze oszacowanie sumy [math]\displaystyle{ \sum_{p \leqslant n} \frac{\log p}{p - 1} }[/math] jest dane wzorem

[math]\displaystyle{ \sum_{p \leqslant n} \frac{\log p}{p - 1} = \log n - \gamma + \ldots }[/math]

gdzie [math]\displaystyle{ \gamma = 0.5772156649 \ldots }[/math] jest stałą Eulera.

Dla [math]\displaystyle{ n \geqslant 318 }[/math] prawdziwe jest oszacowanie[8]

[math]\displaystyle{ \left| \sum_{p \leqslant n} \frac{\log p}{p - 1} - \log n + \gamma \right| \lt \frac{1}{2 \log n} }[/math]


Uwaga D39
Dla [math]\displaystyle{ n \leqslant 10^{10} }[/math] wartości wyrażeń

[math]\displaystyle{ \sum_{p \leqslant n} \frac{\log p}{p} - \log n + E }[/math]
[math]\displaystyle{ \sum_{p \leqslant n} \frac{\log p}{p - 1} - \log n + \gamma }[/math]

są liczbami dodatnimi.


Twierdzenie D40
Prawdziwy jest następujący związek

[math]\displaystyle{ \sum_{p \geqslant 2} \frac{\log p}{p (p - 1)} = \sum_{n = 2}^{\infty} \left( \sum_{p \geqslant 2} \frac{\log p}{p^n} \right) = E - \gamma }[/math]

gdzie

  • [math]\displaystyle{ \quad \gamma = 0.577215664901532 \ldots }[/math] jest stałą Eulera[9]
  • [math]\displaystyle{ \quad E = 1.332582275733220 \ldots }[/math][10]
  • [math]\displaystyle{ \quad E - \gamma = 0.755366610831688 \ldots }[/math][11]
Dowód

Ponieważ

[math]\displaystyle{ \frac{1}{p (p - 1)} = \frac{1}{p - 1} - \frac{1}{p} }[/math]

zatem

[math]\displaystyle{ \sum_{p \leqslant n} \frac{\log p}{p (p - 1)} = \sum_{p \leqslant n} \frac{\log p}{p - 1} - \sum_{p \leqslant n} \frac{\log p}{p} = (\log n - \gamma + \ldots) - (\log n - E + \ldots) }[/math]

Przechodząc z [math]\displaystyle{ n }[/math] do nieskończoności, otrzymujemy

[math]\displaystyle{ \sum_{p \geqslant 2} \frac{\log p}{p (p - 1)} = E - \gamma }[/math]


Zauważmy teraz, że

[math]\displaystyle{ \frac{1}{p - 1} = \frac{1}{p} \cdot \frac{1}{1 - \frac{1}{p}} = }[/math]
[math]\displaystyle{ \;\! = \frac{1}{p} \cdot \left( 1 + \frac{1}{p} + \frac{1}{p^2} + \frac{1}{p^3} + \ldots + \frac{1}{p^k} + \ldots \right) = }[/math]
[math]\displaystyle{ \;\! = \frac{1}{p} + \frac{1}{p^2} + \frac{1}{p^3} + \ldots + \frac{1}{p^k} + \ldots }[/math]

Zatem

[math]\displaystyle{ \sum_{p \geqslant 2} \frac{\log p}{p (p - 1)} = \sum_{p \geqslant 2} \frac{\log p}{p} \cdot \left( \frac{1}{p} + \frac{1}{p^2} + \frac{1}{p^3} + \ldots + \frac{1}{p^k} + \ldots \right) = \sum_{n = 2}^{\infty} \left( \sum_{p \geqslant 2} \frac{\log p}{p^n} \right) }[/math]


Twierdzenie D41
Dla [math]\displaystyle{ n \geqslant 318 }[/math] prawdziwe jest oszacowanie

[math]\displaystyle{ \left| \sum_{p \leqslant n} \frac{\log p}{p - 1} - \log n + \gamma \right| \lt \frac{1}{2 \log n} }[/math]
Dowód

Należy zauważyć, że tak dokładnego oszacowania nie można udowodnić metodami elementarnymi, dlatego punktem wyjścia jest oszacowanie podane w pracy Pierre'a Dusarta[12]

[math]\displaystyle{ - \left( \frac{0.2}{\log n} + \frac{0.2}{\log^2 n} \right) \; \underset{n \geqslant 2}{\lt } \; \sum_{p \leqslant n} \frac{\log p}{p} - \log n + E \; \underset{n \geqslant 2974}{\lt } \; \frac{0.2}{\log n} + \frac{0.2}{\log^2 n} }[/math]

Ponieważ dla [math]\displaystyle{ x \gt e^2 \approx 7.389 }[/math] jest [math]\displaystyle{ 1 + \frac{1}{\log x} \lt 1.5 }[/math], to dla [math]\displaystyle{ n \geqslant 8 }[/math] mamy

[math]\displaystyle{ \frac{0.2}{\log n} + \frac{0.2}{\log^2 n} = \frac{0.2}{\log n} \left( 1 + \frac{1}{\log n} \right) \lt \frac{0.3}{\log n} }[/math]


Zatem wyjściowy układ nierówności możemy zapisać w postaci

[math]\displaystyle{ - \frac{0.3}{\log n} \; \underset{n \geqslant 8}{\lt } \; \sum_{p \leqslant n} \frac{\log p}{p} - \log n + E \; \underset{n \geqslant 2974}{\lt } \; \frac{0.3}{\log n} }[/math]


Z tożsamości

[math]\displaystyle{ \frac{1}{p} = \frac{1}{p - 1} - \frac{1}{p (p - 1)} }[/math]


wynika natychmiast, że

[math]\displaystyle{ - \frac{0.3}{\log n} \; \underset{n \geqslant 8}{\lt } \; \sum_{p \leqslant n} \frac{\log p}{p - 1} - \sum_{p \leqslant n} \frac{\log p}{p (p - 1)} - \log n + E \; \underset{n \geqslant 2974}{\lt } \; \frac{0.3}{\log n} }[/math]


Prawa nierówność

Rozważmy prawą nierówność prawdziwą dla [math]\displaystyle{ n \geqslant 2974 }[/math]

[math]\displaystyle{ \sum_{p \leqslant n} \frac{\log p}{p - 1} - \sum_{p \leqslant n} \frac{\log p}{p (p - 1)} - \log n + E \lt \frac{0.3}{\log n} }[/math]


Z twierdzenia D40 wiemy, że

[math]\displaystyle{ \sum_{p \geqslant 2} \frac{\log p}{p (p - 1)} - E = - \gamma }[/math]

Zatem

[math]\displaystyle{ \sum_{p \leqslant n} \frac{\log p}{p - 1} - \log n \lt \sum_{p \leqslant n} \frac{\log p}{p (p - 1)} - E + \frac{0.3}{\log n} }[/math]
[math]\displaystyle{ \;\;\: \lt \sum_{p \geqslant 2} \frac{\log p}{p (p - 1)} - E + \frac{0.3}{\log n} }[/math]
[math]\displaystyle{ \;\;\: = - \gamma + \frac{0.3}{\log n} }[/math]
[math]\displaystyle{ \;\;\: \lt - \gamma + \frac{0.5}{\log n} }[/math]


Bezpośrednio obliczając, sprawdzamy, że nierówność

[math]\displaystyle{ \sum_{p \leqslant n} \frac{\log p}{p - 1} - \log n \lt - \gamma + \frac{0.5}{\log n} }[/math]

jest prawdziwa dla wszystkich liczb [math]\displaystyle{ 318 \leqslant n \leqslant 3000 }[/math]


Lewa nierówność

Rozważmy teraz lewą nierówność prawdziwą dla [math]\displaystyle{ n \geqslant 8 }[/math]

[math]\displaystyle{ \sum_{p \leqslant n} \frac{\log p}{p - 1} - \sum_{p \leqslant n} \frac{\log p}{p (p - 1)} - \log n + E \gt - \frac{0.3}{\log n} }[/math]

Mamy

[math]\displaystyle{ \sum_{p \leqslant n} \frac{\log p}{p - 1} - \log n \gt \sum_{p \leqslant n} \frac{\log p}{p (p - 1)} - E - \frac{0.3}{\log n} }[/math]
[math]\displaystyle{ \;\;\, = \sum_{p \geqslant 2} \frac{\log p}{p (p - 1)} - \sum_{p \gt n} \frac{\log p}{p (p - 1)} - E - \frac{0.3}{\log n} }[/math]
[math]\displaystyle{ \;\;\, = - \gamma - \frac{0.3}{\log n} - \sum_{p \gt n} \frac{\log p}{p (p - 1)} }[/math]
[math]\displaystyle{ \;\;\, \gt - \gamma - \frac{0.3}{\log n} - \sum_{k = n + 1}^{\infty} \frac{\log k}{k (k - 1)} }[/math]
[math]\displaystyle{ \;\;\, \gt - \gamma - \frac{0.3}{\log n} - \sum_{k = n + 1}^{\infty} \frac{\log k}{(k - 1)^2} }[/math]


Korzystając kolejno z twierdzeń D15 i C18, dostajemy

[math]\displaystyle{ \sum_{p \leqslant n} \frac{\log p}{p - 1} - \log n \gt - \gamma - \frac{0.3}{\log n} - \int_{n}^{\infty} \frac{\log x}{(x - 1)^2} d x }[/math]
[math]\displaystyle{ \;\;\, = - \gamma - \frac{0.3}{\log n} - \frac{\log n}{n - 1} + \log \left( 1 - \frac{1}{n} \right) }[/math]
[math]\displaystyle{ \;\;\, \gt - \gamma - \frac{0.3}{\log n} - \frac{\log n}{n - 1} - \frac{1}{n - 1} }[/math]
[math]\displaystyle{ \;\;\, = - \gamma - \frac{0.5}{\log n} + \left( \frac{0.2}{\log n} - \frac{\log n + 1}{n - 1} \right) }[/math]
[math]\displaystyle{ \;\;\, \gt - \gamma - \frac{0.5}{\log n} }[/math]


Do znalezienia całki oznaczonej Czytelnik może wykorzystać stronę WolframAlpha. Ostatnia nierówność jest prawdziwa dla [math]\displaystyle{ n \geqslant 153 }[/math]. Bezpośrednio obliczając, sprawdzamy, że nierówność

[math]\displaystyle{ \sum_{p \leqslant n} \frac{\log p}{p - 1} - \log n \gt - \gamma - \frac{0.5}{\log n} }[/math]

jest prawdziwa dla wszystkich [math]\displaystyle{ 2 \leqslant n \leqslant 200 }[/math].


Zadanie D42
Niech [math]\displaystyle{ r = 1 - \log (2) \approx 0.30685281944 }[/math]. Pokazać, że z nierówności prawdziwej dla [math]\displaystyle{ x \geqslant 32 }[/math]

[math]\displaystyle{ \sum_{p \leqslant x} \frac{\log p}{p - 1} \lt \log x - r }[/math]

wynika twierdzenie Czebyszewa.

Rozwiązanie

Z twierdzenia D41 wiemy, że dla [math]\displaystyle{ x \geqslant 318 }[/math] jest

[math]\displaystyle{ \sum_{p \leqslant x} \frac{\log p}{p - 1} - \log x \lt - \gamma + \frac{1}{2\log x} \leqslant - \gamma + \frac{1}{2 \log (318)} = - 0.490441 \ldots \lt - 0.306852 \ldots = - r }[/math]

Zatem postulowane oszacowanie jest prawdziwe dla [math]\displaystyle{ n \geqslant 318 }[/math]. Sprawdzając bezpośrednio dla [math]\displaystyle{ 2 \leqslant x \leqslant 317 }[/math], łatwo potwierdzamy prawdziwość nierówności

[math]\displaystyle{ \sum_{p \leqslant x} \frac{\log p}{p - 1} \lt \log x - r }[/math]

dla [math]\displaystyle{ x \geqslant 32 }[/math].


Niech [math]\displaystyle{ a \in \mathbb{Z} }[/math] i [math]\displaystyle{ a \geqslant 32 }[/math]. Korzystając z twierdzenia D32, łatwo znajdujemy oszacowanie

[math]\displaystyle{ a! = p^{\alpha_1}_1 \cdot \ldots \cdot p^{\alpha_n}_n }[/math]
[math]\displaystyle{ \quad \leqslant p^{(a - 1) / (p_1 - 1)}_1 \cdot \ldots \cdot p^{(a - 1) / (p_n - 1)}_n }[/math]
[math]\displaystyle{ \quad = (p^{1 / (p_1 - 1)}_1 \cdot \ldots \cdot p^{1 / (p_n - 1)}_n)^{a - 1} }[/math]

gdzie [math]\displaystyle{ p_n \leqslant a \lt p_{n + 1} }[/math]. Oznaczając wyrażenie w nawiasie przez [math]\displaystyle{ U }[/math], mamy

[math]\displaystyle{ \log U = \frac{\log p_1}{p_1 - 1} + \ldots + \frac{\log p_n}{p_n - 1} = \sum_{p \leqslant a} \frac{\log p}{p - 1} \lt \log a - r }[/math]

gdzie skorzystaliśmy z oszacowania wskazanego w treści zadania. Zatem [math]\displaystyle{ U \lt a \cdot e^{- r} }[/math].


Przypuśćmy, że mnożymy liczbę [math]\displaystyle{ a! }[/math] przez kolejne liczby naturalne [math]\displaystyle{ a + 1, a + 2, \ldots, b - 1, b }[/math]. Możemy postawić pytanie: kiedy w rozkładzie na czynniki pierwsze liczby [math]\displaystyle{ b! }[/math] musi pojawić się nowy czynnik pierwszy? Jeżeli takiego nowego czynnika pierwszego nie ma, to

[math]\displaystyle{ a! \cdot (a + 1) \cdot \ldots \cdot b = b! }[/math]
[math]\displaystyle{ \;\;\; = p^{\beta_1}_1 \cdot \ldots \cdot p^{\beta_n}_n }[/math]
[math]\displaystyle{ \;\;\; \leqslant p^{(b - 1) / (p_1 - 1)}_1 \cdot \ldots \cdot p^{(b - 1) / (p_n - 1)}_n }[/math]
[math]\displaystyle{ \;\;\; = (p^{1 / (p_1 - 1)}_1 \cdot \ldots \cdot p^{1 / (p_n - 1)}_n)^{b - 1} }[/math]
[math]\displaystyle{ \;\;\; = U^{b - 1} }[/math]
[math]\displaystyle{ \;\;\; \lt (a \cdot e^{- r})^{b - 1} }[/math]


Jednocześnie z twierdzenia D31 wiemy, że prawdziwa jest nierówność [math]\displaystyle{ b! \gt b^b e^{- b} }[/math], zatem

[math]\displaystyle{ b^b e^{- b} \lt b! \lt \frac{(a \cdot e^{- r})^b}{a \cdot e^{-r}} }[/math]
[math]\displaystyle{ b e^{- 1} \lt \frac{a \cdot e^{- r}}{(a \cdot e^{- r})^{1 / b}} }[/math]
[math]\displaystyle{ b \lt \frac{a \cdot e^{1 - r}}{(a \cdot e^{- r})^{1 / b}} }[/math]


Ponieważ [math]\displaystyle{ e^{1 - r} = e^{\log (2)} = 2 }[/math], to

[math]\displaystyle{ b \lt \frac{2 a}{(a \cdot e^{- r})^{1 / b}} \lt 2 a }[/math]


Z oszacowania [math]\displaystyle{ b \lt 2 a }[/math] wynika, że [math]\displaystyle{ (a \cdot e^{- r})^{1 / b} \gt (a \cdot e^{-r})^{1 / 2 a} }[/math]. Możemy teraz zapisać uzyskane wyżej oszacowanie w postaci, w której prawa strona nierówności nie zależy od [math]\displaystyle{ b }[/math]

[math]\displaystyle{ b \lt \frac{2 a}{(a \cdot e^{- r})^{1 / b}} \lt \frac{2 a}{(a \cdot e^{- r})^{1 / 2 a}} }[/math]


Ponieważ [math]\displaystyle{ e^{- r} = 0.735758 \ldots }[/math], to [math]\displaystyle{ (a \cdot e^{- r})^{1 / 2 a} \gt (a / 2)^{1 / 2 a} }[/math], co pozwala uprościć uzyskane oszacowanie

[math]\displaystyle{ b \lt \frac{2 a}{(a \cdot e^{- r})^{1 / 2 a}} \lt \frac{2 a}{(a / 2)^{1 / 2 a}} }[/math]


Pokażemy, że dla [math]\displaystyle{ a \gt 303.05 }[/math]

[math]\displaystyle{ \frac{2 a}{(a / 2)^{1 / 2 a}} \lt 2 a - 5 }[/math]

Istotnie

[math]\displaystyle{ \frac{1}{(a / 2)^{1 / 2 a}} \lt 1 - \frac{5}{2 a} }[/math]
[math]\displaystyle{ \frac{a}{2} \cdot \left( 1 - \frac{5}{2 a} \right)^{2 a} \gt 1 }[/math]
[math]\displaystyle{ \frac{a}{2} \cdot \left[ \left( 1 - \frac{5}{2 a} \right)^{\tfrac{2 a}{5}} \right]^5 \gt 1 }[/math]

Wyrażenie w nawiasie kwadratowym jest funkcją rosnącą i ograniczoną (zobacz twierdzenie C17) i dla [math]\displaystyle{ a \geqslant 32 }[/math] przyjmuje wartości z przedziału [math]\displaystyle{ [0.353 \ldots, e^{- 1}) }[/math]. Zatem dla odpowiednio dużego [math]\displaystyle{ a }[/math] powyższa nierówność z pewnością jest prawdziwa. Łatwo sprawdzamy, że dla [math]\displaystyle{ a = 304 }[/math] jest

[math]\displaystyle{ \frac{a}{2} \cdot \left( 1 - \frac{5}{2 a} \right)^{2 a} = 1.003213 \ldots }[/math]

Wynika stąd, że wszystkie kolejne liczby naturalne [math]\displaystyle{ a + 1, a + 2, \ldots, b - 1, b }[/math] mogą być liczbami złożonymi co najwyżej do chwili, gdy [math]\displaystyle{ b \lt 2 a - 5 }[/math], czyli [math]\displaystyle{ b \leqslant 2 a - 6 }[/math]. Zatem w przedziale [math]\displaystyle{ (a, 2 a) }[/math] musi znajdować się przynajmniej jedna liczba pierwsza. Dla [math]\displaystyle{ a \leqslant 303 }[/math] prawdziwość twierdzenia sprawdzamy bezpośrednio.


Definicja D43
Powiemy, że liczby pierwsze [math]\displaystyle{ p, q }[/math] są liczbami bliźniaczymi (tworzą parę liczb bliźniaczych), jeżeli [math]\displaystyle{ \left | p - q \right | = 2 }[/math]


Twierdzenie D44* (Viggo Brun, 1919)
Suma odwrotności par liczb pierwszych [math]\displaystyle{ p }[/math] i [math]\displaystyle{ p + 2 }[/math], takich że liczba [math]\displaystyle{ p + 2 }[/math] jest również pierwsza, jest skończona

[math]\displaystyle{ \underset{p + 2 \in \mathbb{P}}{\sum_{p \geqslant 2}} \left( \frac{1}{p} + \frac{1}{p + 2} \right) = \left( \frac{1}{3} + \frac{1}{5} \right) + \left( \frac{1}{5} + \frac{1}{7} \right) + \left( \frac{1}{11} + \frac{1}{13} \right) + \left( \frac{1}{17} + \frac{1}{19} \right) + \ldots = B_2 }[/math]

gdzie [math]\displaystyle{ B_2 = 1.90216058 \ldots }[/math] jest stałą Bruna[13][14].


Zadanie D45
Pokazać, że istnieje nieskończenie wiele liczb pierwszych nie tworzących par liczb bliźniaczych.

Rozwiązanie

Niech [math]\displaystyle{ p }[/math] i [math]\displaystyle{ q = p + 4 }[/math] będą liczbami pierwszymi i [math]\displaystyle{ n \geqslant 1 }[/math]. Ponieważ liczby [math]\displaystyle{ p q }[/math] i [math]\displaystyle{ p + 2 }[/math] są względnie pierwsze, to z twierdzenia Dirichleta wiemy, że wśród liczb [math]\displaystyle{ a_n = p q n + (p + 2) }[/math] jest nieskończenie wiele liczb pierwszych, a jednocześnie żadna z liczb [math]\displaystyle{ a_n }[/math] nie tworzy pary liczb bliźniaczych, bo

[math]\displaystyle{ a_n - 2 = p q n + p = p (q n + 1) }[/math]
[math]\displaystyle{ a_n + 2 = p q n + (p + 4) = q (p n + 1) }[/math]

są liczbami złożonymi. Najprostsze przykłady to [math]\displaystyle{ a_n = 21 n + 5 }[/math] i [math]\displaystyle{ b_n = 77 n + 9 }[/math]

Najłatwiej wszystkie przypadki takich ciągów wyszukać w programie PARI/GP. Polecenie

for(a=1,50, for(b=3,floor(a/2), g=gcd(a,b); g1=gcd(a,b-2); g2=gcd(a,b+2); if( g==1 && g1>1 && g2>1, print("a= ", a, "   b= ",b) )))

wyszukuje wszystkie liczby dodatnie [math]\displaystyle{ a, b }[/math], gdzie [math]\displaystyle{ b \leqslant \left\lfloor \frac{a}{2} \right\rfloor }[/math], które tworzą ciągi [math]\displaystyle{ a k + b }[/math] o poszukiwanych właściwościach. Oczywiście ciągi [math]\displaystyle{ a k + (a - b) }[/math] również są odpowiednie. Przykładowo dla [math]\displaystyle{ a \leqslant 50 }[/math] mamy

[math]\displaystyle{ 15 k + 7, \quad 21 k + 5, \quad 30 k + 7, \quad 33 k + 13, \quad 35 k + 12, \quad 39 k + 11, \quad 42 k + 5, \quad 45 k + 7, \quad 45 k + 8, \quad 45 k + 22 }[/math]









Przypisy

  1. Wikipedia, Funkcja η, (Wiki-pl), (Wiki-en)
  2. Wikipedia, Funkcja dzeta Riemanna, (Wiki-pl), (Wiki-en)
  3. Twierdzenie: funkcja ciągła w przedziale domkniętym jest całkowalna w tym przedziale.
  4. W szczególności: funkcja ograniczona i mająca skończoną liczbę punktów nieciągłości w przedziale domkniętym jest w tym przedziale całkowalna.
  5. 5,0 5,1 Wikipedia, Twierdzenia Mertensa, (Wiki-pl), (Wiki-en)
  6. 6,0 6,1 Wikipedia, Franciszek Mertens, (Wiki-pl)
  7. J. B. Rosser and L. Schoenfeld, Approximate formulas for some functions of prime numbers, Illinois J. Math. 6 (1962), 64-94, (LINK)
  8. Zobacz twierdzenie D41.
  9. The On-Line Encyclopedia of Integer Sequences, A001620 - Decimal expansion of Euler's constant, (A001620)
  10. The On-Line Encyclopedia of Integer Sequences, A083343 - Decimal expansion of constant B3 (or B_3) related to the Mertens constant, (A083343)
  11. The On-Line Encyclopedia of Integer Sequences, A138312 - Decimal expansion of Mertens's constant minus Euler's constant, (A138312)
  12. P. Dusart, Estimates of Some Functions Over Primes without R.H., (LINK)
  13. Wikipedia, Stałe Bruna, (Wiki-pl), (Wiki-en)
  14. The On-Line Encyclopedia of Integer Sequences, A065421 - Decimal expansion of Viggo Brun's constant B, (A065421)